Вы находитесь на странице: 1из 61

Republic of the Philippines Co-insurance allowed, particulars of which

SUPREME COURT to be declared in the event of loss or claim.


Manila
The company hereby agrees with the insured
EN BANC (but subject to the conditions on the back
hereof, which are to be taken as a part of
G.R. No. L-5715 December 20, 1910 this policy) that if the property above
described, or any part thereof, shall be
destroyed or damaged by fire, at any time
E. M. BACHRACH, plaintiff-appellee, between the 21st day of February, 1908, and
vs. 4 o'clock in the afternoon of the 21st day of
BRITISH AMERICAN ASSURANCE COMPANY, a February, 1909, or (in case of the renewal of
corporation, defendant-appellant. this policy) at any time afterwards, so long
as, and during the period in respect of which
Haussermann, Ortigas, Cohn and Fisher, for appellant the insured shall have paid to the company,
Kincaid & Hurd and Thomas L. Hartigan, for appellee. and they shall have accepted, the sum
required for the renewal of this policy, the
company will, out of their capital stock, and
funds, pay or make good to the insured the
value of the property so destroyed, or the
JOHNSON, J.: amount of such damage thereto, to any
amount not exceeding, in respect of each or
On the 13th of July, 1908, the plaintiff commenced any of the several matters above specified,
an action against the defendant to recover the sum of the sum set opposite thereto, respectively,
P9,841.50, the amount due, deducting the salvage, and not exceeding in the whole the sum of
upon the following fire insurance policy issued by the ten thousand pesos, and also not exceeding,
defendant to the plaintiff: in any case, the amount of the insurable
interest therein of the insured at the time of
[Fire policy No. 3007499.] the happening of such fire.

This policy of insurance witnesseth, that E. In witness whereof, the British American
M. Bachrach, esq., Manila (hereinafter called Assurance Company has accused these
the insured), having paid to the undersigned, presents to be signed this 21st day of
as authorized agent of the British American February, in the year of our Lord 1908.
Assurance Company (hereinafter called the
company), the sum of two thousand pesos For the company.
Philippine currency, for insuring against loss
or damage by fire, as hereinafter mentioned, W. F. STEVENSON & Co. LTD.,
the property hereinafter described, in the
sum of several sums following, viz:
"By...............................................,
"Manager Agents."
Ten thousand pesos Philippine currency, on
goods, belonging to a general furniture
store, such as iron and brass bedsteads, And indorsed on the back the following:
toilet tables, chairs, ice boxes, bureaus,
washstands, mirrors, and sea-grass furniture The within policy and includes a
(in accordance with warranty "D" of the tariff "Calalac" automobile to the extent
attached hereto) the property of the of (P1,250) twelve hundred and
assured, in trust, on commission or for which fifty pesos Philippine currency.
he is responsible, whilst stored in the ground
floor and first story of house and dwelling Memo: Permission is hereby granted
No. 16 Calle Martinez, district 3, block 70, for the use of gasoline not to
Manila, built, ground floor of stone and or exceed 10 gallons for the above
brick, first story of hard wood and roofed automobile, but only whilst
with galvanized iron bounded in the front contained in the reservoir of the
by the said calle, on one side by Calle David car. It is further warranted that the
and on the other two sides by buildings of car be neither filled nor emptied in
similar construction and occupation. the within-described building or
this policy be null and void.
Manila, 27th February, 1908. Second. That her had made no proof of the loss set up
in his complaint for the reason that immediately after
"W. F. STEVENSON & Co. LTD., he had, on the 20th of April, 1908, given the
defendant due notice in writing of said loss, the
defendant, on the 21st of April, 1908, and thereafter
"By.................................................. on other occasions, had waived all right to require
....., proof of said loss by denying all liability under the
"Manager Agents." policy and by declaring said policy to be null and void.

The defendant answered the complaint, admitting After hearing the evidence adduced during the trial of
some of the facts alleged by the plaintiff and denying the cause, the lower court found that the defendant
others. The defendant also alleged certain facts was liable to the plaintiff and rendered a judgment
under which it claimed that it was released from all against the defendant for the sum of P9,841.50, with
obligations whatever under said policy. These special interest for a period of one year at 6 per cent, making
facts are as follows: a total of P10,431.99, with costs.

First. That the plaintiff maintained a paint and From that decision the defendant appealed and made
varnish shop in the said building where the goods the following assignments of error:
which were insured were stored.
1. The court erred in failing to hold that the use of
Second. That the plaintiff transferred his interest in the building, No. 16 Calle Martinez, as a paint and
and to the property covered by the policy to H. W. varnish shop annulled the policy of insurance.
Peabody & Co. to secure certain indebtedness due and
owing to said company, and also that the plaintiff had
transferred his interest in certain of the goods 2. The court erred in failing to hold the execution of
covered by the said policy to one Macke, to secure the chattel mortgages without the knowledge and
certain obligations assumed by the said Macke for and consent of the insurance company annulled the policy
on behalf of the insured. That the sanction of the said of insurance.
defendant had not been obtained by the plaintiff, as
required by the said policy. 3. The court erred in holding that the keeping of
gasoline and alcohol not in bottles in the building No.
Third. That the plaintiff, on the 18th of April, 1908, 16 Calle Martinez was not such a violation of the
and immediately preceding the outbreak of the conditions of the policy as to render the same null and
alleged fire, willfully placed a gasoline can containing void.
10 gallons of gasoline in the upper story of said
building in close proximity to a portion of said goods, 4. The court erred in failing to find as a fact that E.
wares, and merchandise, which can was so placed by M. Bachrach, the insured, willfully placed a gasoline
the plaintiff as to permit the gasoline to run on the can containing about 10 gallons of gasoline in the
floor of said second story, and after so placing said upper story of said building, No. 16 Calle Martinez, in
gasoline, he, the plaintiff, placed in close proximity close proximity to a portion of the goods, wares, and
to said escaping gasoline a lighted lamp containing merchandise stored therein, and that said can was so
alcohol, thereby greatly increasing the risk of fire. placed by said Bachrach as to permit the gasoline to
run on the floor of said second story.
Fourth. That the plaintiff made no proof of the loss
within the time required by condition five of said 5. The court erred in failing to find as a fact that E.
policy, nor did the insured file a statement with he M. Bachrach, after placing said gasoline can in close
municipal or any other judge or court of the goods proximity to the goods, wares, and merchandise
alleged to have been in said building at the time of covered by the policy of insurance, the he (Bachrach)
the alleged fire, nor of the goods saved, nor the loss placed in close proximity to said escaping gasoline a
suffered. lighted lamp containing alcohol, thereby greatly
increasing the risk of fire.
The plaintiff, after denying nearly all of the facts set
out in the special answer of the defendant, alleged: 6. The court erred in holding that the policy of
insurance was in force at the time of said fire, and
First. That he had been acquitted in a criminal action that the acts or omissions on the part of the insured
against him, after a trial duly and regularly had, upon which cause, or tended to cause, the forfeiture of the
a charge of arson, based upon the same alleged facts policy, were waived by the defendant.
set out in the answer of the defendant.
7. The court erred in holding the defendant liable for It may be added that there was no provision in the
the loss under the policy.lawphil.net policy prohibiting the keeping of paints and varnishes
upon the premises where the insured property was
8. The court erred in refusing to deduct from the loss stored. If the company intended to rely upon a
sustained by Bachrach the value of the automobile, condition of that character, it ought to have been
which was saved without damage. plainly expressed in the policy.

9. The court erred in refusing to grant the motion for With reference to the second above assignment of
a new trial. error, the defendant and appellant contends that the
lower court erred in failing to hold that the execution
of the said chattel mortgage, without the knowledge
10. The court erred in refusing to enter judgment in and consent of the insurance company and without
favor of the defendant and against the plaintiff. receiving the sanction of said company, annulled the
said policy of insurance.
With reference to the first above assignment of error,
the lower court in its decision said: With reference to this assignment of error, upon
reading the policy of insurance issued by the
It is claimed that either gasoline or alcohol defendant to the plaintiff, it will be noted that there
was kept in violation of the policy in the is no provision in said policy prohibiting the plaintiff
bodega containing the insured property. The from placing a mortgage upon the property insured,
testimony on this point is somewhat but, admitting that such a provision was intended, we
conflicting, but conceding all of the think the lower court has completely answered this
defendant's claims, the construction given to contention of the defendant. He said, in passing upon
this claim by American courts would not this question as it was presented:
justify the forfeiture of the policy on that
ground. The property insured consisted It is claimed that the execution of a chattel
mainly of household furniture kept for the mortgage on the insured property violated
purpose of sale. The preservation of the what is known as the "alienation clause,"
furniture in a salable condition by retouching which is now found in most policies, and
or otherwise was incidental to the business. which is expressed in the policies involved in
The evidence offered by the plaintiff is to cases 6496 and 6497 by a purchase imposing
the effect that alcohol was used in preparing forfeiture if the interest in the property pass
varnish for the purpose of retouching, though from the insured. (Cases 6496 and 6497, in
he also says that the alcohol was kept in which are involved other action against other
store and not in the bodega where the insurance companies for the same loss as in
furniture was. It is well settled that the the present action.)
keeping of inflammable oils on the premises,
though prohibited by the policy, does not
void it if such keeping is incidental to the This clause has been the subject of a vast
business. Thus, where a furniture factory number of judicial decisions (13 Am. & Eng.
keeps benzine for the purposes of operation Encyc. of Law, 2d ed., pp. 239 et seq.), and
(Davis vs. Pioneer Furniture Company, 78 N. it is held by the great weight of authority
W. Rep., 596; Faust vs. American Fire that the interest in property insured does not
Insurance Company, 91 Wis., 158), or where pass by the mere execution of a chattel
it is used for the cleaning machinery (Mears mortgage and that while a chattel mortgage
vs. Humboldt Insurance Company, 92 Pa. St., is a conditional sale, there is no alienation
15; 37 Am. Rep., 647), the insurer can not on within the meaning of the insurance law until
that ground avoid payment of loss, though the mortgage acquires a right to take
the keeping of the benzine on the premises possession by default under the terms of the
is expressly prohibited. These authorities mortgage. No such right is claimed to have
also appear sufficient to answer the accrued in the case at bar, and the
objection that the insured automobile alienation clause is therefore inapplicable.
contained gasoline and that the plaintiff on
one occasion was seen in the bodega with a With reference to the third assignment of error above
lighted lamp. The first was incidental to the noted, upon a reading of the decision of the lower
use of the insured article and the second court it will be found that there is nothing in the
being a single instance falls within the decision of the lower court relating to the facts stated
doctrine of the case last cited. in this assignment of error, neither is there any
provision in the policy relating to the facts alleged in
said assignment of error.
Assignment of error numbers 4 and 5 above noted may Regardless of the question whether the
be considered together. plaintiff's letter of April 20 (Exhibit B) was a
sufficient compliance with the requirement
The record discloses that some time prior to the that he furnish notice of loss, the fact
commencement of this present action, a criminal remains that on the following day the
action was commenced against the plaintiff herein in insurers replied by a letter (Exhibit C)
the Court of First Instance of the city of Manila, in declaring that the "policies were null and
which he was charged with willfully and maliciously void," and in effect denying liability. It is
burning the property covered by the policy in the well settled by a preponderance of
present case. At the conclusion of the criminal action authorities that such a denial is a waiver of
and after hearing the evidence adduced during the notice of loss, because if the "policies are
trial, the lower court, with the assistance of two null and void," the furnishing of such notice
assessors, found that the evidence was insufficient to would be vain and useless. (13 Am. & Eng.
show beyond peradventure of doubt that the Encyc. of Law, 347, 348, 349.) Besides,
defendant was guilty of the crime. The evidence "immediate notice" is construed to mean only
adduced during the trial of the criminal cause was within a reasonable time.
introduced as evidence in the present cause. While
the evidence shows some very peculiar and suspicious Much the same may be said as to the
circumstances concerning the burning of the goods objection that the insured failed to furnish
covered by the said policy, yet, nevertheless, in view to the insurers his books and papers or to
of the findings of the lower court and in view of the present a detailed statement to the "juez
apparent conflict in the testimony, we can not find municipal," in accordance with article 404 of
that there is a preponderance of evidence showing the Code of Commerce. The last-named
that the plaintiff did actually set fire or cause fire to provision is similar to one appearing in many
be set to the goods in question. The lower court, in American policies requiring a certificate
discussing this question, said: from a magistrate nearest the loss regarding
the circumstance thereof. A denial of
As to the claim that the loss occurred liability on other grounds waives this
through the voluntary act of the insured, we requirement (O'Niel vs. Buffalo Fire
consider it unnecessary to review the Insurance Company, 3 N. Y., 122; Peoria
evidence in detail. That was done by another Marine Ins. Co. vs. Whitehill, 25 Ill., 382), as
branch of this court in disposing of the well as that relating to the production of
criminal prosecution brought against the books and papers (Ga. Home Ins. Co. vs.
insured, on the same ground, based mainly Goode & Co., 95 Va., 751; 66 Jur. Civ., 16).
on the same evidence. And regardless of Besides, the insured might have had
whether or not the judgment in that difficulty in attempting to comply with this
proceeding is res adjudicata as to anything clause, for there is no longer an official here
here, we are at least of the opinion that the with the title of "juez municipal."
evidence to establish this defense should not
be materially less convincing than that Besides the foregoing reasons, it may be added that
required in order to convict the insured of there was no requirement in the policy in question
the crime of arson. (Turtell vs. Beamount, 25 that such notice be given.
Rev. Rep., 644.) In order to find that the
defense of incendiarism was established With reference to the assignments of error numbers
here, we would be obliged, therefore, in 7, 9, and 10, they are too general in their character
effect to set aside the findings of the judge to merit consideration.
and assessors in the criminal cause, and this
we would be loath to do even though the
evidence now produced were much stronger With reference to the eight assignment of error above
than it is. noted, the defendant and appellant contends that he
was entitled to have the amount of his responsibility
reduced by the full value (P1,250) of the said
With reference to the sixth assignment of error above automobile.
noted, to wit:itc@alf That the court erred in holding
that the policy of insurance was in force at the time
of said fire and that the acts or omissions on the part It does not positively appear of record that the
of the insured which caused or tended to cause a automobile in question was not included in the other
forfeiture of the policy were waived by the policies. It does appear that the automobile was
defendant, the lower court, in discussing this saved and was considered as a part of the salvaged. It
question, said: is alleged that the salvage amounted to P4,000,
including the automobile. This amount (P4,000) was
distributed among the different insurers and the
amount of their responsibility was proportionately Life Insurance Co. But all the information contained
reduced. The defendant and appellant in the present in the applications was furnished the agent by
case made no objection at any time in the lower court Bernardo Argente.
to that distribution of the salvage. The claim is now
made for the first time. No reason is given why the Pursuant to his application, Bernardo Argente was
objection was not made at the time of the distribution examined by Dr. Cesareo Sta. Ana, a medical
of the salvage, including the automobile, among all of examiner for the West Coast Life Insurance Co., on
the insurers. The lower court had no opportunity to February 10, 1925, in the office of the Customs House.
pass upon the question now presented for the first The result of such examination was recorded in the
time. The defendant stood by and allowed the other Medical Examiner's Report, and with the exception of
insurers to share in the salvage, which he claims now the signature of Bernardo Argente, was in the hand-
wholly belonged to him. We think it is now too late to writing of Doctor Sta. Ana. But the information or
raise the question. answers to the questions contained on the face of the
Medical Examiner's Report were furnished the doctor
For all the foregoing reasons, we are of the opinion by the applicant, Bernardo Argente.
that the judgment of the lower court should be
affirmed, and it is hereby ordered that judgment be Pursuant to her application, Vicenta de Ocampo, wife
entered against the defendant and in favor of the of the plaintiff, was examined by Dr. Cesareo Sta. Ana
plaintiff for the sum of P9,841.50, with interest at the on February 10, 1925, at her residence in Manila. The
rate of 6 per cent from the 13th of July, 1908, with result of the medical examination, including among
costs. So ordered. other things, the answers given by Vicenta de Ocampo
to the questions propounded to her by the physician,
Republic of the Philippines appears in the Medical Examiner's Report.
SUPREME COURT
Manila On May 9, 1925, Bernardo Argente and his wife
submitted to the West Coast Life Insurance Co. an
EN BANC amended application for insurance, increasing the
amount thereof to P15,000, and asked that the policy
G.R. No. L-24899 March 19, 1928 be dated May 15, 1925. The amended application was
accompanied by the documents entitled "Short Form
Medical Report." In both of these documents appear
BERNARDO ARGENTE, plaintiff-appellant, certain questions and answers.
vs.
WEST COAST LIFE INSURANCE CO., defendant-
appellee. A temporary policy for P15,000 was issued to Bernardo
Argente and his wife as of May 15, but it was not
delivered to Bernardo Argente until July 2, 1925,
Abad Santos, Camus, Delgado & Recto for appellant. when the first quarterly premium on the policy was
Gibbs & McDonough and Roman Ozaeta for appellee. paid. In view of the fact that more than thirty days
had elapsed since the applicants were examined by
MALCOLM, J.: the company's physician, each of them was required
to file a certificate of health before the policy was
This is an action upon a joint life insurance policy for delivered to them.
P15,000 issued by the defendant, the West Coast Life
Insurance Co., on May 15, 1925, in favor of the On November 18, 1925, Vicenta de Ocampo died of
plaintiff, Bernardo Argente, and his wife, Vicenta de cerebral apoplexy. Thereafter Bernardo Argente
Ocampo, the latter having died on November 18, presented a claim in due form to the West Coast Life
1925. Fraud in obtaining the policy was pleaded by Insurance Co. for the payment of the sum of P15,000
way of special defense. On the issue thus suggested, the amount of the joint life Insurance policy.
the court adopted the theory of the defendant, and Following investigation conducted by the Manager of
held the insurance policy null and void, with the result the Manila office of the insurance company, it was
that the complaint was dismissed, with costs. apparently disclosed that the answers given by the
insured in their medical examinations with regard to
On February 9, 1925, Bernardo Argente signed an their health and previous illness and medical
application for joint insurance with his wife in the attendance were untrue. For that reason, the West
sum of P2,000. The wife, Vicenta de Ocampo, signed Coast Life Insurance Co. refused to pay the claim of
a like application for the same policy. Both Bernardo Argente, and on May 25, 1926, wrote him to
applications, with the exception of the names and the the effect that the claim was rejected because the
signatures of the applicants, were written by Jose insurance was obtained through fraud and
Geronimo del Rosario, an agent for the West Coast misrepresentation.
It is admitted that it appears in the Medical Examiner's of fact thus raised, the trial judge found with the
Report that Bernardo Argente, in response to the insurance company. In so doing, we believe that His
question asked by the medical examiner, "Have you Honor gave proper inclination to the weight of the
ever consulted a physician for, or have you ever proof. There appears no motive whatever on the part
suffered from any ailment or disease of, the brain or of Doctor Sta. Ana to falsify the Medical Examiner's
nervous system?" answered "No." To the question, Reports and thereby not only jeopardize his career as
"Have you consulted a physician for any ailment or a physician, but also gravely implicate himself
disease not included in your above answer," answered criminally.
"Yes. Nature of Ailment, Disease or Injury. Scabies,
Number of attacks 1, Date 1911. Duration 1 month, What has heretofore been stated in this decision is
Severity Fair, results and, if within five years, name gleaned to a great extent the carefully prepared
and address of every physician consulted. Dr. P. decision of the trial judge, the Honorable George R.
Guazon. Cured. Dr. Guazon is dead now." And to the Harvey. The court found from the evidence that the
question, "What physician or physicians, if any, not representations made by Bernardo Argente and his
named above, have you consulted or been treated by, wife in their applications to the defendant for life
within the last five years and for what illness or insurance were false with respect to their estate of
ailment? (If none, so state)" answered "No." It is, health during the period of five years preceding the
however, not disputed that on January 10, 11, and 13, date of such applications, and that they knew the
1923, Bernardo Argente was confined in the Philippine representations made by them in their applications
General Hospital where he was treated by Dr. Agerico were false. The court further found from the evidence
B. M. Sison for cerebral congestion and Bell's Palsy. that the answers given by Bernardo Argente and his
wife at the time of the medical examination by Doctor
It is further admitted that it appears in the Medical Sta. Ana were false with respect to the condition of
Examiner's Report that Vicenta de Ocampo, in their health at that time and for a period of several
response to the question asked by the medical years prior thereto. Based on these findings which
examiner, "How frequently, if at all, and in what must here be accepted since the stenographic
quantity do you use beer, wine, spirits or other transcript is incomplete, the question arises as to the
intoxicants?" answered "Beer only in small quantities estate of the law in relation thereto.
occasionally." To the question, "Have you ever
consulted a physician for or have you ever suffered One ground for the rescission of a contract of
from any ailment or disease of the brain or nervous insurance under the Insurance Act is "a concealment,"
system?" answered "No." To the question, "What which in section 25 is defined as "A neglect to
physician or physicians, if any, not named above, have communicate that which a party knows and ought to
you consulted or been treated by, within the last five communicate." Appellant argues that the alleged
years and for what illness or ailment? (If none, so concealment was immaterial and insufficient to avoid
state)" answered "None." And to the question, "Are the policy. We cannot agree. In an action on a life
you in good health as far as you know and believe?" insurance policy where the evidence conclusively
answered "Yes." It is, however, not disputed that shows that the answers to questions concerning
Vicenta de Ocampo was taken by a patrolman, at the diseases were untrue, the truth of falsity of the
request of her husband, Bernardo Argente, on May 19, answers become the determining factor. In the policy
1924, to the Meisic police station, and from there was was procured by fraudulent representations, the
transferred to the San Lazaro Hospital. In San Lazaro contract of insurance apparently set forth therein was
Hospital, her case was diagnosed by the admitting never legally existent. It can fairly be assumed that
physician as "alcoholism," but later Doctor Domingo had the true facts been disclosed by the assured, the
made a diagnosis of probable "manic-depressive insurance would never have been granted.
psychosis," and still, later in Mary Chiles Hospital,
made a final diagnosis of "phycho-neurosis."
In Joyce, The Law of Insurance, second edition,
volume 3, Chapter LV, is found the following:
The plaintiff, Bernardo Argente, while readily
conceding most of the facts herein narrated, yet
alleges that both he and his wife revealed to the Concealment exists where the assured has
company's physician. Doctor Sta. Ana, all the facts knowledge of a fact material to the risk, and
concerning the previous illnesses and medical honesty, good faith, and fair dealing requires
attendance, but that Doctor Sta. Ana, presumably that he should communicate it to the
acting in collusion, with the insurance agent, Jose assured, but he designated and intentionally
Geronimo del Rosario, failed to record them in the with holds the same.
medical reports. The evidence on these points
consists of the testimony of the plaintiff and his Another rule is that if the assured undertakes
subordinate clerk, Apolonio Espiritu, on the one hand, to state all the circumstances affecting the
and of the testimony of Doctor Sta. Ana and Jose risk, a full and fair statement of all is
Geronimo del Rosario on the other. On the question required.
It is also held that the concealment must, in The basis of the rule vitiating the contract in
the absence of inquiries, be not only case of concealment is that it misleads or
material, but fraudulent, or the fact must deceives the insurer into accepting the risk,
have been intentionally withheld; so it is or accepting it at the rate of premium agreed
held under English law that if no inquiries are upon. The insurer, relying upon the belief
made and no fraud or design to conceal that the assured will disclose every material
enters into the concealment the contract is within his actual or presumed knowledge, is
not avoided. And it is determined that even misled into a belief that the circumstance
though silence may constitute withheld does not exist, and he is thereby
misrepresentation or concealment it is not induced to estimate the risk upon a false
itself necessarily so as it is a question of fact. basis that it does not exist. The principal
Nor is there a concealment justifying a question, therefore, must be, Was the
forfeiture where the fact of insanity is not assurer misled or deceived into entering a
disclosed no questions being asked contract obligation or in fixing the premium
concerning the same. . . . of insurance by a withholding of material
information of facts within the assured's
But it would seem that if a material fact is knowledge or presumed knowledge?
actually known to the assured, its
concealment must of itself necessarily be a It therefore follows that the assurer in
fraud, and if the fact is one which the assuming a risk is entitled to know every
assured ought to know, or is presumed to material fact of which the assured has
know, the presumption of knowledge ought exclusive or peculiar knowledge, as well as
to place the assured in the same position as all material facts which directly tend to
in the former case with relation to material increase the hazard or risk which are known
facts; and if the jury in such cases find the by the assured, or which ought to be or are
fact material, and one tending to increase presumed to be known by him. And a
the risk, it is difficult to see how the concealment of such facts vitiates the
inference of a fraudulent intent or policy. "It does not seem to be necessary . .
intentional concealment can be avoided. . that the . . . suppression of the truth should
And it is declared that if a material fact have been willful." If it were but an
concealed by assured it is equivalent to a inadvertent omission, yet if it were material
false representation that it does not exist to the risk and such as the plaintiff should
and that the essentials are the truth of the have known to be so, it would render the
representations whether they were intended policy void. But it is held that if untrue or
to mislead and did insurer accept them as false answers are given in response to
true and act upon them to his prejudice. So inquiries and they relate to material facts
it is decided that under a stipulation voiding the policy is avoided without regard to the
the policy for concealment or knowledge or fraud of assured, although
misrepresentation of any material fact or if under the statute statements are
his interest is not truly stated or is either representations which must be fraudulent to
than the sole and unconditional ownership avoid the policy. So under certain codes the
the facts are unimportant that insured did important inquiries are whether the
not intend to deceive or withhold concealment was willful and related to a
information as to encumbrances even though matter material to the risk.
no questions were asked. And if insured
while being examined for life insurance and xxx xxx xxx
knowing that she had heart disease, falsely
stated that she was in good health, and
though she could not read the application, it If the assured has exclusive knowledge of
was explained to her and the questions asked material facts, he should fully and fairly
through an interpreter, and the application disclose the same, whether he believes them
like the policy contained and provision that material or not. But notwithstanding this
no liability should be incurred unless the general rule it will not infrequently happen,
policy was delivered while the insured was in especially in life risks, that the assured may
good health, the court properly directed a have a knowledge actual or presumed of
verdict for the insurer, though a witness who material facts, and yet entertain an honest
was present at the examination testified that belief that they are not material. . . . The
the insured was not asked whether she had determination of the point whether there
heart disease. has or has not been a material concealment
must rest largely in all cases upon the form
of the questions propounded and the exact
xxx xxx xxx
terms of the contract. Thus, where in "may." Nevertheless, there are two answers to the
addition to specifically named diseases the problem as propounded. The first is that the
insured was asked whether he had any California law as construed by the code examiners, at
sickness within ten years, to which he whose recommendation it was adopted, conceded
answered "No," and it was proven that within that "A failure to exercise the right (of rescission),
that period he had a slight of pharyngitis, it cannot, of course, prejudice any defense to the action
was held a question properly for the jury which the concealment may furnish." (Codes of
whether such an inflammation of the throat California annotated; Tan Chay Heng vs. West Coast
was a "sickness" within the intent of the Life Insurance Company [1927], p. 80, ante.) The
inquiry, and the court remarked on the second answer is that the insurance company more
appeal decision that if it could be held as a than one month previous to the commencement of
matter of law that the policy was thereby the present action wrote the plaintiff and informed
avoided, then it was a mere device on the him that the insurance contract was void because it
part of insurance companies to obtain money had been procured through fraudulent
without rendering themselves liable under representations, and offered to refund to the plaintiff
the policy. . . . the premium which the latter had paid upon the
return of the policy for cancellation. As held in
. . . The question should be left to the jury California as to a fire insurance policy, where any of
whether the assured truly represented the the material representations are false, the insurer's
state of his health so as not mislead or tender of the premium and notice that the policy is
deceive the insurer; and if he did not deal a canceled, before the commencement of suit thereon,
good faith with insurer in that matter, that operate to rescind the contract of insurance. (Rankin
the inquiry should be made, Did he know the vs. Amazon Insurance Co. [1891], 89 Cal., 203.)
state of his health so as to be able to furnish
a proper answer to such questions as are We are content to rest our judgment on the findings
propounded? A Massachusetts case, if of the trial court, and on the law governing those
construed as it is frequently cited, would be facts, with the result that the various assignments of
opposed to the above conclusion; but, on the error are found to be without persuasive merit.
contrary, it sustains it, for the reason that
symptoms of consumption had so far Judgment affirmed, with the costs of this instance
developed themselves within a few months against the appellant.
prior to effecting the insurance as to induce
a reasonable belief that the applicant had
that fatal disease, and we should further Republic of the Philippines
construe this case as establishing the rule SUPREME COURT
that such a matter cannot rest alone upon Manila
the assured's belief irrespective of what is a
reasonable belief, but that it ought to be EN BANC
judged by the criterion whether the belief is
one fairly warranted by the circumstances. A G.R. No. L-12465 May 29, 1959
case in Indiana, however, holds that if the
assured has some affection or ailment of one
or more of the organs inquired about so well- YU PANG CHENG alias YU PANG CHING, petitioner,
defined and marked as to materially derange vs.
for a time the functions of such organ, as in THE COURT OF APPEALS, ET AL., respondents.
the case of Bright's disease, the policy will
be avoided by a nondisclosure, irrespective M. de la Rosa and Yuseco, Abdon, Yuseco and Narvasa
of the fact whether the assured knew of such for petitioner.
ailment or not. . . . Perkin and Ponce Enrile for respondents.

Lastly, appellant contends that even if the insurance BAUTISTA ANGELO, J.:
company had a right to rescind the contract, such
right cannot now be enforced in view of the provisions Plaintiff brought this action to collect from defendant
of section 47 of the Insurance Act providing the sum of P10,000.00, value of an insurance policy
"Whenever a right to rescind a contract of insurance taken upon the life of one Yu Pang Eng, plus interest
is given to their insurer by provision of this chapter, thereon at the legal rate, the sum of P10,000.00 as
such right must be exercised previous to the moral damages the further sum of P3,000.00 as
commencement of an action on the contract." This attorney's fees, and the costs of action.
section was derived from section 2583 of the
California Civil Code, but in contrast thereto, makes
use of the imperative "must" instead of the permissive
Defendant, in its answer, set up the defense that the Vertigo, Dizziness, Fainting-spells or
insured was guilty of misrepresentation and Unconscious? No.
concealment of material facts in that he gave false
and untruthful answers to certain questions asked him Cancer, Tumors or Ulcers of any kind? No.
in his application for insurance which were material
to the risk insured against and have the effect of
avoiding the insurance policy. 15. Have you ever consulted any physician
not included in any of the above answers?
Give names and address or physicians list
After trial, the court rendered judgment ordering ailments or accidents and date. No.
defendant to pay plaintiff the sum of P10,000.00,
with legal interest thereon from the filing of the
complaint, plus the sum of P2,000.00 as attorney's It appears that the insured entered the Chinese
fees, and the costs of suit. On appeal, the Court of General Hospital for medical treatment on January
Appeals reversed the decision of the trial court, 29, 1950 having stayed there up to February 11, 1950.
holding that the insured was guilty of concealment of Upon entering the hospital, he complained of
material facts which relieves defendant from liability. dizziness, anemia, abdominal pains and tarry stools,
Hence the present petition for review. and in the evening of his admission he had several
abdominal pains and his discharges were with black
tarry stools and felt dizzy and weak. The history of his
On September 5, 1950, Yu Pang Eng submitted parts illness shows that the same "started a year ago as
II and III of his application for insurance consisting of frequent dizziness." An X-Ray picture of his stomach
the medical declaration made by him to the medical was taken and the diagnosis made of him by his
examiner of defendant and the medical examiner's doctors showed that his illness was "peptic ulcer,
report. On September 7, he submitted part I of his bleeding."
application which is the declaration made by him to
an agent of defendant, and on September 8, based on
said application, and upon payment of the first It should be noted that the insured's confinement in
premium in the sum of P591.70, defendant issued to the Chinese General Hospital took place from January
the insured Policy No. 812858. 29, 1950 to February 11, 1950, whereas his
application for insurance wherein he stated his
answer to the questions propounded to him by the
On December 27, 1950, the insured entered St. Luke's examining physician of defendant was submitted to
Hospital for medical treatment but he died on defendant on September 5, 1950. It is apparent that
February 27, 1951. According to the death certificate, when the insured gave his answers regarding his
he died of "infiltrating medullary carcinoma, Grade 4, previous ailment, particularly with regard to
advanced cardiac and of lesser curvature, stomach "Gastritis, Ulcer of the Stomach or any disease of that
metastases spleen." Plaintiff, brother and beneficiary organ" and "Vertigo, Dizziness, Fainting-spells or
of the insured, demanded from the defendant the Unconsciousness", he concealed the ailment of which
payment of the proceeds of the insurance policy and he was treated in the Chinese General, Hospital which
when the demand was refused, he brought the precisely has direct connection with the subject of
present action. the questions propounded. The negative answers
given by the insured regarding his previous ailment,
The issue to be determined is whether the insured is or his concealment of the fact that he was
guilty of concealment of some facts material to the hospitalized and treated for sometime of peptic ulcer
risk insured against which has the effect of avoiding and had suffered from "dizziness, anemia, abdominal
the policy as found by respondent court. pains and tarry stools", deprived defendant of the
opportunity to make the necessary inquiry as to the
The insured, in his application for insurance, nature of his past illness so that as it may form its
particularly in his declarations to the examining estimate relative to the approval of his application.
physician, stated the following in answering the Had defendant been given such opportunity,
questions propounded to him: considering the previous illness of the insured as
disclosed by the record of the Chinese General
Hospital, defendant would probably had never
14. Have you ever had any of the following consented to the issuance of the policy in question. In
diseases or symptoms? Each question must be fact, according to the death certificate, the insured
read and answered "Yes" or "No". died of "infiltrating medullary carcinoma, Grade 4,
advanced cardiac and of lesser curvature, stomach
xxx xxx xxx metastases spleen", which may have direct
connection with his previous illness.
Gastritis, Ulcer of the Stomach or any
disease of that organ? No. Our Insurance law provides that " A neglect to
communicate that which a party knows and ought to
communicate, is called concealment" (Section 25, Act > Upon payment of the premiums due, the policy was
No. 2427). Whether intentional or unintentional, the reinstated, but in the application for reinstatement,
concealment entitles the insurer to rescind the Henson did not disclose the fact that he had been
contract of insurance (Section 26). Our law even previously diagnosed for pyelonephritis, enlarged
requires the insured to communicate to the insurer all liver and hernia. He also did not disclose that he had
facts within his knowledge which are material to the been examined by a physician.
contract and which the other party has not the means
of ascertaining (Section 27), and the materiality is to > In 1956, Henson died, and his beneficiaries claim
be determined not by the event but solely by the was rejected by Philamlife on the ground of
probable and reasonable influence of the facts upon concealment.
the party to whom the communication is due (Section
30).
> The company then filed for
rescission. Beneficiaries contend that the intent to
In the case of Argente vs. West Coast Life Insurance conceal must be proven to warrant rescission.
Co., 51 Phil., 725 this Court said:

One ground for the rescission of a contract


of insurance under the insurance Act is "a
concealment", which in section 25 is defined Issue:
"A neglect to communicate that which a
party knows and ought to communicate."
Appellant argues that the concealment was
immaterial and insufficient to avoid the Whether or not there is need to prove intent to
policy. We cannot agree. In an action on a conceal to warrant rescission.
life insurance policy where the evidence
conclusively shows that the answers to
questions concerning diseases were untrue, Held:
the truth or falsity of the answers become
the determining factor. If the policy was NO.
procured by fraudulent representations, the
contract of insurance apparently set forth Sec. 26 provides that a concealment whether
therein was never legally existent. It can intentional or unintentional entitles the injured
fairly be assumed that had the true facts party to rescind the contract of insurance. And
been disclosed by the assured, the insurance aside from this, intent, being a state of the mind is
would never have been granted. hard to prove.

Upon the foregoing reasons, we are persuaded to


conclude that respondent court did err in declaring
the policy ineffective on the ground of concealment
and in relieving appellee from liability thereunder. According to Sec. 30 of the Insurance
Code: Materiality is to be determined not by the
event, but solely by the probable and reasonable
Wherefore, the decision appealed from is affirmed, influence of the facts upon the party to whom the
with costs against petitioner-appellant. communication is due, in forming his estimate of the
disadvantages of the proposed contract, or in making
Henson v. Philamlife - Concealment his inquiries. In essence therefore, the insured need
not have died of the very diseases he had failed to
56 OG 7328 reveal to the insurance company. It is sufficient that
his non-revelation had misled the insurer in forming
its estimate of the disadvantages of the proposed
policy reinstatement or in making its inquiries, in
order to entitle the latter to rescind the contract.
Facts:
SECOND DIVISION
> Celestino Henson was insured by Philamlife in 1954
upon his application or a 20-yr endowment life policy. G.R. No. 176652, June 04, 2014

> In 1955, the policy lapsed due to non-payment of AUGUSTO C. SOLIMAN, Petitioner, v. JUANITO C.
the premiums. FERNANDEZ, IN HIS CAPACITY AS RECEIVER OF SMC
PNEUMATICS (PHILS.), INC., Respondent.
DECISION admitted. Said Order was received by counsel for the
[respondent] on September 21, 2004 but to date, said
PEREZ, J.: counsel has not taken any step for the further
prosecution of this case.
Before this Court is a Petition for Review on WHEREFORE, for failure to prosecute let this case be,
Certiorari1 under Rule 45 of the Rules of Court as it is hereby DISMISSED.15cralawlawlibrary
assailing the 27 July 2006 Decision 2 and the 12
February 2007 Resolution of the Sixteenth Division of
the Court of Appeals (CA) in CA-G.R. CV No. 84983. Respondent filed a Motion for Reconsideration16 but it
The Decision reversed the Orders of the Regional Trial was denied by the trial court in its 22 April 2005
Court (RTC) of Quezon City, Branch 83, dated 31 Order. Respondent filed a Notice of Appeal17 seeking
January 20053 and 22 April 2005,4 which dismissed the the reversal of the Orders of the RTC.
complaint filed by Juanito C. Fernandez (respondent)
against Augusto C. Soliman (petitioner) in Civil Case In his Brief,18 respondent argued that it is the duty of
No.Q-04-52183 and denied respondents motion for the Branch Clerk of Court to set a case for pre-trial.
reconsideration. Respondent hinged this argument on the Supreme
Court Resolution entitled Guidelines to be Observed
Culled from the records are the following antecedent by Trial Court Judges and Clerks of Court in the
facts:chanroblesvirtuallawlibrary Conduct of Pre-Trial and Use of Deposition-Discovery
Measures,19 wherein the Court ruled
On 10 March 2003, SMC Pneumatics Philippines, Inc. that:chanroblesvirtuallawlibrary
(SMC Pneumatics) filed a Motion for Appointment of
Management Committee before the RTC (Special Within five (5) days from the date of filing of reply,
Commercial Court) of Calamba City, Branch 34, the plaintiff must promptly move ex parte that the
docketed as RTC SEC Case No. 44-2003-C.5 It was case be set for pre-trial conference. If the plaintiff
consolidated with SEC Case No. 50-2003-C and SEC No. fails to file said motion within the given period, the
49-2003. The latter two cases refer to the involuntary Branch COC shall issue a notice of pre-trial.
dissolution cases filed by SMC Pneumatics.6 As a
result, the RTC issued an Order7 appointing Respondent contended that the Guidelines set by the
respondent as the Receiver pending the hearing on Supreme Court has effectively relaxed Rule 18,
the composition and appointment of the members of Section 1 of the Revised Rules of Court, which states
the Management Committee.8 that it shall be the duty of the plaintiff to promptly
move ex parte that the case be set for pre-trial.
All the assets, affairs and operations of SMC
Pneumatics were placed under receivership. 9 Finding merit in the contentions advanced by
Respondent discovered that two (2) of the vehicles respondent, the appellate court held that the lower
owned by the SMC Pneumatics are still in the court need not immediately dismiss the case for
possession of the petitioner. Respondent demanded failure of respondent to file a motion to set the case
that the petitioner return the vehicles. For failure of for pre-trial because the Branch Clerk of Court should
petitioner to surrender possession, respondent filed a
have issued a Notice of Pre-Trial. The dispositive
Complaint for Recovery of Personal Properties with portion of the assailed Decision of the CA
Writ of Replevin10 before the RTC-QC Branch 83. read:chanroblesvirtuallawlibrary
The lower court issued a Writ of Replevin and
subsequently, a Writ of Seizure11 was issued. WHEREFORE, premises considered, the appeal is
Petitioner filed a Motion for Extension of Time to File GRANTED. The assailed Orders are hereby REVERSED
Answer.12 Thereafter, he filed an Urgent Motion to and SET ASIDE. Let this case be REMANDED to the
Admit Answer.13 The Answer was admitted in an Regional Trial Court of Quezon City, Branch 83 for
further proceedings.20cralawlawlibrary
Order14 dated 20 August 2004.

In his Answer, petitioner maintained that the receiver Aggrieved by the Decision, petitioner filed with the
is not entitled to the possession of the subject CA a Motion for Reconsideration of the 27 July 2006
vehicles. As president of SMC Pneumatics he insisted Decision.21 Petitioners Motion for Reconsideration
that he is entitled to the possession and use thereof. was denied in a Resolution22 of the CA dated 12
February 2007. Petitioner elevated the case to this
On 31 January 2005, the RTC issued the following Court by filing the present Petition for Review on
Order:chanroblesvirtuallawlibrary Certiorari.

Per Order dated August 20, 2004, [petitioners] Petitioner raised in issue the CA jurisdiction to
Urgent Motion Admit Answer dated June 10, 2004 was entertain respondents appeal from the order of
granted and the Answer attached thereto was dismissal of the RTC. He contends that since the
respondents appeal from the RTC order of dismissal exercised and not be abused, as there must be
raised a question purely of law, the same was within sufficient reason to justify its extinctive effect on the
the exclusive appellate jurisdiction of the Supreme plaintiffs cause of action. Deferment of proceedings
Court.23 He maintained that the Decision of the CA may be tolerated so that the court, aimed at a just
should therefore be deemed null and void ab initio. and inexpensive determination of the action, may
adjudge cases only after a full and free presentation
Respondent, on the other hand, emphasized that of all the evidence by both parties. In this regard,
petitioner cannot raise the issue of jurisdiction in the courts are reminded to exert earnest efforts to
present Petition for Review on Certiorari under Rule resolve the matters before them on the merits, and
45 of the Rules of Court. He argued that petitioner adjudicate the case in accord with the relief sought
should have filed instead a special civil action under by the parties so that appeals may be discouraged;
Rule 65. For such error, he insisted that the instant otherwise, in hastening the proceedings, they further
petition should be dismissed outright. Respondent delay the final settlement of the case.
further pointed out that petitioner never questioned
the jurisdiction of the CA while the case was pending
Petitioner argued that the appellate court mistakenly
before the appellate court. He even invoked its
concluded that the trial court need not immediately
jurisdiction when it prayed for the reconsideration of
dismiss the case for failure of the respondent to file a
the assailed decision. Petitioner should not be
motion to set the case for pre-trial. He alleged that a
allowed to assail the CAs jurisdiction after he failed
closer reading of the Regional Trial Court Order29
to get what he wanted.
would reveal that the Order simply stated that
respondent did not take any step for the further
We deem it unnecessary to pass upon these questions
prosecution of the case. He noted that any step for
thoroughly because, whether we deal with the
the further prosecution of the case is not necessarily
proceedings before us as one for review on certiorari
limited to the setting of the case for pre-trial. The
of the Decision of the CA, or as a direct appeal from
phrase may include an equally significant, available
the order of dismissal of the RTC, the result is the
remedy and course of action such as a motion for a
same.24
judgment on the pleadings or for summary judgment.
He maintained that the failure to take any of the
We find it proper to delve into the more important
three (3) available courses of action prompted the
issue to be resolved, that is, whether the trial court
trial court to conclude that the respondent has not
was correct in dismissing the complaint of the
taken any step for the further prosecution of the case
plaintiff for failure to prosecute. We do so to avoid
and to dismiss the same for failure to prosecute.
the invocation of procedural rules for observance of
yet another rule on technicality.
Such contention is speculative. We cannot presume
that the respondent had the intention of availing of
It has long been established and settled that the
the remedies of motion for judgment on the pleadings
question of whether a case should be dismissed for
or summary judgment but failed to file the same. The
failure to prosecute is mainly addressed to the sound
fact remains that the respondent had the option to
discretion of the trial court.25 Pursuant to Rule 17,
move for pre-trial and if he fails to do so as he did,
Section 3 of the Rules of Court, a court can dismiss a
the branch clerk of court had the duty to have the
case on the ground of failure to prosecute. The true
case set for pre-trial. Moreover, the period of more
test for the exercise of such power is whether, under
than four (4) months or from 21 September 2004 up
the prevailing circumstances, the plaintiff is culpable
to 31 January 2005 may not be considered an
for want of due diligence in failing to proceed with
unreasonable length of time to warrant the terminal
reasonable promptitude.26 As to what constitutes
consequence of dismissal of the case.
unreasonable length of time, this Court has ruled
that it depends on the circumstances of each
To be sure, the dismissal of the case cannot be for
particular case and that the sound discretion of the
respondents failing to take any step for further
court in the determination of the said question will
prosecution of this case because the further step is
not be disturbed, in the absence of patent abuse.27
not his, but for the clerk of court, to take.
The Court, however, in the case of Belonio v.
Rodriguez,28 held that:chanroblesvirtuallawlibrary
In Malayan Insurance Co, Inc. v. Ipil International,
Inc.,30 this Court held that the failure of a plaintiff to
The power of the trial court to dismiss an action for prosecute the action without any justifiable cause
non-prosequitur is not without its limits. If a pattern within a reasonable period of time will give rise to the
or scheme to delay the disposition of the case or a presumption that he is no longer interested to obtain
wanton failure to observe the mandatory requirement from the court the relief prayed for in the complaint.
of the rules on the part of the plaintiff is not present, The presumption is not, by any means, conclusive
as in this case, courts should not wield their authority because the plaintiff, on a motion for reconsideration
to dismiss. Indeed, while the dismissal rests on the of the order of dismissal, may allege and establish a
prerogative of the trial court, it must soundly be justifiable cause for such failure.
BENGZON, J.:
We also note that in the trial court, petitioner as
defendant was in delay in filing his answer yet the Suit to recover on a fire insurance policy. The insurer
court showed some leniency in admitting his answer presented several defenses in the Manila court of first
despite of the delay. We find no reason why instance. After trial, it was required to pay.
respondent as plaintiff should not be granted the
same leniency for his failure to move for pre-trial. For
after all, and to underscore the point, the resolution On appeal to the Courts of Appeal, the judgment was
of the Court in A.M. No. 03-1-09-SC31 provides that: affirmed.
Within five (5) days from date of filing of the reply,
the plaintiff must move ex parte that the case be set This is now a revision on certiorari, upon the insurer's
for pre-trial conference. If the plaintiff fails to file insistence on two of its main defenses: prescription
said motion within the given period, the Branch Clerk and breach of warranty.
of Court shall issue a notice of pre-trial. Dismissal of
the case for failure to prosecute is not the result The principal of facts on which adjudication may rest
stated in the rule. The trial court is required to are these:
proceed to pre-trial through the notice of pre-trial
and setting the case for pre-trial by the Branch Clerk
of Court. On April 15, 1952, the defendant General Insurance
and Surety Corporation issued its insurance Policy No.
On a final note, we emphasize that in the absence of 471, insuring against fire, for one year, the stock in
a pattern or scheme to delay the disposition of the trade of the Central Pomade Factory owned by Ng
case or a wanton failure to observe the mandatory Hua, the court insured. The next day, the Pomade
requirement of the rules on the part of the plaintiff, factory building burned, resulting in destruction by
as in the case at bar, courts should decide to dispense fire of the insured properties. Ng Hua claimed
with rather than wield their authority to dismiss.32 indemnity from the insurer. The policy covered
This is in line with the time-honoured principle that damages up to P10,000.00; but after some
cases should be decided only after giving all parties negotiations and upon suggestion of the Manila
the chance to argue their causes and defenses. Adjustment Company, he reduced the claim of
Technicality and procedural imperfections should P5,000.00. Nevertheless, the defendant insurer
thus not serve as basis of decisions.33 refused to pay for various reasons, namely (a) action
was not filed in time; (b) violation of warranty; (c)
WHEREFORE, in light of the foregoing, the instant submission of fraudulent claim; and (f) failure to pay
Petition for Review on Certiorari is DENIED. The 27 the premium.
July 2006 Decision of the Court of Appeals in CA-G.R.
CV No. 84983 and its 12 February 2007 Resolution The aforesaid Policy No. 471 contains this stipulation
denying petitioners Motion for Reconsideration are on the back thereof;.
hereby AFFIRMED.
3. The insured shall give notice to the
SO ORDERED. company of any insurance or insurances
already affected, or which may subsequently
be effected, covering any of the property
Republic of the Philippines hereby insured, and unless such notice be
SUPREME COURT given and the particulars of such insurance
Manila or insurances be stated in or endorsed on this
Policy by or on behalf of the Company before
EN BANC the occurrence of any loss or damage, all
benefits under the policy shall be forfeited.
(Emphasis ours.)
G.R. No. L-14373 January 30, 1960
The face of the policy bore the annotation: "Co-
GENERAL INSURANCE AND SURETY CORPORATION, Insurance Declared NIL"
petitioner,
vs.
NG HUA, respondent. It is undenied that Ng Hua had obtained fire insurance
on the same goods, for the same period of time, in
the amount of P20,000.00 from General Indemnity Co.
Jose P. Bengzon, Guido Advincula and Potenciano However, the Court of Appeals referring to the
Villegas, Jr., petitioner. annotation and overruling the defense, held that
there was no violation of the above clause, inasmuch
Crispin D. Baizas for respondent. as "co-insurance exists when a condition of the policy
requires the insured to bear ratable proportion of the Indeed, this concealment and violation was expressly
loss when the value of the insured property exceeds set up as a special defense in the answer. Yet plaintiff
the face value of the policy," hence there is no co- did not, in avoidance, reply nor assert such
insurance here. knowledge. And it is doubtful whether the evidence
on the point would be admissible under the pleadings.
Discussion Undoubtedly, co-insurance exists under (See Rule 11, sec. 1.)
the condition described by the appellate court. But
that is one kind of co-insurance. It is not the only All the above considerations lead to the conclusion
situation where co-insurance exists. Other insurers of that the defendant insurer successfully established its
the same property against the same hazard are defense of warranty breach or concealment of the
sometimes referred as co-insurers and the ensuing other insurance and/or violation of the provision of
combination as co-insurance.1 And considering the the policy above-mentioned.
terms of the policy which required the insured to
declare other insurances, the statement in question Having reached the conclusion, we deem it
must be deemed to be a statement (warranty) binding unnecessary to discuss the other defenses.
on both insurer and insured, that there were no other
insurance on the property. Remember it runs "Co-
Insurance declared"; emphasis on the last word. If Wherefore, the judgment under review will be
"Co-Insurance" means that the Court of Appeals says, revoked, and the defendant insurer (herein
the annotation served no purpose. It would even be petitioner) acquitted from all the liability under the
contrary to the policy itself, which in its clause No. policy. Costs against respondent. So ordered.
17 made the insured a co-insurer for the excess of the
value of the property over the amount of the policy.

The annotation then, must be deemed to be a Republic of the Philippines


warranty that the property was not insured by any SUPREME COURT
other policy. Violation thereof entitles the insurer to Manila
rescind. (Sec. 69. Insurance Act) Such
misrepresentation is fatal in the light of our views in THIRD DIVISION
Santa Ana vs. Commercial Union Assurance Company,
Ltd., 55 Phil., 329. The materiality of non-disclosure
of other insurance policies is not open to doubt.

Furthermore, even if the annotations were G.R. No. 92492 June 17, 1993
overlooked, the defendant insurer would still be free
from liability because there is no question that the THELMA VDA. DE CANILANG, petitioner,
policy issued by General Indemnity had not been vs.
stated in nor endorsed on Policy No. 471 of HON. COURT OF APPEALS and GREAT PACIFIC LIFE
defendant. And as stipulated in the above-quoted ASSURANCE CORPORATION, respondents.
provisions of such policy "all benefit under this policy
shall be forfeited."2
Simeon C. Sato for petitioner.

To avoid the dissastrous effect of the


FELICIANO, J.:
misrepresentation or concealment of the other
insurance policy, Ng Hua alleges "actual knowledge"
on the part of General insurance of the fact that he On 18 June 1982, Jaime Canilang consulted Dr.
had taken out additional insurance with General Wilfredo B. Claudio and was diagnosed as suffering
Indemnity. He does not say when such knowledge was from "sinus tachycardia." The doctor prescribed the
acquired or imparted. If General Insurance know following fro him: Trazepam, a tranquilizer; and
before issuing its policy or before the fire, such Aptin, a beta-blocker drug. Mr. Canilang consulted
knowledge might overcome the insurer's defense.3 the same doctor again on 3 August 1982 and this time
However, the Court of Appeals found no evidence of was found to have "acute bronchitis."
such knowledge. We have read the pages of the
stenographic notes cited by Ng Hua and we all gather On next day, 4 August 1982, Jaime Canilang applied
is evidence of the existence of the Insurance General for a "non-medical" insurance policy with respondent
Indemnity Company. As to knowledge of General Great Pacific Life Assurance Company ("Great
Insurance before issuance of its policy or the fire, Pacific") naming his wife, Thelma Canilang, as his
there was none. beneficiary.1 Jaime Canilang was issued ordinary life
insurance Policy No. 345163, with the face value of
P19,700, effective as of 9 August 1982.
On 5 August 1983, Jaime Canilang died of "congestive whether or not concealment was
heart failure," "anemia," and "chronic anemia."2 intentionally made, was not
Petitioner, widow and beneficiary of the insured, applicable to Canilang's case as that
filed a claim with Great Pacific which the insurer law became effective only on 1
denied on 5 December 1983 upon the ground that the June 1985.
insured had concealed material information from it.
On appeal by Great Pacific, the Court of Appeals
Petitioner then filed a complaint against Great reversed and set aside the decision of the Insurance
Pacific with the Insurance Commission for recovery of Commissioner and dismissed Thelma Canilang's
the insurance proceeds. During the hearing called by complaint and Great Pacific's counterclaim. The
the Insurance Commissioner, petitioner testified that Court of Appealed found that the use of the word
she was not aware of any serious illness suffered by "intentionally" by the Insurance Commissioner in
her late husband3 and that, as far as she knew, her defining and resolving the issue agreed upon by the
husband had died because of a kidney disorder.4 A parties at pre-trial before the Insurance
deposition given by Dr. Wilfredo Claudio was Commissioner was not supported by the evidence;
presented by petitioner. There Dr. Claudio stated that the issue agreed upon by the parties had been
that he was the family physician of the deceased whether the deceased insured, Jaime Canilang, made
Jaime Canilang5 and that he had previously treated a material concealment as the state of his health at
him for "sinus tachycardia" and "acute bronchitis."6 the time of the filing of insurance application,
Great Pacific for its part presented Dr. Esperanza justifying respondent's denial of the claim. The Court
Quismorio, a physician of Appeals also found that the failure of Jaime
and a medical underwriter working for Great Canilang to disclose previous medical consultation
Pacific.7 She testified that the deceased's insurance and treatment constituted material information
application had been approved on the basis of his which should have been communicated to Great
medical declaration.8 She explained that as a rule, Pacific to enable the latter to make proper inquiries.
medical examinations are required only in cases The Court of Appeals finally held that the Ng Gan Zee
where the applicant has indicated in his application case which had involved misrepresentation was not
for insurance coverage that he has previously applicable in respect of the case at bar which involves
undergone medical consultation and concealment.
hospitalization.9
Petitioner Thelma Canilang is now before this Court
In a decision dated 5 November 1985, Insurance on a Petition for Review on Certiorari alleging that:
Commissioner Armando Ansaldo ordered Great
Pacific to pay P19,700 plus legal interest and 1. . . . the Honorable Court of
P2,000.00 as attorney's fees after holding that: Appeals, speaking with due
respect, erred in not holding that
1. the ailment of Jaime Canilang the issue in the case agreed upon
was not so serious that, even if it between the parties before the
had been disclosed, it would not Insurance Commission is whether or
have affected Great Pacific's not Jaime Canilang "intentionally"
decision to insure him; made material concealment in
stating his state of health;
2. Great Pacific had waived its right
to inquire into the health condition 2. . . . at any rate, the non-
of the applicant by the issuance of disclosure of certain facts about his
the policy despite the lack of previous health conditions does not
answers to "some of the pertinent amount to fraud and private
questions" in the insurance respondent is deemed to have
application; waived inquiry thereto. 11

3. there was no intentional The medical declaration which was set out in the
concealment on the part of the application for insurance executed by Jaime Canilang
insured Jaime Canilang as he had read as follows:
thought that he was merely
suffering from a minor ailment and MEDICAL DECLARATION
simple cold; 10 and
I hereby declare that:
4. Batas Pambansa Blg. 847 which
voids an insurance contract,
(1) I have not been confined in any
hospital, sanitarium or infirmary,
nor receive any medical or surgical
advice/attention within the last
five (5) years.

(2) I have never been treated nor


consulted a physician for a heart
condition, high blood pressure,
cancer, diabetes, lung, kidney,
stomach disorder, or any other
physical impairment.

(3) I am, to the best of my


knowledge, in good health.

EXCEPTIONS:

______________________________
______________________________
____________________

GENERAL DECLARATION

I hereby declare that all the


foregoing answers and statements
are complete, true and correct. I
hereby agree that if there be any
fraud or misrepresentation in the
above statements material to the
risk, the INSURANCE COMPANY
upon discovery within two (2) years
from the effective date of We note that in addition to the negative statements
insurance shall have the right to made by Mr. Canilang in paragraph 1 and 2 of the
declare such insurance null and medical declaration, he failed to disclose in the
void. That the liabilities of the appropriate space, under the caption "Exceptions,"
Company under the said that he had twice consulted Dr. Wilfredo B. Claudio
Policy/TA/Certificate shall accrue who had found him to be suffering from "sinus
and begin only from the date of tachycardia" and "acute bronchitis."
commencement of risk stated in
the Policy/TA/Certificate,
provided that the first premium is The relevant statutory provisions as they stood at the
paid and the Policy/TA/Certificate time Great Pacific issued the contract of insurance
is delivered to, and accepted by me and at the time Jaime Canilang died, are set out in
in person, when I am in actual good P.D. No. 1460, also known as the Insurance Code of
health. 1978, which went into effect on 11 June 1978. These
provisions read as follows:
Signed at Manila his 4th day of
August, 1992. Sec. 26. A neglect to communicate
that which a party knows and ought
to communicate, is called a
concealment. I
l
l
xxx xxx xxx e
g
Sec. 28. Each party to a contract of i
insurance must communicate to the b
other, in good faith, all factors l
within his knowledge which are e
material to the contract and as to We agree with the Court of Appeals that the
which he makes no warranty, and information which Jaime Canilang failed to disclose
which the other has not the means was material to the ability of Great Pacific to
of ascertaining. (Emphasis estimate the probable risk he presented as a subject
supplied) of life insurance. Had Canilang disclosed his visits to
his doctor, the diagnosis made and medicines
Under the foregoing provisions, the information prescribed by such doctor, in the insurance
concealed must be information which the concealing application, it may be reasonably assumed that Great
party knew and "ought to [have] communicate[d]," Pacific would have made further inquiries and would
that is to say, information which was "material to the have probably refused to issue a non-medical
contract." The test of materiality is contained in insurance policy or, at the very least, required a
Section 31 of the Insurance Code of 1978 which reads: higher premium for the same coverage. 15 The
materiality of the information withheld by Great
Pacific did not depend upon the state of mind of
Sec. 31. Materially is to be Jaime Canilang. A man's state of mind or subjective
determined not by the event, but belief is not capable of proof in our judicial process,
solely by the probable and except through proof of external acts or failure to
reasonable influence of the facts act from which inferences as to his subjective belief
upon the party to whom the may be reasonably drawn. Neither does materiality
communication is due, in forming depend upon the actual or physical events which
his estimate of the disadvantages ensue. Materiality relates rather to the "probable
of the proposed contract, or in and reasonable influence of the facts" upon the party
making his inquiries. (Emphasis to whom the communication should have been made,
supplied) in assessing the risk involved in making or omitting to
make further inquiries and in accepting the
"Sinus tachycardia" is considered present "when the application for insurance; that "probable and
heart rate exceeds 100 beats per minute." 13 The reasonable influence of the facts" concealed must, of
symptoms of this condition include pounding in the course, be determined objectively, by the judge
chest and sometimes faintness and weakness of the ultimately.
person affected. The following elaboration was
offered by Great Pacific and set out by the Court of The insurance Great Pacific applied for was a "non-
Appeals in its Decision: medical" insurance policy. In Saturnino v. Philippine-
American Life Insurance Company, 16 this Court held
Sinus tachycardia is defined as that:
sinus-initiated; heart rate faster
than 100 beats per minute. . . . if anything, the waiver of
(Harrison' s Principles of Internal medical examination [in a non-
Medicine, 8th ed. [1978], p. 1193.) medical insurance contract]
It is, among others, a common renders even more material the
reaction to heart disease, including information required of the
myocardial infarction, and heart applicant concerning previous
failure per se. (Henry J.L. Marriot, condition of health and diseases
M.D., Electrocardiography, 6th ed., suffered, for such information
[1977], p. 127.) The medication necessarily constitutes an
prescribed by Dr. Claudio for important factor which the insurer
treatment of Canilang's ailment on takes into consideration in deciding
June 18, 1982, indicates the whether to issue the policy or not .
condition that said physician was . . . 17 (Emphasis supplied)
trying to manage. Thus, he
prescribed Trazepam, (Philippine
Index of Medical Specialties (PIMS), The Insurance Commissioner had also ruled that the
Vol. 14, No. 3, Dec. 1985, p. 112) failure of Great Pacific to convey certain information
which is anti-anxiety, anti- to the insurer was not "intentional" in nature, for the
convulsant, muscle-relaxant; and reason that Jaime Canilang believed that he was
Aptin, (Idem, p. 36) a cardiac drug, suffering from minor ailment like a common cold.
for palpitations and nervous heart. Section 27 of the Insurance Code of 1978 as it existed
Such treatment could have been a from 1974 up to 1985, that is, throughout the time
very material information to the range material for present purposes, provided that:
insurer in determining the action to
be take on Canilang's application
for life insurance coverage. 14
Sec. 27. A concealment entitles the times rise to high and alarming levels and that he had
injured party to rescind a contract consulted a doctor twice in the two (2) months before
of insurance. applying for non-medical insurance. Indeed, the last
medical consultation took place just the day before
The preceding statute, Act No. 2427, as it the insurance application was filed. In all probability,
stood from 1914 up to 1974, had provided: Jaime Canilang went to visit his doctor precisely
because of the discomfort and concern brought about
by his experiencing "sinus tachycardia."
Sec. 26. A concealment, whether
intentional or unintentional,
entitles the injured party to We find it difficult to take seriously the argument
rescind a contract of insurance. that Great Pacific had waived inquiry into the
(Emphasis supplied) concealment by issuing the insurance policy
notwithstanding Canilang's failure to set out answers
to some of the questions in the insurance application.
Upon the other hand, in 1985, the Insurance Code of Such failure precisely constituted concealment on the
1978 was amended by part of Canilang. Petitioner's argument, if accepted,
B.P. Blg. 874. This subsequent statute modified would obviously erase Section 27 from the Insurance
Section 27 of the Insurance Code of 1978 so as to read Code of 1978.
as follows:
It remains only to note that the Court of Appeals
Sec. 27. A concealment whether finding that the parties had not agreed in the pretrial
intentional or unintentional before the Insurance Commission that the relevant
entitles the injured party to issue was whether or not Jaime Canilang had
rescind a contract of insurance. intentionally concealed material information from
(Emphasis supplied) the insurer, was supported by the evidence of record,
i.e., the Pre-trial Order itself dated 17 October 1984
The unspoken theory of the Insurance Commissioner and the Minutes of the Pre-trial Conference dated 15
appears to have been that by deleting the phrase October 1984, which "readily shows that the word
"intentional or unintentional," the Insurance Code of "intentional" does not appear in the statement or
1978 (prior to its amendment by B.P. Blg. 874) definition of the issue in the said Order and Minutes."
intended to limit the kinds of concealment which 18
generate a right to rescind on the part of the injured
party to "intentional concealments." This argument is WHEREFORE, the Petition for Review is DENIED for
not persuasive. As a simple matter of grammar, it lack of merit and the Decision of the Court of Appeals
may be noted that "intentional" and "unintentional" dated 16 October 1989 in C.A.-G.R. SP No. 08696 is
cancel each other out. The net result therefore of the hereby AFFIRMED. No pronouncement as to the costs.
phrase "whether intentional or unitentional" is
precisely to leave unqualified the term
"concealment." Thus, Section 27 of the Insurance SO ORDERED.
Code of 1978 is properly read as referring to "any
concealment" without regard to whether such Republic of the Philippines
concealment is intentional or unintentional. The SUPREME COURT
phrase "whether intentional or unintentional" was in Manila
fact superfluous. The deletion of the phrase "whether
intentional or unintentional" could not have had the FIRST DIVISION
effect of imposing an affirmative requirement that a
concealment must be intentional if it is to entitle the
injured party to rescind a contract of insurance. The
restoration in 1985 by B.P. Blg. 874 of the phrase
"whether intentional or unintentional" merely G.R. No. 105135 June 22, 1995
underscored the fact that all throughout (from 1914
to 1985), the statute did not require proof that SUNLIFE ASSURANCE COMPANY OF CANADA,
concealment must be "intentional" in order to petitioner,
authorize rescission by the injured party. vs.
The Hon. COURT OF APPEALS and Spouses
In any case, in the case at bar, the nature of the facts ROLANDO and BERNARDA BACANI, respondents.
not conveyed to the insurer was such that the failure
to communicate must have been intentional rather
than merely inadvertent. For Jaime Canilang could
not have been unaware that his heart beat would at
QUIASON, J.:
This is a petition for review for certiorari under Rule o
45 of the Revised Rules of Court to reverse and set o
aside the Decision dated February 21, 1992 of the d
Court of Appeals in CA-G.R. CV No. 29068, and its t
Resolution dated April 22, 1992, denying e
reconsideration thereof. s
t
We grant the petition. s
?
o
I t
h
On April 15, 1986, Robert John B. Bacani procured a e
life insurance contract for himself from petitioner. He r
was issued Policy No. 3-903-766-X valued at t
P100,000.00, with double indemnity in case of e
accidental death. The designated beneficiary was his s
mother, respondent Bernarda Bacani. t
s
On June 26, 1987, the insured died in a plane crash. ?
Respondent Bernarda Bacani filed a claim with
petitioner, seeking the benefits of the insurance c) attended or
policy taken by her son. Petitioner conducted an been admitted to
investigation and its findings prompted it to reject the any hospital or
claim. other medical
facility?
In its letter, petitioner informed respondent Bernarda
Bacani, that the insured did not disclose material 6. Have you ever had or sought
facts relevant to the issuance of the policy, thus advice for:
rendering the contract of insurance voidable. A check
representing the total premiums paid in the amount xxx xxx xxx
of P10,172.00 was attached to said letter.
b) urine, kidney or bladder
Petitioner claimed that the insured gave false disorder? (Rollo, p. 53)
statements in his application when he answered the
following questions:
The deceased answered question No. 5(a) in the
affirmative but limited his answer to a consultation
5. Within the past 5 years have you: with a certain Dr. Reinaldo D. Raymundo of the
Chinese General Hospital on February 1986, for cough
a) consulted any and flu complications. The other questions were
doctor or other answered in the negative (Rollo, p. 53).
health
practitioner? Petitioner discovered that two weeks prior to his
application for insurance, the insured was examined
b) submitted to: and confined at the Lung Center of the Philippines,
where he was diagnosed for renal failure. During his
E confinement, the deceased was subjected to
G urinalysis, ultra-sonography and hematology tests.
G
? On November 17, 1988, respondent Bernarda Bacani
X and her husband, respondent Rolando Bacani, filed an
- action for specific performance against petitioner
r with the Regional Trial Court, Branch 191,
a Valenzuela, Metro Manila. Petitioner filed its answer
y with counterclaim and a list of exhibits consisting of
s medical records furnished by the Lung Center of the
? Philippines.
b
l
On January 14, 1990, private respondents filed a II
"Proposed Stipulation with Prayer for Summary
Judgment" where they manifested that they "have no We reverse the decision of the Court of Appeals.
evidence to refute the documentary evidence of
concealment/misrepresentation by the decedent of
his health condition (Rollo, p. 62). The rule that factual findings of the lower court and
the appellate court are binding on this Court is not
absolute and admits of exceptions, such as when the
Petitioner filed its Request for Admissions relative to judgment is based on a misappreciation of the facts
the authenticity and due execution of several (Geronimo v. Court of Appeals, 224 SCRA 494 [1993]).
documents as well as allegations regarding the health
of the insured. Private respondents failed to oppose
said request or reply thereto, thereby rendering an In weighing the evidence presented, the trial court
admission of the matters alleged. concluded that indeed there was concealment and
misrepresentation, however, the same was made in
"good faith" and the facts concealed or
Petitioner then moved for a summary judgment and misrepresented were irrelevant since the policy was
the trial court decided in favor of private "non-medical". We disagree.
respondents. The dispositive portion of the decision is
reproduced as follows:
Section 26 of The Insurance Code is explicit in
requiring a party to a contract of insurance to
WHEREFORE, judgment is hereby communicate to the other, in good faith, all facts
rendered in favor of the plaintiffs within his knowledge which are material to the
and against the defendant, contract and as to which he makes no warranty, and
condemning the latter to pay the which the other has no means of ascertaining. Said
former the amount of One Hundred Section provides:
Thousand Pesos (P100,000.00) the
face value of insured's Insurance
Policy No. 3903766, and the A neglect to communicate that
Accidental Death Benefit in the which a party knows and ought to
amount of One Hundred Thousand communicate, is called
Pesos (P100,000.00) and further concealment.
sum of P5,000.00 in the concept of
reasonable attorney's fees and costs Materiality is to be determined not by the event, but
of suit. solely by the probable and reasonable influence of the
facts upon the party to whom communication is due,
Defendant's counterclaim is hereby in forming his estimate of the disadvantages of the
Dismissed (Rollo, pp. 43-44). proposed contract or in making his inquiries (The
Insurance Code, Sec. 31).
In ruling for private respondents, the trial court
concluded that the facts concealed by the insured The terms of the contract are clear. The insured is
were made in good faith and under a belief that they specifically required to disclose to the insurer matters
need not be disclosed. Moreover, it held that the relating to his health.
health history of the insured was immaterial since the
insurance policy was "non-medical". The information which the insured failed to disclose
were material and relevant to the approval and
Petitioner appealed to the Court of Appeals, which issuance of the insurance policy. The matters
affirmed the decision of the trial court. The appellate concealed would have definitely affected petitioner's
court ruled that petitioner cannot avoid its obligation action on his application, either by approving it with
by claiming concealment because the cause of death the corresponding adjustment for a higher premium
was unrelated to the facts concealed by the insured. or rejecting the same. Moreover, a disclosure may
It also sustained the finding of the trial court that have warranted a medical examination of the insured
matters relating to the health history of the insured by petitioner in order for it to reasonably assess the
were irrelevant since petitioner waived the medical risk involved in accepting the application.
examination prior to the approval and issuance of the
insurance policy. Moreover, the appellate court In Vda. de Canilang v. Court of Appeals, 223 SCRA 443
agreed with the trial court that the policy was "non- (1993), we held that materiality of the information
medical" (Rollo, pp. 4-5). withheld does not depend on the state of mind of the
insured. Neither does it depend on the actual or
Petitioner's motion for reconsideration was denied; physical events which ensue.
hence, this petition.
Thus, "goad faith" is no defense in concealment. The MERCEDES VARGAS VDA. DE SONGCO, ET AL. and
insured's failure to disclose the fact that he was COURT OF APPEALS, respondents.
hospitalized for two weeks prior to filing his
application for insurance, raises grave doubts about Jose S. Suarez for petitioner.
his bonafides. It appears that such concealment was Eligio G. Lagman for respondents.
deliberate on his part.

The argument, that petitioner's waiver of the medical


examination of the insured debunks the materiality of
the facts concealed, is untenable. We reiterate our FERNANDO, J.:
ruling in Saturnino v. Philippine American Life
Insurance Company, 7 SCRA 316 (1963), that " . . . the An insurance firm, petitioner Fieldmen's Insurance
waiver of a medical examination [in a non-medical Co., Inc., was not allowed to escape liability under a
insurance contract] renders even more material the common carrier insurance policy on the pretext that
information required of the applicant concerning what was insured, not once but twice, was a private
previous condition of health and diseases suffered, for vehicle and not a common carrier, the policy being
such information necessarily constitutes an important issued upon the insistence of its agent who discounted
factor which the insurer takes into consideration in fears of the insured that his privately owned vehicle
deciding whether to issue the policy or not . . . " might not fall within its terms, the insured moreover
being "a man of scant education," finishing only the
Moreover, such argument of private respondents first grade. So it was held in a decision of the lower
would make Section 27 of the Insurance Code, which court thereafter affirmed by respondent Court of
allows the injured party to rescind a contract of Appeals. Petitioner in seeking the review of the above
insurance where there is concealment, ineffective decision of respondent Court of Appeals cannot be so
(See Vda. de Canilang v. Court of Appeals, supra). sanguine as to entertain the belief that a different
outcome could be expected. To be more explicit, we
sustain the Court of Appeals.
Anent the finding that the facts concealed had no
bearing to the cause of death of the insured, it is well
settled that the insured need not die of the disease The facts as found by respondent Court of Appeals,
he had failed to disclose to the insurer. It is sufficient binding upon us, follow: "This is a peculiar case.
that his non-disclosure misled the insurer in forming Federico Songco of Floridablanca, Pampanga, a man
his estimates of the risks of the proposed insurance of scant education being only a first grader ..., owned
policy or in making inquiries (Henson v. The Philippine a private jeepney with Plate No. 41-289 for the year
American Life Insurance Co., 56 O.G. No. 48 [1960]). 1960. On September 15, 1960, as such private vehicle
owner, he was induced by Fieldmen's Insurance
Company Pampanga agent Benjamin Sambat to apply
We, therefore, rule that petitioner properly exercised for a Common Carrier's Liability Insurance Policy
its right to rescind the contract of insurance by reason covering his motor vehicle ... Upon paying an annual
of the concealment employed by the insured. It must premium of P16.50, defendant Fieldmen's Insurance
be emphasized that rescission was exercised within Company, Inc. issued on September 19, 1960,
the two-year contestability period as recognized in Common Carriers Accident Insurance Policy No. 45-
Section 48 of The Insurance Code. HO- 4254 ... the duration of which will be for one (1)
year, effective September 15, 1960 to September 15,
WHEREFORE, the petition is GRANTED and the 1961. On September 22, 1961, the defendant
Decision of the Court of Appeals is REVERSED and SET company, upon payment of the corresponding
ASIDE. premium, renewed the policy by extending the
coverage from October 15, 1961 to October 15, 1962.
SO ORDERED. This time Federico Songco's private jeepney carried
Plate No. J-68136-Pampanga-1961. ... On October 29,
1961, during the effectivity of the renewed policy,
Republic of the Philippines the insured vehicle while being driven by Rodolfo
SUPREME COURT Songco, a duly licensed driver and son of Federico
Manila (the vehicle owner) collided with a car in the
municipality of Calumpit, province of Bulacan, as a
EN BANC result of which mishap Federico Songco (father) and
Rodolfo Songco (son) died, Carlos Songco (another
G.R. No. L-24833 September 23, 1968 son), the latter's wife, Angelita Songco, and a family
friend by the name of Jose Manuel sustained physical
injuries of varying degree." 1
FIELDMEN'S INSURANCE CO., INC., petitioner,
vs.
It was further shown according to the decision of knew all along when not once but twice its agent, no
respondent Court of Appeals: "Amor Songco, 42-year- doubt without any objection in its part, exerted the
old son of deceased Federico Songco, testifying as utmost pressure on the insured, a man of scant
witness, declared that when insurance agent education, to enter into such a contract.
Benjamin Sambat was inducing his father to insure his
vehicle, he butted in saying: 'That cannot be, Mr. Nor is there any merit to the second alleged error of
Sambat, because our vehicle is an "owner" private respondent Court that no legal liability was incurred
vehicle and not for passengers,' to which agent under the policy by petitioner. Why liability under the
Sambat replied: 'whether our vehicle was an "owner" terms of the policy 5 was inescapable was set forth in
type or for passengers it could be insured because the decision of respondent Court of Appeals. Thus:
their company is not owned by the Government and "Since some of the conditions contained in the policy
the Government has nothing to do with their issued by the defendant-appellant were impossible to
company. So they could do what they please comply with under the existing conditions at the time
whenever they believe a vehicle is insurable' ... In and 'inconsistent with the known facts,' the insurer 'is
spite of the fact that the present case was filed and estopped from asserting breach of such conditions.'
tried in the CFI of Pampanga, the defendant company From this jurisprudence, we find no valid reason to
did not even care to rebut Amor Songco's testimony deviate and consequently hold that the decision
by calling on the witness-stand agent Benjamin appealed from should be affirmed. The injured
Sambat, its Pampanga Field Representative." 2 parties, to wit, Carlos Songco, Angelito Songco and
Jose Manuel, for whose hospital and medical expenses
The plaintiffs in the lower court, likewise respondents the defendant company was being made liable, were
here, were the surviving widow and children of the passengers of the jeepney at the time of the
deceased Federico Songco as well as the injured occurrence, and Rodolfo Songco, for whose burial
passenger Jose Manuel. On the above facts they expenses the defendant company was also being
prevailed, as had been mentioned, in the lower court made liable was the driver of the vehicle in question.
and in the respondent Court of Appeals.1awphl.nt Except for the fact, that they were not fare paying
passengers, their status as beneficiaries under the
The basis for the favorable judgment is the doctrine policy is recognized therein." 6
announced in Qua Chee Gan v. Law Union and Rock
Insurance Co., Ltd., 3 with Justice J. B. L. Reyes Even if it be assumed that there was an ambiguity, an
speaking for the Court. It is now beyond question that excerpt from the Qua Chee Gan decision would reveal
where inequitable conduct is shown by an insurance anew the weakness of petitioner's contention. Thus:
firm, it is "estopped from enforcing forfeitures in its "Moreover, taking into account the well known rule
favor, in order to forestall fraud or imposition on the that ambiguities or obscurities must be strictly
insured." 4 interpreted against the party that caused them, the
'memo of warranty' invoked by appellant bars the
As much, if not much more so than the Qua Chee Gan latter from questioning the existence of the
decision, this is a case where the doctrine of estoppel appliances called for in the insured premises, since its
undeniably calls for application. After petitioner initial expression, 'the undernoted appliances for the
Fieldmen's Insurance Co., Inc. had led the insured extinction of fire being kept on the premises insured
Federico Songco to believe that he could qualify hereby, ... it is hereby warranted ...,' admits of
under the common carrier liability insurance policy, interpretation as an admission of the existence of
and to enter into contract of insurance paying the such appliances which appellant cannot now
premiums due, it could not, thereafter, in any contradict, should the parol evidence rule apply." 7
litigation arising out of such representation, be
permitted to change its stand to the detriment of the To the same effect is the following citation from the
heirs of the insured. As estoppel is primarily based on same leading case: "This rigid application of the rule
the doctrine of good faith and the avoidance of harm on ambiguities has become necessary in view of
that will befall the innocent party due to its injurious current business practices. The courts cannot ignore
reliance, the failure to apply it in this case would that nowadays monopolies, cartels and concentration
result in a gross travesty of justice. of capital, endowed with overwhelming economic
power, manage to impose upon parties dealing with
That is all that needs be said insofar as the first them cunningly prepared 'agreements' that the
alleged error of respondent Court of Appeals is weaker party may not change one whit, his
concerned, petitioner being adamant in its far-from- participation in the 'agreement' being reduced to the
reasonable plea that estoppel could not be invoked by alternative to 'take it or leave it' labelled since
the heirs of the insured as a bar to the alleged breach Raymond Saleilles 'contracts by adherence' (contrats
of warranty and condition in the policy. lt would now d'adhesion), in contrast to those entered into by
rely on the fact that the insured owned a private parties bargaining on an equal footing, such contracts
vehicle, not a common carrier, something which it (of which policies of insurance and international bills
of lading are prime examples) obviously call for FELICIANO, represented by their guardian ad litem
greater strictness and vigilance on the part of courts SERAFIN D. FELICIANO, respondents.
of justice with a view to protecting the weaker party
from abuses and imposition, and prevent their Araneta, Zaragosa, Araneta & Bautista for petitioner.
becoming traps for the unwary (New Civil Code. Delfin Joven for respondents.
Article 24; Sent. of Supreme Court of Spain, 13 Dec.
1934, 27 February 1942)." 8
LAUREL, J.:
The last error assigned which would find fault with
the decision of respondent Court of Appeals insofar as One Evaristo Feliciano filed an application for
it affirmed the lower court award for exemplary insurance with the herein petitioner upon the
damages as well as attorney's fees is, on its face, of solicitation of one of its agents. Two insurance
no persuasive force at all. policies to the aggregate amount of P25,000 were
issued to him. Feliciano died on September 29, 1935.
The defendant company refused to pay on the ground
The conclusion that inescapably emerges from the that the policies were fraudulently obtained, the
above is the correctness of the decision of respondent insured having given false answers and statements in
Court of Appeals sought to be reviewed. For, to the application as well as in the medical report. The
borrow once again from the language of the Qua Chee present action was brought to recover on said
Gan opinion: "The contract of insurance is one of policies. The lower court rendered judgment in favor
perfect good faith (uberima fides) not for the insured of the plaintiffs. The lower court found that at the
alone,but equally so for the insurer; in fact, it is more time Feliciano filed his application and at the time he
so for the latter, since its dominant bargaining was subjected to physical examination by the medical
position carries with it stricter responsibility." 9 examiner of the herein petitioner, he was already
suffering from tuberculosis. This fact appears in the
This is merely to stress that while the morality of the negative both in the application and in the medical
business world is not the morality of institutions of report. The lower court, after an exhaustive
rectitude like the pulpit and the academe, it cannot examination of the conflicting testimonies, also found
descend so low as to be another name for guile or that Feliciano was made to sign the application and
deception. Moreover, should it happen thus, no court the examiner's report in blank, and that afterwards
of justice should allow itself to lend its approval and the blank spaces therein were filled in by the agent
support.1awphl.nt and the medical examiner, who made it appear
therein that Feliciano was a fit subject for insurance.
We have no choice but to recognize the monetary The lower court also held that neither the insured nor
responsibility of petitioner Fieldmen's Insurance Co., any member of his family concealed the real state of
Inc. It did not succeed in its persistent effort to avoid health of the insured. That as a matter of fact the
complying with its obligation in the lower court and insured, as well as the members of his family, told the
the Court of Appeals. Much less should it find any agent and the medical examiner that the applicant
receptivity from us for its unwarranted and had been sick and coughing for sometime and that he
unjustified plea to escape from its liability. had also gone three times to the Santol Sanatarium.
On appeal, this finding of facts of the lower court was
sustained by the Court of Appeals. This concludes the
WHEREFORE, the decision of respondent Court of controversy over the facts in so far as this Court is
Appeals of July 20, 1965, is affirmed in its entirety. concerned.
Costs against petitioner Fieldmen's Insurance Co., Inc.
The first assignment of error of the petitioner raises
Republic of the Philippines the question we are now called upon to decide:
SUPREME COURT
Manila
The Court of Appeals erred in holding that an
insurance company has no right to avoid a policy
EN BANC where its agent knowingly and intentionally wrote
down the answers in the application differing from
G.R. No. L-47593 September 13, 1941 those made by the insured, in disregard of the
exception that when the agent, instead of serving the
THE INSULAR LIFE ASSURANCE CO., LTD., interests of his principal, acts in his own or another's
petitioner, interest and adversely to that of his principal, the said
vs. principal is not bound by said acts of the agent."
SERAFIN D. FELICIANO and ANGEL, FLORENDA,
EUGENIO, HERMINIO and LETICIA, all surnamed On the proposition thus presented, there are two
main avenues of approach indicated: one leading to
the validation of a policy where its agent, without The company received the money of the applicant as
fraud, collusion or bad faith on the part of the the price of the risk to be taken by it. If the policy
insured, falsified the answers given by the insured; should be avoided, it must be because it was void
and the other, leading to the avoidance of the policy from the very beginning, and the result would be that
under the circumstances. We see no need for an the insurer, while it received the money, never
extended discussion of the conflicting authorities. assumed any risk. The result would be, in the
Whenever courts are given the choice between two language of one of the cases, "to place every simple
conflicting principles, the determinative fact which or uneducated person seeking insurance at the mercy
should sway them is the conformity of its of the insurer who could, through its agent, insert in
contemplated course to reason and to "the common every application, unknown to the applicant and over
sense of the situation." The life of the law is not only his signature, some false statements which would
logic but experience. enable him to avoid all liability while retaining the
price paid for the supposes insurance." (State
The phenomenal growth of insurance from almost Insurance Company v. Taylor, 14 Colo. 499, 24 Pac.
nothing a hundred years ago to its present gigantic 333.) The weight of authority is that if an agent of the
proportion is not of the outstanding marvels of insurer, after obtaining from an applicant for
present-day business life. The demand for economic insurance a correct and truthful answer to
security, the growing need for social stability, and the interrogatories contained in the application for
clamor for protection against the hazards of cruel- insurance, without knowledge of the applicant fills in
crippling calamities and sudden economic shocks, false answers, either fraudulently or otherwise, the
have made insurance one of the felt necessities of insurer cannot assert the falsity of such answers as a
modern life. Insurance is no longer a rich man's defense to liability on the policy, and this is true
monopoly. Upon it are heaped the assured hopes of generally without regard to the subject matter of the
many families of modest means. It is woven, as it answers or the nature of the agent's duties or
were, into the very warp and woof of national limitations on his authority, at least if not brought to
economy. It touches the holiest and most sacred ties the attention of the applicant.
in the life of man-love of parents, love of wives and
love of children. It is of common knowledge that the The fact that the insured did not read the application
selling of insurance today is subjected to the which he signed, is not indicative of bad faith. It has
whilrlwind pressure of modern salesmanship. been held that it is not negligence for the insured to
Insurance companies send detailed instructions to sign an application without first reading it if the
their agents to solicit and procure applications. These insurer by its conduct in appointing the agent
agents are to be found all over the length and breadth influenced the insured to place trust and confidence
of the land. They are stimulated to more active in the agent. (Den Hartog v. Home Nat. Ins. Asso., 197
efforts by contests and by the keen competition Iowa, 143 196 N. W. 944.) As the court said in the case
offered by other rival insurance companies. They are of Germania L. Ins. Co. v. Lunkebiemer, 127 Ind. 538,
supplied with blank applications and paid large 26 N. E. 1082, "Nor can it be said that the assured,
commissions on the policies secured by them. All who has fully, frankly, truthfully, and in good faith
transactions are generally done through these agents. answered all the required questions, is guilty of
They act, in fact and in theory, as the general negligence in signing, without reading, the
representatives of the insurance companies. They application which is thereupon prepared by the agent.
supply all the information , prepare and answer the He is justified in assuming that the agent has, with
applications, submit the applications to their equal good faith, truthfully recorded the answers
companies, conclude the transactions, and otherwise given him. He may well say to the company: "You
smooth out all difficulties. The agents, in short, do accredited this man to me as your representative and
what the company set them to do. I signed the application thus prepared by him, relying
upon the character which you gave him when you
In the present case, the agent knew all the time the commissioned him to come to me as your agent. If he
true state of health of the insured. The insurer's acted dishonestly in the matter, you and not I must
medical examiner approve the application knowing suffer the consequences.' ..." In the instant case, it
full well that the applicant was sick. The situation is has been proved that the insured could not read
one in which one of two innocent parties must bear a English, the language in which the application was
loss for his reliance upon a third person. In this case, written, and that after the contract was signed, it was
it was the insurer who gave the agent authority to kept by his mother. As a consequence, the insured had
deal with the applicant. It was the one who selected no opportunity to read or correct any misstatement
the agent, thus implying that the insured could put therein. (Bill of Exceptions, pp. 60-61.)
his trust on him. It was the one who drafted and
accepted the policy and consummated the contract. We have not been insensible to the appeal that the
It seems reasonable that as between the two of them, course we have followed may lead to fraud and work
the one who employed and gave character to the third hardship on insurance companies, for it would be easy
person as its agent should be the one to bear the loss. for insurance agents and applicants to insert false
answers in their applicants to insert false answers in of the same malady he had been suffering-pulmonary
their applications for insurance. This means that it is tuberculosis.
to the particular interest of these companies to
exercise greater care in the selection of their agents The Court of Appeals found in effect that the
and examiners. Their protection is still in their own Company's soliciting agent Romulo M. David, in
hands and which may be achieved by other means. conclusion with the medical examiner Dr. Gregorio
Withal, the attainment of a common good may involve Valdez, knowingly wrote false answers to the question
impairment and even sacrifice of beneficial interests contained in the applications and in the medical
of a particular group, but in life, compromise is examiner's reports which they had made the
inevitable until the hour of doom strikes. applicant sign in blank in order to secure the
Company's approval thereof and have the
The petition is hereby dismissed and the judgment corresponding policies credited to the agent in
sought to be reviewed is affirmed with costs against connection with the interprovincial contest which the
the petitioner. So ordered. Company was then holding among its soliciting agents
to boost the sales of its policies. The Court of Appeals
Abad Santos, Diaz, and Horrilleno, JJ., concur. intimates that Agent David bribed Medical Examiner
Valdez with money which the former borrowed from
the applicant's mother by way of advanced payment
on the premium. In this connection, it may be
mentioned that the premium paid on the first policy
Separate Opinions was P1,111.20, and that on the second policy,
P277.80, or a total of P1,389, which the Company
OZAETA, J., with whom concur AVANCEA, C.J., offers to refund.
and MORAN, J., dissenting:
The Court of Appeals also found that before the
Altho a dissenting opinion is but a voice in the insured signed the first application and medical
wilderness, we have to write it because the examiner's report, he and the members of his family
Constitution so requires. told the agent and the medical examiner that he had
been sick and coughing for some time and that he had
gone three times to the Santol Sanatorium and had X-
The material facts are not disputed in this instance, ray pictures of his lungs taken; but that in spite of
but they are not adequately stated in the majority such information the agent and the medical examiner
opinion, and we apprehend that the significance of told them that the applicant was a fit subject for
those not stated therein may have been overlooked insurance.
by the majority of the Court.
Each of the policies sued upon contains the following
This is a suit on two life insurance policies issued by stipulations:
the petitioner (hereinafter referred to as the
Company) to Evaristo Feliciano as of October 1 and
November 1, 1934, for P20,000 and P5,000, This policy and the application therefor
respectively. The application for the first policy was constitute the entire contract between the
signed on October 12, and that for the second policy, parties hereto. ... Only the President, or the
on October 28, 1934. On those dates Feliciano "had an Manager, acting jointly with the Secretary of
advanced disease of the lungs ... He was breathless, Assistant Secretary (and then only in writing
having difficulty in breathing, and he had the signed by them) have power in behalf of the
appearance of one with high fever." As a matter of Company to issue permits or to modify this
fact, on October 12, 1934, the very day the insured or any contract, or to extend the time for
signed the first applications, after the last X-ray making any premium payment, and the
examination of his lungs had been made at the Santol Company shall not be bound by any promise
Sanatorium by Doctor Trepp, the latter informed the or representation heretofore or hereafter
respondent Serafin D. Feliciano, brother of the given by any person other than the above-
insured, of the result of the X-ray examination and named officials, and by them only in writing
told him that in his opinion his brother "was already and signed conjointly as stated.
in a very serious and practically hopeless condition."
(Trial court's decision, P.27, B. of E.) After the first The application referred to in and made a part of the
application for insurance of P20,000 had been policy contains, among others, the following
approved and the corresponding policy issued, the statements:
insured applied on October 28, 1934, for another
insurance of P5,000, and the policy therefor was 18. I [the applicant] hereby declare that
issued as of November 1, 1934. Less than one year all the above statements and answers as well
later, to wit, on September 29, 1935, the insured died
as those that I may make to the Company's wrongful issuance of the policies in question by the
Medical Examiner in continuation of this Company to defraud the latter. It is difficult to
application, to be complete, true and believe that in so doing he and his relatives were not
correct to the best of my knowledge and actuated by the desire for lucre. They knew that a
belief, and I hereby agree as follows: person in bad health let alone one who was "in a
very serious and practically hopeless condition" was
1. That this declaration, with the answers to not insurable. So they must also have known, or at
be given by me to the Medical Examiner, least they had good reason to suspect, that Agent
shall be the basis of the policy and from part David and Medical Examiner Valdez were not acting in
of same. good faith when they made the applicant sign the
application in blank and told him (the hopelessly sick
man) that he was fit of insurance. If the applicant and
xxx xxx xxx his relatives were acting in good faith, they would
have been curious enough to scrutinize the
3. That the said policy shall not take effect application and the medical examiner's report
until the first premium has been paid and the contained in the first policy upon receipt of it, to see
policy has been delivered to and accepted by whether the medical examiner had correctly stated
me, while I am in good health. therein the state of the applicant's health. It is
significant that shortly after they had received the
4. That the agent taking this application has first policy of P20,000, the insured applied for and
no authority to make, modify or discharge secured another policy of P5,000. As held by the
contracts, or to waive any of the company's Supreme Court of the United States in the analogous
rights or requirements. case on New York Life Insurance company v. Fletcher,
117 U. S. 519: "He could not hold the policy without
approving the action of the agents and thus becoming
xxx xxx xxx a participant in the fraud committed. The retention
of the policy was an approval of the application and
Upon the facts above set forth, we are of the opinion of its statements. The consequences of that approval
that respondents are not entitled to recover the cannot after his death be avoided."
amounts of the policies in question but only the
premiums paid thereon, for the following reasons: 3. Life insurance is a savings institution; it is not a
gambling scheme. The premiums paid by the insured,
1. Under the very terms of the policies sued upon plus a participation in the profits realized by the life
there is no valid contract of insurance here. The insurance company from the investment of those
policies were issued on the basis of the statement premiums, are returned to him if he survives the
subscribed to by the applicant to the effect that he policy. If, contrary to the life expectancy of the
was and had been in good health. The basis being insured, he dies before the policy matures, the full
false, there was no real meeting of the minds of the amount of the insurance is paid to his beneficiary. The
parties. The agents had no authority to bind the insured is not expected to lose, but neither is he
company thru oral representations, and less so when supposed to expect a windfall or an inordinate gain.
such representations were false and fraudulent. That is elemental in every sound business. The life
insurance business is a co-operative enterprise in the
2. The insured and the members of his family who are sense that the policy-holders as well as the company
the respondents herein were not entirely innocent of are interested in making profits and in avoiding
bad faith. They were not candid, unsophisticated unnecessary or bad losses. The company is, to a
rustics. They were well to do and well educated. They certain extent, a trustee of the funds paid to it by its
were not ignorant of the practices in the life policyholders. No insurance company which would
insurance business. In 1924, the insured had taken an issue policies indiscriminately could expect to survive
insurance policy of P10,000 from the Sun Life or, for that matter, be licensed by the Government to
Insurance Company, which, however, he allowed to do business. That is fundamental. Every fraud
lapse. The insured was a "proprietor and agriculturist" perpetrated upon the company affects the policy-
(see policy Exhibit E). The respondent Serafin D. holders because their share in the profits is thereby
Feliciano, brother of the insured, is a physician who unduly minimized. That is why the Government, thru
for some years had worked in the Santol Sanatorium the Insurance Commissioner, closely supervises the
with Doctor Trepp (Exhibit B, p. 16). The most insurance business (see section 169 et seq., The
charitable view that one could take of the insured's Insurance Act). We think it is bad law to hold valid a
part in the transaction is that he, with the approval policy procured thru fraud on the life of a person who
of his relatives, particularly his mother who furnished was almost on the brink of his grave. Avaricious
the money with which to pay the premiums and who persons, with the connivance of unscrupulous agents
was named beneficiary to the extent of P12,000, of insurance companies, could make money on the
allowed himself to be used as instrument in the lives of their relatives who were expected to die soon,
by fraudulently insuring them, and could get away the disease made no difference, and that if he would
with it, as in the instant case. The real or ultimate take out a policy and pay the premiums required he
victim is not the company alone but also its numerous would have no trouble; that finally, about the 18th of
policyholders who have put their savings in it. December, 1877, he consented to take a policy; that
they then told him it would be necessary for him to
It is suggested that the remedy is for the insurance answer certain questions as a matter of form; that
companies to exercise greater care in the selection of one of them thereupon read to him certain questions
their agents and examiners. As a matter of fact, under from a printed blank, and as he answered them the
the law no one may act as soliciting agent of an other pretended to take down and write in the blank
insurance company without authority or license from the substance of the answers as given, not reading
the Insurance Commissioner(section 189, The over to the assured what he had written, nor
Insurance Act); and the Insurance Commissioner consulting him about it, nor informing him what it
makes a careful, confidential investigation of the was, but saying that what he did was a mere
conduct and reputation of the applicant for such formality; that when he was asked with respect to his
license before issuing the same. But no amount of having any disease of the kidneys he replied that his
care taken by both the company and the Insurance condition was well known to the agents, who were
Commissioner in the selection of soliciting agents and aware that he had been sick and under treatment by
medical examiners can insure the company against Doctor Brokaw for diabetes, and that the doctor's
bad faithly and the cupidity of the evil-minded. The office was opposite, and they could go there and find
company would have to exact a huge bond of every out everything they wanted to know; that the assured
one of its numerous agents and medical examiners to had faithfully answered all the questions, but the
guarantee his fidelity, and that would be too agents inserted in the blanks false answers; that he
expensive to make the insurance business profitable. had no reason to suppose that the answers were taken
In other words, the suggested remedy is, we believe, down differently from those given; that after
impracticable. The only safe and sound policy is, not answering all their question he was asked to sign his
to condone but to condemn fraud under any and all name to the paper to identify him as the party for
circumstances. whose benefit the policy was to be issued, and for
that purpose he signed the paper twice, without
reading it or the written answers; that the agents did
4. If we are to be guided and persuaded by cases not read to him any part of the application except the
adjudicated in other jurisdictions as the Court of questions, and did not read the clause set forth in the
Appeals was in deciding this case, we should follow defendant's answer, nor call the attention to the fact
that decided by the Supreme Court of the United that his signatures were intended as an acceptance or
States upon facts similar or analogous to those assent to that clause; that when the policy was
obtaining in the instant case, instead of adopting delivered to him he neither read it nor the copy of the
doctrines laid down by the state supreme courts and application attached to it, that the agent who
inferior federal courts in cases the facts of which bear delivered it informed him that it was right, and he
little or no anology to those of the case at bar. The was insured, and he gave no further attention to the
case squarely in point, but which the Court of Appeals matter; that the annual premiums, as they fell due,
rejected, is New York Life Insurance Company v. were paid to said agent, who received them with full
Fletcher, supra. In that case it was stated in the knowledge of all the facts; and that, therefore, the
application for insurance that the applicant never had company was estopped form pretending that any of
a disease of the kidneys or any serious disease, and the answers as written rendered the policy void." (117
had never been seriously ill, and had no regular U.S. 521-523.)
medical attendant, whereas he had been afflicted
with diabetes, which is a serious disease of the
kidneys, and had been under medical treatment for In reversing the judgment rendered by the trial court
it, and he actually died of that disease. The plaintiff in favor of the plaintiff, the Federal Supreme Court
therein, however, alleged, and adduced evidence to held "that the agent had no authority from the
show: company to falsify the answers," and that "the assured
could acquire no right by virtue of his falsified
answers." The Court further said:
. . . That two agents of the company at St. Louis, who
were personally acquainted with the assured and
knew his past and then physical condition, had . . . Both he and the company were deceived
solicited him on different occasions to take out a by the fraudulent conduct of the agent. The
policy in the company; that he told each of them on assured was placed in the position of making
those occasions that he did not believe he was false representations in order to secure a
insurable; that they knew he had been in bad health valuable contract which, upon a truthful
and had been under medical treatment for diabetes, report of his condition, could not have been
though he thought he was then well; that they assured obtained. By them the company was imposed
him that he was insurable, that the fact that he had upon and induced to enter into the contract.
In such a case, assuming that both parties
acted in good faith, justice would require He would have discovered by inspection that
that the contract be cancelled and the a fraud had been perpetrated, not only upon
premiums returned. As the present action is himself but upon the company, and it would
not for such a cancellation, the only recovery have been his duty to make the fact known
which the plaintiff could properly have upon to the company. He could not hold the policy
the facts he asserts, taken in connection without approving the action of the agents
with the limitation upon the powers of the and thus becoming a participant in the fraud
agent, is for the amount of the premiums committed. The retention of the policy was
paid, and to that only would he be entitled an approval of the application and of its
by virtue of the statute of Missouri. statements. The consequences of that
approval cannot after his death be avoided.
But the case as presented by the record is by
no means as favorable to him as we have . . . No one can claim the benefit of an
assumed. It was his duty to read the executory contract fraudulently obtained,
application he signed. He knew that upon it after the discovery of the fraud, without
the policy would be issued, if issued at all. It approving and sanctioning it. (117 U. S. 529-
would introduce great uncertainty in all 530, 534, 535.)
business transactions, if a party making
written proposals for a contract, with Our attention has been called to al later case
representations to induce its execution, Continental Life Insurance Company v. Chamberlain,
should be allowed to show, after it had been 132 U. S. 304 in which the court held the company
obtained, that he did not know the contents liable upon a policy of insurance of the life of one
of his proposals, and to enforce it, Richard Stevens issued under the following
notwithstanding their falsity as to matters circumstances, as stated in the decision:
essential to its obligation and validity.
Contracts could not be made, or business
fairly conducted, if such a rule should The application for insurance was taken in
prevail; and there is no reason why it should Iowa by one Boak, a district agent of the
be applied merely to contract of insurance. company in certain named counties of the
There is nothing in their nature which States, fourteen in number, having written
distinguishes them in this particular from authority "to prosecute the business of
others. But here the right is asserted to soliciting and procuring applications for life
prove not only that the assured did not make insurance policies within and throughout said
the statements contained in his answers, but territory."
that he never read the application, and to
recover upon a contract obtained by Among the numerous questions propounded
representations admitted to be false, just as in the application was the following: "Has the
though they were true. If he had read even said party [the applicant] any other
the printed lines of his application, he would insurance on his life; if so, where and for
have seen that it stipulated that the rights of what amounts?" The answer, as it appears in
the company could in no respect be affected the application, is: "No other." That answer,
by his verbal statements, or by those of its as were all the answers to questions
agents, unless the same were reduced to propounded to the applicant, was written by
writing and forwarded with his application to the company's agent, Boak. In reference to
the home office. The company, like any the above question and answer, the latter
other principal, could limit the authority of testified: "I asked him [Stevens] the question
its agents, and thus bind all parties dealing if he had any other insurance, as printed in
with them with knowledge of the limitation. the application and as we ask every
It must be presumed that he read the applicant, and he told me he had certain
application, and was cognizant of the certificates of membership with certain
limitations thereon expressed. cooperative societies, and he enumerated
different ones, and said he did not know
xxx xxx xxx whether I would consider them insurance or
not. I told him emphatically that I did not
consider them insurance and we had
There is another view of this case equally considerable conversation about it. He
fatal to a recovery. Assuming that the wanted to know my authority for saying I did
answers of the assured were falsified, as not consider them insurance. I gave him my
alleged, the fact would be at once disclosed authority give him my reasons and he
by the copy of the application, annexed to agreed with me that these cooperative
the policy, to which his attention was called. societies were in no sense insurance
companies, and in that light I answered the different. Suffice it for us to say that the facts of the
question "No" after he had stated the facts? instant case are analogous to those of the Fletcher
A. I did. Q. Who wrote the answer in there? case and different from those of the Chamberlain
A. I did." case.

xxx xxx xxx We have examined the three cases cited in the
majority opinion, from the supreme courts of
It was admitted on the trial that at the date Colorado, Iowa, and Indiana, respectively, and we
of Stevens' application he had insurance in find that the facts of each and everyone of them bear
cooperative companies to the amount of no analogy to those of the present case.
$12,000. (132 U. S. 306, 308.)
5. The majority opinion says: "The situation is one in
The court, after quoting the pertinent provision of the which one of two innocent parties must bear a loss for
statute of Iowa, observed that "by force of the his reliance upon a third person." We cannot subscribe
statute, he was the agent of the company in soliciting to this proposition (1) because, as we have pointed
and procuring the application. He could not, by any out above, the insured and his relatives, the herein
of his, shake of the character of agent for the respondents, were not innocent of bad faith and (2)
company. Nor could the company by any provision in because, even if the policies in question should be
the application or policy convert him into an agent of held invalid, the respondents would not suffer any
the assured." Referring to the incorrectness loss since the Company has offered to return the
propounded by him to the applicant in relation to the premiums paid, and it could be ordered to make such
stipulation in the policy that the terms thereof could refund with legal interest. By such judgment neither
not be varied except in writing signed by the party would be permitted to enrich himself at the
president or the secretary of the company, the court expense of the other. This, we feel, is urged by
said: justice, reason, and "the common sense of the
situation."
. . . The purport of the word "insurance" in
the question, "Has the said party any other Republic of the Philippines
insurance on his life?" is not so absolutely SUPREME COURT
certain as, in an action upon the policy, to Manila
preclude proof as to what kind of life
insurance the contracting parties had in EN BANC
mind when that question was answered.
Such proof does not necessarily contradict G.R. No. L-47593 December 29, 1943
the written contract. Consequently, the
above clause, printed on the back of the
policy, is to be interpreted in the light of the THE INSULAR LIFE ASSURANCE CO., LTD.,
statute and of the understanding reached petitioner,
between the assured and the company by its vs.
agent when the application was completed, SERAFIN D. FELICIANO ET AL., respondents.
namely, that the particular kind of insurance
inquired about did not include insurance in OZAETA, J.:
cooperative societies. In view of the statute
and of that understanding, upon the faith of In a four-to-three decision promulgated on September
which the assured made his application, paid 13, 1941, 1 this Court affirmed the judgment of the
the first premium, and accepted the policy, Court of Appeals in favor of the respondents and
the company is estopped, by every principle against the petitioner for the sum of P25,000,
of justice, from saying that its question representing the value of two insurance policies
embraced insurance in cooperative issued by the petitioner on the life of Evaristo
associations. The answer of "No other" having Feliciano. A motion to reconsider and set aside said
been written by its own agent, invested with decision has been filed by the petitioner, and both
authority to solicit and procure applications, parties have submitted exhaustive and luminous
to deliver policies, and, under certain written arguments in support of their respective
limitations, to receive premiums, should be contentions.
held as properly interpreting both the
question and the answer as to other
insurance. (132 U. S. 311-312.) The facts of the case are set forth in the majority and
dissenting opinions heretofore handed down by this
Court, the salient points of which may be briefly
There is no conflict between the two cases. They restated as follows:
were decided differently because the facts were
Evaristo Feliciano, who died on September 29, 1935, bound by any promise or representation
was suffering with advanced pulmonary tuberculosis heretofore or hereafter given by any person
when he signed his applications for insurance with the other than the above-named officials, and
petitioner on October 12, 1934. On that same date by them only in writing and signed conjointly
Doctor Trepp, who had taken X-ray pictures of his as stated.
lungs, informed the respondent Dr. Serafin D.
Feliciano, brother of Evaristo, that the latter "was The application contains, among others, the following
already in a very serious ad practically hopeless statements:
condition." Nevertheless the question contained in
the application "Have you ever suffered from any
ailment or disease of the lungs, pleurisy, pneumonia 18. I [the applicant] hereby declare that
or asthma?" appears to have been answered , "No" all the above statements and answers as well
And above the signature of the applicant, following as all those that I may make to the Company's
the answers to the various questions propounded to Medical Examiner in continuation of this
him, is the following printed statement:1awphil.net application, to be complete, true and
correct to the best of my knowledge and
belief, and I hereby agree as follows:
I declare on behalf of myself and of any
person who shall have or claim any interest
in any policy issued hereunder, that each of 1. That his declaration, with the answers to
the above answers is full, complete and true, be given by me to the Medical Examiner,
and that to the best of my knowledge and shall be the basis of the policy and form part
belief I am a proper subject for life of same.
insurance. (Exhibit K.)
xxx xxx xxx
The false answer above referred to, as well as the
others, was written by the Company's soliciting agent 3. That the said policy shall not take effect
Romulo M. David, in collusion with the medical until the first premium has been paid and the
examiner Dr. Gregorio Valdez, for the purpose of policy has been delivered to and accepted by
securing the Company's approval of the application so me, while I am in good health.
that the policy to be issued thereon might be credited
to said agent in connection with the inter-provincial 4. That the agent taking this application has
contest which the Company was then holding among no authority to make, modify or discharge
its soliciting agents to boost the sales of its policies. contracts, or to waive any of the Company's
Agent David bribed Medical Examiner Valdez with rights or requirements.
money which the former borrowed from the
applicant's mother by way of advanced payment on
the premium, according to the finding of the Court of 5. My acceptance of any policy issued on this
Appeals. Said court also found that before the insured application will constitute a ratification by
signed the application he, as well as the members of me of any corrections in or additions to this
his family, told the agent and the medical examiner application made by the Company in the
that he had been sick and coughing for some time and space provided "For Home Office Corrections
that he had gone three times to the Santol Sanatorium or Additions Only." I agree that photographic
and had X-ray pictures of his lungs taken; but that in copy of this applications as corrected or
spite of such information the agent and the medical added to shall constitute sufficient notice to
examiner told them that the applicant was a fit me of the changes made. (Emphasis added.)
subject for insurance.
The petitioner insists that upon the facts of the case
Each of the policies sued upon contains the following the policies in question are null and void ab initio and
stipulations: that all that the respondents are entitled to is the
refund of the premiums paid thereon. After a careful
re-examination of the facts and the law, we are
This policy and the application herefor persuaded that petitioner's contention is correct. To
constitute the entire contract between the the reasons adduced in the dissenting opinion
parties hereto. . . . Only the President, or heretofore published, we only desire to add the
the Manager, acting jointly with the following considerations:
Secretary or Assistant Secretary (and then
only in writing signed by them) have power
in behalf of the Company to issue permits, or When Evaristo Feliciano, the applicant for insurance,
to modify this or any contract, or to extend signed the application in blank and authorized the
the same time for making any premium soliciting agent and/or medical examiner of the
payment, and the Company shall not be Company to write the answers for him, he made them
his own agents for that purpose, and he was upon the oral representation of said agent and
responsible for their acts in that connection. If they medical examiner that he (the applicant) was a fit
falsified the answers for him, he could not evade the subject for insurance notwithstanding that he had
responsibility for he falsification. He was not been and was still suffering with advanced pulmonary
supposed to sign the application in blank. He knew tuberculosis.
that the answers to the questions therein contained
would be "the basis of the policy," and for that every From all the facts and circumstances of this case, we
reason he was required with his signature to vouch for are constrained to conclude that the insured was a
truth thereof. coparticipant, and coresponsible with Agent David
and Medical Examiner Valdez, in the fraudulent
Moreover, from the facts of the case we cannot procurement of the policies in question and that by
escape the conclusion that the insured acted in reason thereof said policies are void ab initio.
connivance with the soliciting agent and the medical
examiner of the Company in accepting the policies in Wheretofore, the motion for reconsideration is
question. Above the signature of the applicant is the sustained and the judgment of the Court of Appeals is
printed statement or representation: " . . . I am a hereby reversed. Let another judgment be entered in
proper subject for life insurance." In another sheet of favor of the respondents and against the petitioner
the same application and above another signature of for the refund of the premiums amounting to P1,389,
the applicant was also printed this statement: "That with legal interest thereon from the date of the
the said policy shall not take effect until he first complaint, and without any finding as to costs.
premium has been paid and the policy as been
delivered to and accepted by me, while I am in good
health." When the applicant signed the application he Insular Life v. Feliciano - Concealment
was "having difficulty in breathing, . . . with a very
high fever." He had gone three times to the Santol 73 PHIL 201
Sanatorium and had X-ray pictures taken of his lungs.
He therefore knew that he was not "a proper subject Facts:
for life insurance." When he accepted the policy, he
knew that he was not in good health. Nevertheless,
he not only accepted the first policy of P20,000 but > Evaristo Feliciano filed an application with Insular
then and there applied for and later accepted another Life upon the solicitation of one of its agents.
policy of P5,000.
> It appears that during that time, Evaristo was
We cannot bring ourselves to believe that the insured already suffering from tuberculosis. Such fact
did not take the trouble to read the answers appeared during the medical exam, but the examiner
contained in the photostatic copy of the application and the companys agent ignored it.
attached to and made a part of the policy before he
accepted it and paid the premium thereon. He must > After that, Evaristo was made to sign an application
have notice that the answers to the questions therein form and thereafter the blank spaces were filled by
asked concerning his clinical history were false, and the medical examiner and the agent making it appear
yet he accepted the first policy and applied for that Evaristo was a fit subject of insurance. (Evaristo
another. In any event, he obligated himself to read could not read and understand English)
the policy when he subscribed to this statement: "My
acceptance of any policy issued on this application > When Evaristo died, Insular life refused to pay the
will constitute a ratification by me of any corrections proceeds because of concealment.
in or additions to this application made by the
Company . . ." By accepting the policy he became
charged with knowledge of its contents, whether he
actually read it or not. He could not ostrich-like hide
his head from it in order to avoid his part of the Issue:
bargain and at the same time claim the benefit
thereof. He knew, or was chargeable with knowledge, Whether or not Insular Life was bound by their agents
from the very terms of the two policies sued upon acts.
(one of which is printed in English and the other in
Spanish) that the soliciting agent and the medical
examiner had no power to bind the Company by any
verbal promise or oral representation. The insured,
therefore, had no right to rely and we cannot Held:
believe he relied in good faith upon the oral
representation. The insured, therefore, had no right Yes.
to rely and we cannot believe he relied in good faith
The insurance business has grown so vast and The Court of Appeals certified this appeal to Us, as
lucrative within the past century. Nowadays, even the same involves solely a question of law.
people of modest means enter into insurance
contracts. Agents who solicit contracts are paid large On May 12, 1962, Kwong Nam applied for a 20-year
commissions on the policies secured by them. They endowment insurance on his life for the sum of
act as general representatives of insurance P20,000.00, with his wife, appellee Ng Gan Zee as
companies. beneficiary. On the same date, appellant, upon
receipt of the required premium from the insured,
approved the application and issued the
corresponding policy. On December 6, 1963, Kwong
IN the case at bar, the true state of health of the Nam died of cancer of the liver with metastasis. All
insured was concealed by the agents of the premiums had been religiously paid at the time of his
insurer. The insurers medical examiner approved death.
the application knowing fully well that the applicant
was sick. The situation is one in which of two On January 10, 1964, his widow Ng Gan Zee presented
innocent parties must bear a loss for his reliance upon a claim in due form to appellant for payment of the
a third person. In this case, it is the one who drafted face value of the policy. On the same date, she
and accepted the policy and consummated the submitted the required proof of death of the insured.
contract. It seems reasonable that as between the Appellant denied the claim on the ground that the
two of them, the one who employed and gave answers given by the insured to the questions
character to the third person as its agent should be appealing in his application for life insurance were
the one to bear the loss. Hence, Insular is liable to untrue.
the beneficiaries.
Appellee brought the matter to the attention of the
Republic of the Philippines Insurance Commissioner, the Hon. Francisco Y.
SUPREME COURT Mandamus, and the latter, after conducting an
Manila investigation, wrote the appellant that he had found
no material concealment on the part of the insured
SECOND DIVISION and that, therefore, appellee should be paid the full
face value of the policy. This opinion of the Insurance
Commissioner notwithstanding, appellant refused to
G.R. No. L-30685 May 30, 1983 settle its obligation.

NG GAN ZEE, plaintiff-appellee, Appellant alleged that the insured was guilty of
vs. misrepresentation when he answered "No" to the
ASIAN CRUSADER LIFE ASSURANCE CORPORATION, following question appearing in the application for
defendant-appellant. life insurance-

Alberto Q. Ubay for plaintiff-appellee. Has any life insurance company


ever refused your application for
Santiago F. A lidio for defendant-appellant. insurance or for reinstatement of a
lapsed policy or offered you a policy
different from that applied for? If,
so, name company and date.
ESCOLIN, J.:
In its brief, appellant rationalized its thesis thus:
This is an appeal from the judgment of the Court of
First Instance of Manila, ordering the appellant Asian- ... As pointed out in the foregoing
Crusader Life Assurance Corporation to pay the face summary of the essential facts in
value of an insurance policy issued on the life of this case, the insured had in
Kwong Nam the deceased husband of appellee Ng Gan January, 1962, applied for
Zee. Misrepresentation and concealment of material reinstatement of his lapsed life
facts in obtaining the policy were pleaded to avoid insurance policy with the Insular
the policy. The lower court rejected the appellant's Life Insurance Co., Ltd, but this was
theory and ordered the latter to pay appellee "the declined by the insurance company,
amount of P 20,000.00, with interest at the legal rate although later on approved for
from July 24, 1964, the date of the filing of the reinstatement with a very high
complaint, until paid, and the costs. " premium as a result of his medical
examination. Thus notwithstanding
the said insured answered 'No' to the Court finds that there is no
the [above] question propounded to misrepresentation on this matter. 2
him. ... 1
Appellant further maintains that when the insured
The lower court found the argument bereft of factual was examined in connection with his application for
basis; and We quote with approval its disquisition on life insurance, he gave the appellant's medical
the matter- examiner false and misleading information as to his
ailment and previous operation. The alleged false
On the first question there is no statements given by Kwong Nam are as follows:
evidence that the Insular Life
Assurance Co., Ltd. ever refused Operated on for a Tumor [mayoma]
any application of Kwong Nam for of the stomach. Claims that Tumor
insurance. Neither is there any has been associated with ulcer of
evidence that any other insurance stomach. Tumor taken out was hard
company has refused any and of a hen's egg size. Operation
application of Kwong Nam for was two [2] years ago in Chinese
insurance. General Hospital by Dr. Yap. Now,
claims he is completely recovered.
... The evidence shows that the
Insular Life Assurance Co., Ltd. To demonstrate the insured's misrepresentation,
approved Kwong Nam's request for appellant directs Our attention to:
reinstatement and amendment of
his lapsed insurance policy on April [1] The report of Dr. Fu Sun Yuan the physician who
24, 1962 [Exh. L-2 Stipulation of treated Kwong Nam at the Chinese General Hospital
Facts, Sept. 22, 1965). The Court on May 22, 1960, i.e., about 2 years before he applied
notes from said application for for an insurance policy on May 12, 1962. According to
reinstatement and amendment, said report, Dr. Fu Sun Yuan had diagnosed the
Exh. 'L', that the amount applied for patient's ailment as 'peptic ulcer' for which, an
was P20,000.00 only and not for operation, known as a 'sub-total gastric resection was
P50,000.00 as it was in the lapsed performed on the patient by Dr. Pacifico Yap; and
policy. The amount of the
reinstated and amended policy was
also for P20,000.00. It results, [2] The Surgical Pathology Report of Dr. Elias
therefore, that when on May 12, Pantangco showing that the specimen removed from
1962 Kwong Nam answered 'No' to the patient's body was 'a portion of the stomach
the question whether any life measuring 12 cm. and 19 cm. along the lesser
insurance company ever refused his curvature with a diameter of 15 cm. along the
application for reinstatement of a greatest dimension.
lapsed policy he did not
misrepresent any fact. On the bases of the above undisputed medical data
showing that the insured was operated on for peptic
... the evidence shows that the ulcer", involving the excision of a portion of the
application of Kwong Nam with the stomach, appellant argues that the insured's
Insular Life Assurance Co., Ltd. was statement in his application that a tumor, "hard and
for the reinstatement and of a hen's egg size," was removed during said
amendment of his lapsed insurance operation, constituted material concealment.
policy-Policy No. 369531 -not an
application for a 'new insurance The question to be resolved may be propounded thus:
policy. The Insular Life Assurance Was appellant, because of insured's aforesaid
Co., Ltd. approved the said representation, misled or deceived into entering the
application on April 24, 1962. Policy contract or in accepting the risk at the rate of
No. 369531 was reinstated for the premium agreed upon?
amount of P20,000.00 as applied for
by Kwong Nam [Exhs. 'L', 'L-l' and 'L- The lower court answered this question in the
2']. No new policy was issued by the negative, and We agree.
Insular Life Assurance Co., Ltd. to
Kwong Nam in connection with said
application for reinstatement and Section 27 of the Insurance Law [Act 2427] provides:
amendment. Such being the case,
Sec. 27. Such party a contract of either by the terms of insurance or
insurance must communicate to the by neglect to make inquiries as to
other, in good faith, all facts within such facts where they are distinctly
his knowledge which are material to implied in other facts of which
the contract, and which the other information is communicated.
has not the means of ascertaining,
and as to which he makes no It has been held that where, upon the face of the
warranty. 3 application, a question appears to be not answered at
all or to be imperfectly answered, and the insurers
Thus, "concealment exists where the assured had issue a policy without any further inquiry, they waive
knowledge of a fact material to the risk, and honesty, the imperfection of the answer and render the
good faith, and fair dealing requires that he should omission to answer more fully immaterial. 6
communicate it to the assurer, but he designedly and
intentionally withholds the same." 4 As aptly noted by the lower court, "if the ailment and
operation of Kwong Nam had such an important
It has also been held "that the concealment must, in bearing on the question of whether the defendant
the absence of inquiries, be not only material, but would undertake the insurance or not, the court
fraudulent, or the fact must have been intentionally cannot understand why the defendant or its medical
withheld." 5 examiner did not make any further inquiries on such
matters from the Chinese General Hospital or require
Assuming that the aforesaid answer given by the copies of the hospital records from the appellant
insured is false, as claimed by the appellant. Sec. 27 before acting on the application for insurance. The
of the Insurance Law, above-quoted, nevertheless fact of the matter is that the defendant was too eager
requires that fraudulent intent on the part of the to accept the application and receive the insured's
insured be established to entitle the insurer to rescind premium. It would be inequitable now to allow the
the contract. And as correctly observed by the lower defendant to avoid liability under the circumstances."
court, "misrepresentation as a defense of the insurer
to avoid liability is an 'affirmative' defense. The duty Finding no reversible error committed by the trial
to establish such a defense by satisfactory and court, the judgment appealed from is hereby
convincing evidence rests upon the defendant. The affirmed, with costs against appellant Asian-Crusader
evidence before the Court does not clearly and life Assurance Corporation.
satisfactorily establish that defense."
SO ORDERED.
It bears emphasis that Kwong Nam had informed the
appellant's medical examiner that the tumor for Republic of the Philippines
which he was operated on was "associated with ulcer SUPREME COURT
of the stomach." In the absence of evidence that the Manila
insured had sufficient medical knowledge as to enable
him to distinguish between "peptic ulcer" and "a
tumor", his statement that said tumor was "associated EN BANC
with ulcer of the stomach, " should be construed as an
expression made in good faith of his belief as to the G.R. No. L-12707 August 10, 1918
nature of his ailment and operation. Indeed, such
statement must be presumed to have been made by MRS. HENRY E. HARDING, and her husband,
him without knowledge of its incorrectness and plaintiffs-appellees,
without any deliberate intent on his part to mislead vs.
the appellant. COMMERCIAL UNION ASSURANCE COMPANY,
defendant-appellant.
While it may be conceded that, from the viewpoint of
a medical expert, the information communicated was Lawrence & Ross for appellant.
imperfect, the same was nevertheless sufficient to Gibbs, McDonough & Johnson for appellees.
have induced appellant to make further inquiries
about the ailment and operation of the insured.
FISHER, J.:
Section 32 of Insurance Law [Act No. 24271 provides
as follows: This was an action by plaintiffs to recover from
defendant the sum of P3,000 and interest, alleged to
be due under the terms of a policy of insurance. The
Section 32. The right to information trial court gave plaintiffs judgment for the amount
of material facts maybe waived
demanded, with interest and costs, and from that delivered by said plaintiff to the defendant,
decision the defendant appeals. guaranteeing the truth of the statements
contained therein which said proposal is
The court below stated the issues made by the referred to in the said policy of insurance
pleadings in this case, and its finding of fact, as made a part thereof; (2) that certain of the
follows: statements and representations contained in
said proposal and warranted by said plaintiff
to be true, to wit: (a) the price paid by the
It is alleged by plaintiffs and admitted by proposer for the said automobile; (b) the
defendant that plaintiffs are husband and value of said automobile at the time of the
wife and residents of the city of Manila; that execution and delivery of the said proposal
the defendant is a foreign corporation and (c) the ownership of said automobile,
organized and existing under and by virtue of were false and known to be false by the said
the laws of Great Britain and duly registered plaintiff at the time of signing and delivering
in the Philippine Islands, and Smith, Bell & the said proposal and were made for the
Co. (limited), a corporation organized and purpose of misleading and deceiving the
existing under the laws of the Philippine defendant, and inducing the defendant,
Islands, with its principal domicile in the city relying upon the warranties, statements, and
of Manila, is the agent in the Philippine representations contained in the said
Islands of said defendant. proposal and believing the same to be true,
issued the said policy of insurance.
The plaintiffs alleged that on February 16,
1916, the plaintiff Mrs. Henry E. Harding was The defendant prays that judgment be
the owner of a Studebaker automobile, entered declaring the said policy of
registered number 2063, in the city of insurance to be null and void, and that
Manila; that on said date; in consideration of plaintiffs take nothing by this action; and for
the payment to the defendant of the such further relief as to the court may seem
premium of P150, by said plaintiff, Mrs. just and equitable.
Henry E. Harding, with the consent of her
husband, the defendant by its duly
authorized agent, Smith, Bell & Company The evidence in this case shows that some
(limited), made its policy of insurance in time in the year 1913 Levy Hermanos, the
writing upon said automobile was set forth in Manila agents for the Studebaker
said policy to be P3,000 that the value of said automobile, sold the automobile No. 2063 to
automobile was set forth in said policy John Canson for P3,200 (testimony of Mr.
(Exhibit A) to be P3,000; that on March 24, Diehl); that under date of October 14, 1914,
1916, said automobile was totally destroyed John Canson sold the said automobile to
by fire; that the loss thereby to plaintiffs was Henry Harding for the sum of P1,500 (Exhibit
the sum of P3,000; that thereafter, within 2); that under date of November 19, 1914,
the period mentioned in the said policy of the said Henry Harding sold the said
insurance, the plaintiff, Mrs. Henry E. automobile No. 2063 to J. Brannigan, of Los
Harding, furnished the defendant the proofs Baos, Province of Laguna, P.I., for the sum
of her said loss and interest, and otherwise of P2,000 (Exhibit 3); that under date of
performed all the conditions of said policy on December 20, 1915, J. C. Graham of Los
her part, and that the defendant has not paid Baos, Province of Laguna, P.I., sold the said
said loss nor any part thereof, although due automobile No. 2063 to Henry Harding of the
demand was made upon defendant therefor. city of Manila for the sum of P2,800 (Exhibit
4 and testimony of J. C. Graham); that on or
about January 1, 1916, the said Henry
The defendant, by its answer, admitted the Harding gave the said automobile to his wife;
allegations of the residence and status of the Mrs. Henry E. Harding, one of the plaintiffs,
parties and denied all the other allegation of as a present; that said automobile was
the said complaint, and for a separate and repaired and repainted at the Luneta Garage
affirmative defense alleged (1) that on at a cost of some P900 (testimony of Mr.
February 17, 1916, at the city of Manila, P.I. Server); that while the said automobile was
the defendant upon request of plaintiff, Mrs. at the Luneta Garage; the said Luneta
Henry E. Harding, issued to the said plaintiff Garage, acting as agent for Smith, Bell &
the policy of insurance on an automobile Company, (limited), solicited of the plaintiff
alleged by the said plaintiff to be her Mrs. Harding the insurance of said
property; that the said request for the automobile by the defendant Company
issuance of said policy of insurance was made (testimony of Mrs. Henry Harding and Mr.
by means of a proposal in writing signed and Server); that a proposal was filled out by the
said agent and signed by the plaintiff Mrs. The evidence further shows that on March
Henry E. Harding, and in said proposal under 24, 1916, the said automobile was totally
the heading "Price paid by proposer," is the destroyed by fire, and that the iron and steel
amount of "3,500" and under another heading portions of said automobile which did not
"Present value" is the amount of "3,000" burn were taken into the possession of the
(Exhibit 1). defendant by and through its agent Smith,
Bell & Company (limited), and sold by it for
The evidence tends to show that after the a small sum, which had never been tendered
said proposal was made a representative of to the plaintiff prior to the trial of this case,
the Manila agent of defendant went to the but in open court during the trial the sum of
Luneta Garage and examined said P10 as the proceeds of such sale was
automobile No. 2063 and Mr. Server, the tendered to plaintiff and refused.
General Manager of the Luneta Garage, an
experienced automobile mechanic, testified Upon the facts so found, which we hold are supported
that at the time this automobile was insured by the evidence, the trial judge decided that there
it was worth about P3,000, and the was no proof of fraud on the part of plaintiff in her
defendant, by and through its said agent statement of the value of the automobile, or with
Smith, Bell & Company (limited), thereafter respect to its ownership; that she had an insurable
issued a policy of insurance upon proposal in interest therein; and that defendant, having agreed
which policy the said automobile was to the estimated value, P3,000, and having insured
described as of the "present value" of P3,000 the automobile for that amount, upon the basis of
and the said defendant charged the said which the premium was paid, is bound by it and must
plaintiff Mrs. Henry E. Harding as premium pay the loss in accordance with the stipulated insured
on said policy the sum of P150, or 5 per cent value. The assignments of error made on behalf of
of the then estimated value of P3,000. appellant put in issue the correctness of those
(Exhibit A.) conclusions of law, and some others of minor
importance relating to the exclusion of evidence.
The "Schedule" in said policy of insurance Disposing of the minor objections first, as we have
describes the automobile here in question, reached the conclusion that the trial court was right
and provides in part of follows: in holding that the defendant is bound by the
estimated value of the automobile upon which policy
was issued, and that the plaintiff was not guilty of
"Now it is hereby agreed as follows: fraud in regard thereto, the exclusion of the
testimony of the witness Diehl is without importance.
"That during the period above set It merely tended to show the alleged actual value of
forth and during any period for the automobile, and in the view we take of the case
which the company may agree to such evidence was irrelevant.
renew this policy the company will
subject to the exception and Appellant contends that Mrs. Harding was not the
conditions contained herein or owner of the automobile at the time of the issuance
endorsed hereon indemnify the of the policy, and, therefore, had no insurable
insured against loss of or damage to interest in it. The court below found that the
any motor car described in the automobile was given to plaintiff by her husband
schedule hereto (including shortly after the issuance of the policy here in
accessories) by whatever cause question. Appellant does not dispute the correctness
such loss or damage may be of this finding, but contends that the gift was void,
occasioned and will further citing article 1334 of the Civil Code which provides
indemnify the insured up to the that "All gifts between spouses during the marriage
value of the car or P3,000 shall be void. Moderate gifts which the spouses
whichever is the greater against any bestow on each other on festive days of the family are
claim at common law made by any not included in this rule."
person (not being a person in the
said motor car nor in the insured's
service) for loss of life or for We are of the opinion that this contention is without
accidental bodily injury or damage merit. In the case of Cook vs. McMicking 27 Phil. Rep.,
to property caused by the said 10), this court said:
motor car including law costs
payable in connection with such It is claimed by the appellants that the so-
claim when incurred with the called transfer from plaintiff's husband to
consent of the company." her was completely void under article 1458
of the Civil Code and that, therefore, the
property still remains the property of Edward because the outlay represented by the automobile
Cook and subject to levy under execution was made by the plaintiff's husband and not by his
against him. wife, to whom he had given the automobile. It cannot
be assumed that defendant should not have issued the
In our opinion the position taken by policy unless it were strictly true that the price
appellants is untenable. They are not in a representing the cost of the machine had been paid
position to challenge the validity of the by the insured and by no other person that it would
transfer, if it may be called such. They bore no event insure an automobile acquired by gift,
absolutely no relation to the parties to the inheritance, exchange, or any other title not requiring
transfer at the time it occurred and had no the owner to make a specific cash outlay for its
rights or interests inchoate, present, acquisition.
remote, or otherwise, in the property in
question at the time the transfer occurred. Furthermore, the court below found and the evidence
Although certain transfers from husband to shows, without dispute, that the proposal upon which
wife or from wife to husband are prohibited the policy in question was issued was made out by
in the article referred to, such prohibition defendant's agent by whom the insurance was
can be taken advantage of only by persons solicited, and that appellee simply signed the same.
who bear such a relation to the parties It also appears that an examiner employed by the
making the transfer or to the property itself defendant made an inspection of the automobile
that such transfer interferes with their rights before the acceptance of the risk, and that the sum
or interests. Unless such a relationship after this examination. The trial court found that Mrs.
appears the transfer cannot be attacked. Harding, in fixing the value of the automobile at
P3,000, acted upon information given her by her
Even assuming that defendant might have invoked husband and by Mr. Server, the manager of the Luneta
article 1334 as a defense, the burden would be upon Garage. The Luneta Garage, it will be remembered,
it to show that the gift in question does not fall within was the agent of the defendant corporation in the
the exception therein established. We cannot say, as solicitation of the insurance. Mrs. Harding did not
a matter of law, that the gift of an automobile by a state of her own knowledge that the automobile
husband to his wife is not a moderate one. Whether it originally cost P3,000, or that its value at the time of
is or is not would depend upon the circumstances of the insurance was P3,000. She merely repeated the
the parties, as to which nothing is disclosed by the information which had been given her by her husband,
record. and at the same time disclosed to defendant's agent
the source of her information. There is no evidence to
sustain the contention that this communication was
Defendant contends that the statement regarding the made in bad faith. It appears that the statements in
cost of the automobile was a warranty, that the the proposal as to the price paid for the automobile
statement was false, and that, therefore, the policy and as to its value were written by Mr. Quimby who
never attached to the risk. We are of the opinion that solicited the insurance on behalf of defendant, in his
it has not been shown by the evidence that the capacity as an employee of the Luneta Garage, and
statement was false on the contrary we believe that wrote out the proposal for Mrs. Harding to sign. Under
it shows that the automobile had in fact cost more these circumstances, we do not think that the facts
than the amount mentioned. The court below found, stated in the proposal can be held as a warranty of
and the evidence shows, that the automobile was the insured, even if it should have been shown that
bought by plaintiff's husband a few weeks before the they were incorrect in the absence of proof of willful
issuance of the policy in question for the sum of misstatement. Under such circumstance, the proposal
P2,800, and that between that time and the issuance is to be regarded as the act of the insurer and not of
of the policy some P900 was spent upon it in repairs the insured. This question was considered in the case
and repainting. The witness Server, an expert of the Union Insurance Company vs. Wilkinson (13
automobile mechanic, testified that the automobile Wall., 222; 20 L. ed., 617), in which the Supreme
was practically as good as new at the time the Court of the United States said:
insurance was effected. The form of proposal upon
which the policy was issued does not call for a
statement regarding the value of the automobile at This question has been decided differently
the time of its acquisition by the applicant for the by courts of the highest respectability in
insurance, but merely a statement of its cost. The cases precisely analogous to the present. It
amount stated was less than the actual outlay which is not to be denied that the application
the automobile represented to Mr. Harding, including logically considered, is the work of the
repairs, when the insurance policy was issued. It is assured, and if left to himself or to such
true that the printed form calls for a statement of the assistance as he might select, the person so
"price paid by the proposer," but we are of the opinion selected would be his agent, and he alone
that it would be unfair to hold the policy void simply would be responsible. On the other hand, it
is well-known, so well that no court would be
justified in shutting its eyes to it, that of his employment, as if they proceeded
insurance companies organized under the from the principal. (Sav. Bk. vs. Ins. Co., 31
laws of one State, and having in that State Conn., 517; Hortwitz vs. Ins. Co., 40 Mo.,
their principal business office, send these 557; Ayres vs. Ins. Co., 17 Iowa, 176; Howard
agents all over the land, with directions to Ins. Co. vs. Bruner, 23 Pa., 50.)
solicit and procure applications for policies
furnishing them with printed arguments in In the fifth edition of American Leading
favor of the value and necessity of life Cases, 917, after a full consideration of the
insurance, and of the special advantages of authorities, it is said:
the corporation which the agent represents.
They pay these agents large commissions on
the premiums thus obtained, and the policies "By the interested or officious zeal
are delivered at their hands to the assured. of the agents employed by the
The agents are stimulated by letters and insurance companies in the wish to
instructions to activity in procuring outbid each other and procure
contracts, and the party who is in this customers, they not unfrequently
manner induced to take out a policy, rarely mislead the insured, by a false or
sees or knows anything about the company erroneous statement of what the
or its officers by whom it is issued, but looks application should contain; or,
to and relies upon the agent who has taking the preparation of it into
persuaded him to effect insurance as the full their own hands, procure his
and complete representative of the signature by an assurance that it is
company, in all that is said or done in making properly drawn, and will meet the
the contract. Has he not a right to so regard requirements of the policy. The
him? It is quite true that the reports of better opinion seems to be that,
judicial decisions are filled with the efforts when this course is pursued, the
of these companies, by their counsel, to description of the risk should,
establish the doctrine for the acts of these though nominally proceeding from
agents to the simple receipt of the premium the insured, be regarded as the act
and delivery of the policy, the argument of the insurers." (Rowley vs. Empire
being that, as to all other acts of the agent, Ins. Co., 36 N.Y., 550.)
he is the agent of the assured. This
proposition is not without support in some of The modern decisions fully sustain this
the earlier decision on the subject; and, at a proposition, and they seem to us founded on
time when insurance companies waited for reason and justice, and meet our entire
parties to come to them to seek assurance, approval. This principle does not admit oral
or to forward applications on their own testimony to vary or contradict that which is
motion, the doctrine had a reasonable in writing, but it goes upon the idea that the
foundation to rest upon. But to apply such a writing offered in evidence was not the
doctrine, in its full force, to the system of instrument of the party whose name is signed
selling policies through agents, which we to it; that it was procured under such
have described, would be a snare and a circumstances by the other side as estops
delusion, leading, as it has done in numerous that side from using it or relying on its
instances, to the grossest frauds, of which contents; not that it may be contradicted by
the insurance corporations receive the oral testimony, but that it may be shown by
benefits, and the parties supposing such testimony that it cannot be lawfully
themselves insured are the victims. The used against the party whose name is signed
tendency of the modern decisions in this to it. (See also Am. Life Ins. Co. vs. Mahone,
country is steadily in the opposite direction. 21 Wallace, 152.)
The powers of the agent are, prima facie,
co-extensive with the business intrusted to The defendant, upon the information given by
his care, and will not be narrowed by plaintiff, and after an inspection of the automobile by
limitations not communicated to the person its examiner, having agreed that it was worth P3,000,
with whom he deals. (Bebee vs. Ins. Co., 25 is bound by this valuation in the absence of fraud on
Conn., 51; Lycoming Ins. Co. vs. the part of the insured. All statements of value are,
Schoolenberger, 44 Pa., 259; Beal vs. Ins. of necessity, to a large extent matters of opinion, and
Co., 16 Wis., 241; Davenport vs. Ins. Co., 17 it would be outrageous to hold that the validity of all
Iowa, 276.) An insurance company, valued policies must depend upon the absolute
establishing a local agency, must be held correctness of such estimated value. As was said by
responsible to the parties with whom they the Supreme Court of the United States in the case of
transact business, for the acts and the First National Bank vs. Hartford Fire Insurance Co.
declarations of the agent, within the scope
(5 Otto, 673; 24 L. ed., 563), at. p. 565 of the Lawyers Republic of the Philippines
Edition: SUPREME COURT
Manila
The ordinary test of the value of property is
the price it will commend in the market if EN BANC
offered for sale. But that test cannot, in the
very nature of the case, be applied at the G.R. No. L-41794 August 30, 1935
time application is made for insurance. Men
may honestly differ about the value of
property, or as to what it will bring in the SEGUNDINA MUSGI, ET AL., plaintiffs-appellees,
market; and such differences are often very vs.
marked among those whose special business WEST COAST LIFE INSURANCE CO., defendant-
it is to buy and sell property of all kinds. The appellant.
assured could do no more than estimate such
value; and that, it seems, was all that he was Courtney Whitney for appellants.
required to do in this case. His duty was to Laurel, Del Rosario and Sabido for appellees.
deal fairly with the Company in making such
estimate. The special finding shows that he IMPERIAL, J.:
discharged that duty and observed good
faith. We shall not presume that the
Company, after requiring the assured in his The plaintiffs, as beneficiaries, brought suit against
application to give the "estimated value," the defendant to recover the value of two life
and then to covenant that he had stated all insurance policies. The defendant appealed from a
material facts in regard to such value, so far judgment sentencing it to pay the plaintiffs the
as known to him, and after carrying that amount of said policies, and the costs.
covenant, by express words, into the written
contract, intended to abandon the theory The principal facts of the case are embodied in the
upon which it sought the contract, and make following written stipulation entered into by the
the absolute correctness of such estimated parties:
value a condition precedent to any insurance
whatever. The application, with its covenant 1. That Arsenio T. Garcia was insured by the
and stipulations, having been made a part of defendant company in the sum of P5,000 as
the policy, that presumption cannot be evidenced by Policy No. 129454 effective as
indulged without imputing to the Company a of July 25, 1931, hereby attached and
purpose, by studied intricacy or an ingenious marked as Exhibit A;
framing of the policy, to entrap the assured
into incurring obligations which, perhaps, he
had no thought of assuming. 2. That the said Arsenio T. Garcia was again
insured by the defendant company in the
sum of P10,000 effective as of October 20,
Section 163 of the Insurance Law (Act No. 2427) 1931, as evidenced by Policy No. 130381
provides that "the effect of a valuation in a policy of hereby attached and marked as Exhibit B;
fire insurance is the same as in a policy of marine
insurance."
3. That the two policies aforementioned
were valid and subsisting at the time of the
By the terms of section 149 of the Act cited, the death of the insured on December 30, 1932;
valuation in a policy of marine insurance is conclusive the fact of said death is evidenced by the
if the insured had an insurable interest and was not accompanying death certificate issued by
guilty of fraud. the Civil Register of Pasay, Rizal, which is
marked as Exhibit C;
We are, therefore, of the opinion and hold that
plaintiff was the owner of the automobile in question 4. That the plaintiffs herein are the
and had an insurable interest therein; that there was beneficiaries in said policies, Segundina
no fraud on her part in procuring the insurance; that Musgi of Policy No. 129454, and
the valuation of the automobile, for the purposes of Buenaventura Garcia of Policy No. 130381;
the insurance, is binding upon the defendant
corporation, and that the judgment of the court
below is, therefore, correct and must be affirmed, 5. That demand was made upon the
with interest, the costs of this appeal to be paid by defendant company for the payment of the
the appellant. So ordered. two policies above referred to, but the
defendant company refused to pay on the question here is not whether the physicians' reports
grounds stated in the answer. or the answers which the insured gave to them
relative to his health were correct or not. It is
The two policies were issued upon applications filed admitted that such information was substantially
by the insured on July 20, 1931 and October 15, of the correct, in the sense that the physicians of the
same year, respectively. In both applications, the defendant who examined the insured, for failure to
insured had to answer inquiries as to his state of make a detailed examination, did not discover the
health and that of his family, which he did voluntarily. ailments suffered by the insured. However, the
In each of the said applications the following question question raised for our determination is whether the
was asked: "1. What physician or practitioner or any two answers given by the insured in his applications
other person not named above have you consulted or are false, and if they were the cause, or one of the
been treated by, and for what illness, or ailment? (If causes, which induced the defendant to issue the
none, so state.)" In the first application, the insured policies. On the first point, the facts above set out
answered "None", and in the second, "No". These leave no room for doubt. The insured knew that he
answers of the insured as well as his other statements had suffered from a number of ailments, including
contained in his applications were one of the causes incipient pulmonary tuberculosis, before subscribing
or considerations for the issuance of the policies, and the applications, yet he concealed them and omitted
they so positively appear therein. After the death of the hospital where he was confined as well as the
the insured and as a result of the demand made by name of the lady physician who treated him. That this
the beneficiaries upon the defendant to pay the value concealment and the false statements constituted
of the policies, the latter discovered that the fraud, is likewise clear, because the defendant by
aforementioned answers were false and fraudulent, reason thereof accepted the risk which it would
because the truth was that the insured, before otherwise have flatly refused. When not otherwise
answering and signing the applications and before the specially provided for by the Insurance Law, the
issuance of the policies, had been treated in the contract of life insurance is governed by the general
General Hospital by a lady physician for different rules of the civil law regarding contracts. Article 1261
ailments. It indisputably appears that between May 13 of the Civil Code provides that there is no contract
and 19, 1929, the insured had entered the General unless there should be, in addition to consent and a
Hospital in Manila, and was treated by Doctor Pilar V. definite object, a consideration for the obligation
Cruz for peptic ulcer and chronic catarrhal established. And article 1276 provides that the
nasopharyngitis; on August 5, 1930, he entered the statement of a false consideration shall render the
same hospital and was treated by the same physician contract void. The two answers being one of the
for chronic pyelocystitis and for incipient pulmonary considerations of the policies, and it appearing that
tuberculosis; on the 13th of the same month he they are false and fraudulent, it is evident that the
returned to the hospital and was treated by the same insurance contracts were null and void and did not
physician for chronic suppurative pyelocystitis and for give rise to any right to recover their value or amount.
chronic bronchitis; on the 20th of the same month he A similar case was already decided by this court in
again entered the hospital and was treated by the Argente vs. West Coast Life Insurance Co. (51 Phil.,
same doctor for acute tracheo-bronchitis and chronic 725). In that case the insured concealed from the
suppurative pyelocystitis; on the 27th of the same physician who examined her that she had consulted
month he again entered the same hospital and was and had been treated by another physician for
treated for the same ailments; on December 11, 1930, cerebral congestion and Bell's Palsy, and that she was
he again entered the hospital and was treated for the addicted to alcohol, so much so that on one occasion
same ailments; on the 18th of the same month, he she was confined in the San Lazaro Hospital suffering
again entered the hospital and was treated for the from "alcoholism"; this court held that such
same ailments; on the 28th of the same month he concealments and false and fraudulent statements
again entered the hospital and was treated for the rendered the policy null and void. In discussing the
same ailments, and, finally, on January 11, 1931, he legal phase of the case, this court said:
again entered the hospital and was treated by the
same doctor for the same ailments. One ground for the rescission of a contract
of insurance under the Insurance Act is a
The defendant contended at the outset that the two "concealment", which in section 25 is defined
policies did not create any valid obligation because as "A neglect to communicate that which a
they were fraudulently obtained by the insured. The party knows and ought to communicate".
appealed decision holds that the health of the insured Appellant argues that the alleged
before the acceptance of his applications and the concealment was immaterial and insufficient
issuance of the policies could neither be discussed nor to avoid the policy. We cannot agree. In an
questioned by the defendant, because the insured action on a life insurance policy where the
was examined by three physicians of the company and evidence conclusively shows that the
all of them unanimously certified that he was in good answers to questions concerning diseases
health and that he could be properly insured. The were untrue, the truth or falsity of the
answers become the determining factor. If true and act upon them to his prejudice. So
the policy was procured by fraudulent it is decided that under a stipulation voiding
representations, the contract of insurance the policy for concealment or
apparently set forth therein was never misrepresentation of any material fact or if
legally existent. It can fairly be assumed that his interest is not truly stated or is other than
had the true facts been disclosed by the the sole and unconditional ownership the
assured, the insurance would never have facts are unimportant that insured did not
been granted. intend to deceive or withhold information as
to encumbrances even though no questions
In Joyce, The Law of Insurance, second were asked. And if insured while being
edition, volume 3, Chapter LV, is found the examined for life insurance, and knowing
following: that she had heart disease, falsely stated
that she was in good health, and though she
could not read the application, it was
"Concealment exists where the assured has explained to her and the questions asked
knowledge of a fact material to the risk, and through an interpreter, and the application
honesty, good faith and fair dealing requires like the policy contained a provision that no
that he should communicate it to the liability should be incurred unless the policy
assured, but he designedly and intentionally was delivered while the insured was in good
withholds the same. health, the court properly directed a verdict
for the insurer, though a witness who was
"Another rule is that if the assured present at the examination testified that the
undertakes to state all the circumstances insured was not asked whether she had heart
affecting the risk, a full and fair statement disease.
of all is required.
xxx xxx xxx
"It is also held that the concealment must, in
the absence of inquiries, be not only "The basis of the rule vitiating the contract
material, but fraudulent, or the fact must in cases of concealment is that it misleads or
have been intentionally withheld; so it is deceives the insurer into accepting the risk,
held under English law that if no inquiries are or accepting it at the rate of premium agreed
made and no fraud or design to conceal upon. The insurer, relying upon the belief
enters into the concealment the contract is that the assured will disclose every material
not avoided. And it is determined that even fact within his actual or presumed
though silence may constitute knowledge, is misled into a belief that the
misrepresentation or concealment it is not of circumstance withheld does not exist, and he
itself necessarily so as it is a question of fact. is thereby induced to estimate the risk upon
Nor is there a concealment justifying a a false basis that it does not exist. The
forfeiture where the fact of insanity is not principal question, therefore, must be, Was
disclosed no questions being asked the assurer misled or deceived into entering
concerning the same. . . . a contract obligation or in fixing the
premium of insurance by a withholding of
"But it would seem that if a material fact is material information or facts within the
actually known to the assured, its assured's knowledge or presumed
concealment must of itself necessarily be a knowledge?
fraud, and if the fact is one which the
assured ought to know, or is presumed to "It therefore follows that the assurer in
know, the presumption of knowledge ought assuming a risk is entitled to know every
to place the assured in the same position as material fact of which the assured has
in the former case with relation to material exclusive or peculiar knowledge, as well as
facts; and if the jury in such cases find the all material facts which directly tend to
fact material, and one tending to increase increase the hazard or risk which are known
the risk, it is difficult to see how the by the assured, or which ought to be or are
inference of a fraudulent intent or presumed to be known by him. And a
intentional concealment can be avoided. concealment of such facts vitiates the
And it is declared that if a material fact is policy. "It does not seem to be necessary ...
concealed by assured it is equivalent to a that the ... suppression of the truth should
false representation that it does not exist have been willful." If it were but an
and that the essentials are the truth of the inadvertent omission, yet if it were material
representations whether they were intended to the risk and such as the plaintiff should
to mislead and did insurer accept them as
have known to be so, it would render the a reasonable belief that the applicant had
policy void. But it is held that if untrue or that fatal disease, and we should further
false answers are given in response to construe this case as establishing the rule
inquiries and they relate to material facts that such a matter cannot rest alone upon
the policy is avoided without regard to the the assured's belief irrespective of what is a
knowledge or fraud of assured, although reasonable belief, but that it ought to be
under the statute statements are judged by the criterion whether the belief is
representations which must be fraudulent to one fairly warranted by the circumstances. A
avoid the policy. So under certain codes the case in Indiana, however, holds that if the
important inquiries are whether the assured has some affection or ailment of one
concealment was willful and related to a or more of the organs inquired about so well-
matter material to the risk. defined and marked as to materially derange
for a time the functions of such organ, as in
xxx xxx xxx the case of Bright's disease, the policy will
be avoided by a nondisclosure, irrespective
of the fact whether the assured knew of such
"If the assured has exclusive knowledge of ailment or not. . . ."
material facts, he should fully and fairly
disclose the same, whether he believes them
material or not. But notwithstanding this In view of the foregoing, we are of the opinion that
general rule it will not infrequently happen, the appellant's first two assignments of error are well
especially in life risks, that the assured may founded, wherefore, the appealed judgment is
have a knowledge actual or presumed of reversed and the defendant absolved from the
material facts, and yet entertain an honest complaint, with the costs of both instances to the
belief that they are not material. ... The plaintiffs. So ordered.
determination of the point whether there
has or has not been a material concealment Republic of the Philippines
must rest largely in all cases upon the form SUPREME COURT
of the questions propounded and the exact Manila
terms of the contract. Thus, where in
addition to specifically named diseases the FIRST DIVISION
insured was asked whether he had had any
sickness within ten years, to which he
answered "No", and it was proven that within G.R. No. L-34200 September 30, 1982
that period he had had a slight attack of
pharyngitis, it was held a question properly REGINA L. EDILLON, as assisted by her husband,
for the jury whether such an inflammation of MARCIAL EDILLON, petitioners-appellants,
the throat was a "sickness" within the intent vs.
of the inquiry, and the court remarked on MANILA BANKERS LIFE INSURANCE CORPORATION
the appealed decision that if it could be held and the COURT OF FIRST INSTANCE OF RIZAL,
as a matter of law that the policy was BRANCH V, QUEZON CITY, respondents-appellees.
thereby avoided, then it was a mere devise
on the part of insurance companies to obtain K.V. Faylona for petitioners-appellants.
money without rendering themselves liable
under the policy. . . .
L. L. Reyes for respondents-appellees.
". . . The question should be left to the jury
whether the assured truly represented the
state of his health so as not to mislead or
deceive the insurer; and if he did not deal in VASQUEZ, J.:
good faith with the insurer in that matter,
then the inquiry should be made, Did he The question of law raised in this case that justified a
know the state of his health so as to be able direct appeal from a decision of the Court of First
to furnish a proper answer to such questions Instance Rizal, Branch V, Quezon City, to be taken
as are propounded. A Massachusetts case, if directly to the Supreme Court is whether or not the
construed as it is frequently cited, would be acceptance by the private respondent insurance
opposed to the above conclusion; but, on the corporation of the premium and the issuance of the
contrary, it sustains it, for the reason that corresponding certificate of insurance should be
symptoms of consumption had so far deemed a waiver of the exclusionary condition of
developed themselves within a few months overage stated in the said certificate of insurance.
prior to effecting the insurance as to induce
The material facts are not in dispute. Sometime in trial court that the ruling calling for a liberal
April 1969, Carmen O, Lapuz applied with respondent interpretation of an insurance contract in favor of the
insurance corporation for insurance coverage against insured and strictly against the insurer may not be
accident and injuries. She filled up the blank applied in the present case in view of the peculiar
application form given to her and filed the same with facts and circumstances obtaining therein.
the respondent insurance corporation. In the said
application form which was dated April 15, 1969, she We REVERSE the judgment of the trial court. The age
gave the date of her birth as July 11, 1904. On the of the insured Carmen 0. Lapuz was not concealed to
same date, she paid the sum of P20.00 representing the insurance company. Her application for insurance
the premium for which she was issued the coverage which was on a printed form furnished by
corresponding receipt signed by an authorized agent private respondent and which contained very few
of the respondent insurance corporation. (Rollo, p. items of information clearly indicated her age of the
27.) Upon the filing of said application and the time of filing the same to be almost 65 years of age.
payment of the premium on the policy applied for, Despite such information which could hardly be
the respondent insurance corporation issued to overlooked in the application form, considering its
Carmen O. Lapuz its Certificate of Insurance No. prominence thereon and its materiality to the
128866. (Rollo, p. 28.) The policy was to be effective coverage applied for, the respondent insurance
for a period of 90 days. corporation received her payment of premium and
issued the corresponding certificate of insurance
On May 31, 1969 or during the effectivity of without question. The accident which resulted in the
Certificate of Insurance No. 12886, Carmen O. Lapuz death of the insured, a risk covered by the policy,
died in a vehicular accident in the North Diversion occurred on May 31, 1969 or FORTY-FIVE (45) DAYS
Road. after the insurance coverage was applied for. There
was sufficient time for the private respondent to
On June 7, 1969, petitioner Regina L. Edillon, a sister process the application and to notice that the
of the insured and who was the named beneficiary in applicant was over 60 years of age and thereby cancel
the policy, filed her claim for the proceeds of the the policy on that ground if it was minded to do so. If
insurance, submitting all the necessary papers and the private respondent failed to act, it is either
other requisites with the private respondent. Her because it was willing to waive such disqualification;
claim having been denied, Regina L. Edillon instituted or, through the negligence or incompetence of its
this action in the Court of First Instance of Rizal on employees for which it has only itself to blame, it
August 27, 1969. simply overlooked such fact. Under the
circumstances, the insurance corporation is already
deemed in estoppel. It inaction to revoke the policy
In resisting the claim of the petitioner, the despite a departure from the exclusionary condition
respondent insurance corporation relies on a provision contained in the said policy constituted a waiver of
contained in the Certificate of Insurance, excluding such condition, as was held in the case of "Que Chee
its liability to pay claims under the policy in behalf of Gan vs. Law Union Insurance Co., Ltd.,", 98 Phil. 85.
"persons who are under the age of sixteen (16) years This case involved a claim on an insurance policy
of age or over the age of sixty (60) years ..." It is which contained a provision as to the installation of
pointed out that the insured being over sixty (60) fire hydrants the number of which depended on the
years of age when she applied for the insurance height of the external wan perimeter of the bodega
coverage, the policy was null and void, and no risk on that was insured. When it was determined that the
the part of the respondent insurance corporation had bodega should have eleven (11) fire hydrants in the
arisen therefrom. compound as required by the terms of the policy,
instead of only two (2) that it had, the claim under
The trial court sustained the contention of the private the policy was resisted on that ground. In ruling that
respondent and dismissed the complaint; ordered the the said deviation from the terms of the policy did not
petitioner to pay attorney's fees in the sum of ONE prevent the claim under the same, this Court stated
THOUSAND (P1,000.00) PESOS in favor of the private the following:
respondent; and ordered the private respondent to
return the sum of TWENTY (P20.00) PESOS received We are in agreement with the trial
by way of premium on the insurancy policy. It was Court that the appellant is barred
reasoned out that a policy of insurance being a by waiver (or rather estoppel) to
contract of adhesion, it was the duty of the insured claim violation of the so-called fire
to know the terms of the contract he or she is entering hydrants warranty, for the reason
into; the insured in this case, upon learning from its that knowing fully an that the
terms that she could not have been qualified under number of hydrants demanded
the conditions stated in said contract, what she therein never existed from the very
should have done is simply to ask for a refund of the beginning, the appellant
premium that she paid. It was further argued by the nevertheless issued the policies in
question subject to such warranty, contract
and received the corresponding inconsistent with
premiums. It would be perilously the known facts,
close to conniving at fraud upon the and the insurer is
insured to allow appellant to claim stopped
now as void ab initio the policies thereafter from
that it had issued to the plaintiff asserting the
without warning of their fatal breach of such
defect, of which it was informed, conditions. The
and after it had misled the law is charitable
defendant into believing that the enough to
policies were effective. assume, in the
absence of any
The insurance company was aware, showing to the
even before the policies were contrary, that an
issued, that in the premises insured insurance
there were only two fire hydrants company intends
installed by Que Chee Gan and two to execute a valid
others nearby, owned by the contract in return
municipality of Tabaco, contrary to for the premium
the requirements of the warranty in received; and
question. Such fact appears from when the policy
positive testimony for the insured contains a
that appellant's agents inspected condition which
the premises; and the simple renders it
denials of appellant's voidable at its
representative (Jamiczon) can not inception, and
overcome that proof. That such this result is
inspection was made it moreover known to the
rendered probable by its being a insurer, it will be
prerequisite for the fixing of the presumed to have
discount on the premium to which intended to waive
the insured was entitled, since the the conditions
discount depended on the number and to execute a
of hydrants, and the fire fighting binding contract,
equipment available (See"'Scale of rather than to
Allowances" to which the policies have deceived
were expressly made subject). The the insured into
law, supported by a long line of thinking he is
cases, is expressed by American insured when in
Jurisprudence (Vol. 29, pp. 611- fact he is not, and
612) to be as follows: to have taken is
money without
consideration.'
It is usually held (29 Am. Jur.,
that where the Insurance,
insurer, at the section 807, at
time of the pp. 611-612.)
issuance of a
policy of
insurance, has The reason for
knowledge of the rule is not
existing facts difficult to find.
which, if insisted
on, would The plain, human
invalidate the justice of this
contract from its doctrine is
very inception, perfectly
such knowledge apparent. To
constitutes a allow a company
waiver of to accept one's
conditions in the money for a
policy of Significantly, in the case before Us
insurance which the Capital Insurance accepted the
it then knows to promise of Plastic Era to pay the
be void and of no insurance premium within thirty
effect, though it (30) days from the effective date of
knows as it must, policy. By so doing, it has impliedly
that the assured agreed to modify the tenor of the
believes it to be insurance policy and in effect,
valid and binding, waived the provision therein that it
is so contrary to would only pay for the loss or
the dictates of damage in case the same occurs
honesty and fair after the payment of the premium.
dealing, and so Considering that the insurance
closely related to policy is silent as to the mode of
positive fraud, as payment, Capital Insurance is
to be abhorent to deemed to have accepted the
fairminded men. promissory note in payment of the
It would be to premium. This rendered the policy
allow the immediately operative on the date
company to treat it was delivered. The view taken in
the policy as valid most cases in the United States:
long enough to
get the premium ... is that
on it, and leave it although one of
at liberty to conditions of an
repudiate it the insurance policy
next moment. is that "it shall
This cannot be not be valid or
deemed to be the binding until the
real intention of first premium is
the parties. To paid", if it is
hold that a literal silent as to the
construction of mode of
the policy payment,
expressed the promissory notes
true intention of received by the
the company company must be
would be to indict deemed to have
it, for fraudulent been accepted in
purposes and payment of the
designs which we premium. In
cannot believe it other words, a
to be guilty of requirement for
(Wilson vs. the payment of
Commercial the first or initial
Union Assurance premium in
Co., 96 Atl. 540, advance or actual
543544). cash may be
waived by
A similar view was upheld in the case of Capital acceptance of a
Insurance & Surety Co., Inc. vs. Plastic Era Co., Inc., promissory
65 SCRA 134, which involved a violation of the note...
provision of the policy requiring the payment of
premiums before the insurance shall become WHEREFORE, the judgment appealed from is hereby
effective. The company issued the policy upon the REVERSED and SET ASIDE. In lieu thereof, the private
execution of a promissory note for the payment of the respondent insurance corporation is hereby ordered
premium. A check given subsequent by the insured as to pay to the petitioner the sum of TEN THOUSAND
partial payment of the premium was dishonored for (P10,000.00) PESOS as proceeds of Insurance
lack of funds. Despite such deviation from the terms Certificate No. 128866 with interest at the legal rate
of the policy, the insurer was held liable. from May 31, 1969 until fully paid, the further sum of
TWO THOUSAND (P2,000.00) PESOS as and for After the plaintiff had presented his
attorney's fees, and the costs of suit. evidence, the defendant companies in
cases Nos. 33458, 33868, and 33480,
SO ORDERED. offered to compromise with him by paying
eighty-five per cent of his claim against
them. In view of the fact that said
Republic of the Philippines
defendants had in their answer raised the
SUPREME COURT
question of warranties A and G of the
Manila
plaintiff's policies, providing that the
building used for the effects insured
EN BANC would not be occupied by any other
lessee, nor would be used for the deposit
G.R. No. L-33131 December 13, 1930 of other goods, without the consent of
said defendants, and inasmuch as the
latter alleged in their answer that the
EMILIO GONZALES LA O, plaintiff-appellee,
owner of the burnt building had leased
vs.
the warehouse to several persons for the
THE YEK TONG LIN FIRE AND MARINE INSURANCE
storage of sundry articles, the plaintiff
CO., LTD., defendant-appellant.
had to accept the proposed compromise,
and in consequence thereof, the three
Araneta and Zaragosa for appellant. cases aforesaid were dismissed.
Feria and La O for appellee.
The present case followed the usual
course of procedure because the plaintiffs
refused to accept the compromise which,
VILLAMOR, J.: in the same terms as those made by the
defendants in the three cases mentioned,
was proposed to him by the defendant the
This is an action to recover of the defendant the Yek
Yek Tong Lin Fire & Marine Insurance
Tong Lin Fire & Marine Insurance Co., Ltd., the
Company, the plaintiff contending that
amount of two insurance policies totaling P100,000
said defendant did not, nor could, raise
upon leaf tobacco belonging to the plaintiff, which
the question of warranties A and G
was damaged by the fire that destroyed the building
heretofore mentioned for the simple
on Soler Street No. 188, where said tobacco was
reason that it was the defendant itself, as
stored, on January 11, 1928.
owner, who had leased the building which
later was destroyed by fire, to another
The defendant filed a general and specific denial of person after having already ceded a
each and every allegation of the complaint, set up portion of it to said plaintiff.
three special defenses, and prayed to be absolved
from the complaint with costs against the plaintiff.
The only question to be determined,
having been raised in the defendant's
After the case was tried, the court below rendered answer both parties agreeing that the
judgment as follows: plaintiff insured his leaf tobacco with the
defendant assurance company, and that
In this case and in Nos. 334568, and 33480 said goods were damaged by the fire
of this court, which, by agreement of the which destroyed the warehouse where
interested parties, were jointly tried, the they were stored, on January 11, 1928
plaintiff demands P290,000 from the is whether said goods were worth what
defendant assurance companies, alleging the plaintiff claims, that is, about equal
that to be the amount of the insurance on to the amount for which they were
his leaf tobacco which was damaged by insured in the four above mentioned
the fire that destroyed the warehouse at assurance companies, including the
No. 188 Soler Street, Manila, where it was defendant in this case.
stored, on January 11, 1928, the
plaintiff's claim against the herein The plaintiff has conclusively shown by
defendant, the Yek Tong Lin Fire & Marine the Official Register Book (Exhibit 1) and
Insurance Co. being for P100,000, and the Official Guide (Exhibit J), furnished by
against the defendants in the three other the Bureau of Internal Revenue, and kept
cases mentioned above, for P190,000. under the supervision thereof in the usual
form, in accordance with articles 10, 34
to 38 of the Regulations of the same presented witness Rowlands, whose
promulgated under No. 17, by the testimony or opinion as to the probable
Secretary of Finance; the Stock Book for number of bales of tobacco in the
recording the quantity of tobacco, Exhibit warehouse at the date of the fire does not
K, kept by the plaintiff and presented as deserve serious consideration, not only
part of the testimony of witnesses because of the plaintiff's evidence, but
Claveria, Bonete, and Leoncio Jose; the because his opinion or estimate is based
testimony of Estanislao Lopez, Inspector solely upon photographs of the place
of Internal Revenue, and the latter's taken after the fire.
report (Exhibit N), submitted to the
Collector of Internal Revenue in In view of the foregoing, the court hereby
pursuance of article 33 of the sentences the defendant the Yek Tong Lin
aforementioned Regulations; the tobacco Fire and Marine Insurance Company, Ltd.,
invoices of stock damaged by the fire, to pay the plaintiff Emilio Gonzales La O,
Exhibits L and L-1 to L-20; and by the the amount of one hundred thousand
testimony of Clemente Uson who went pesos (P100,000), for which it had
over the plaintiff's books as auditor and accepted the insurance on the leaf
public accountant, and also prepared tobacco belonging to said plaintiff,
Exhibits T and U, attached to the record, damaged by the fire which destroyed the
that the plaintiff had in the warehouse at warehouse at No. 188 Soler Street, where
No. 188 Soler at the time of the fire, not it was stored, on January 11, 1928, and
less, but rather more, than 6,200 bales of legal interest upon said amount from June
leaf tobacco worth over P300,000, which 27, 1928, when the complaint was filed in
is of course more than the sum total of all this case, plus the costs.
the insurances taken out with the
defendant herein and the defendants in
the three aforementioned cases Nos. So ordered.
33458, 33868, and 33480.lawphi1>net
Manila, P. I., this 24th day of December,
The reason why the entry showing that 1929.
258 bales of tobacco had been removed
from the warehouse, appearing in the ANACLETO DIAZ
Official Register Book, Exhibit I, was not Judge.
posted in the Stock Book, Exhibit K, has
been satisfactorily explained by the
plaintiff's witnesses, who stated that it The defendant duly appealed from this judgment,
was due to the fact that there was no time alleging that the trial court erred in making reference
to post it in the Stock Book, because the to the settlement arrived at by the plaintiff and other
insurance companies, and in declaring that the only
fire took place and the plaintiff told them
not to touch, and to make no further question involved in the case is whether or not the
entries in the books. Witness White, the tobacco damaged by the fire is worth at least
P290,000.
defendant company's adjuster, who
carefully examined then plaintiff's books
not only immediately after the fire, but There is no merit in these assignments of error. Since
also during the hearing of this case, seems the settlement between the plaintiff and the other
not to have found any irregularity therein; defendant companies was reached after the plaintiff
at least he said nothing on the point when had presented his evidence, and as those three cases
he took the witness stand. On the were tried jointly with the instant case, there is no
contrary, in his report Exhibit UU sent to valid reason why the trial court should not refer to it
the defendant herein in his capacity as in deciding this case. Furthermore, the court's holding
adjuster, appointed by the latter, and in here assigned as error, granting there were other
Exhibits WW and XX, admitted by the Yek incidental matters to be decided by the court, does
Tong Lin Ins. Co., Ltd., he admitted that not in itself constitute a reversible error.
the leaf tobacco belonging to the plaintiff
in the warehouse when the fire took place In the third assignment of error, the defendant
exceeded, in quantity and value, the contends that the plaintiff cannot recover under the
amount of the insurance. policy as he has failed to prove that the Bank of the
Philippine Islands, to whom the policy was made
The defendant did not present evidence payable, no longer has any rights and interests in it.
to rebut the plaintiff's evidence, but only It should be noted that the defendant did not in its
answer allege defect of parties plaintiff, and,
besides, it does not appear that the plaintiff ceded to attached to this policy." And attached to said policies
the bank all his rights or interests in the insurance, issued by the defendant there is a sheet of "Other
the note attached to the policies merely stating: insurances" with the amount and the assurance
"There shall be paid to the Bank of the Philippine companies in blank, which, according to the appellee,
Islands an indemnity for any loss caused by fire, constitutes a notification that there were other
according to the interest appearing in its favor." And insurances existing at the time.
the fact that the plaintiff himself presented in
evidence the policies mortgaged to the Bank of the In the case of Benedict vs. Ocean Insurance Co. (31
Philippine Islands gives rise to the presumption that N.Y., 391-393), the construction of the clause,
the debt thus secured has been paid, in accordance "privilege for $4,500 additional insurance," was
with article 1191 of the Civil Code. discussed. One of the printed clauses of the policy
reads as follows:
Corpus Juris, volume 26, pages 483 et seq., states:
If said assured, or his assigns, shall hereafter
Insured, being the person with whom the make any other insurance upon the same
contract was made, is primarily the proper property, and shall not, with all reasonable
person to bring suit thereon. Subject to some diligence, give notice to this corporation,
exceptions, insured may thus sue, although and have the same indorsed on this
the policy is taken wholly or in part for the instrument, or otherwise acknowledged by
benefit of another person named or them, in writing, this policy shall cease and
unnamed, and although it is expressly made be of no further effect.
payable to another as his interest may
appear or otherwise. Although a policy The Supreme Court of New York held that the words
issued to a mortgagor is taken out for the "Privilege for $4,500 additional insurance" made it
benefit of the mortgagee and is made unnecessary for the assured to inform the insurer of
payable to him, yet the mortgagor may sue any other policy up to that amount.
thereon in his own name, especially where
the mortgagee's interest is less than the full
amount recoverable under the policy, . . . . In the case cited the same goods insured by the
defendant company were reinsured to the amount of
$4,500 in accordance with the clause "privilege for
And in volume 33, page 82, of the same work, we read $4,500 additional insurance;" but in the instant case
the following: it may be said that the tobacco insured in the other
companies was different from that insured with the
Insured may be regarded as the real party in defendant, since the number of bales of tobacco in
interest, although he has assigned as the warehouse greatly exceeded that insured with the
collateral security any judgment he may defendant and the other companies put together. And
obtain. according to the doctrine enunciated in 26 Corpus
Juris, 188, "to be insurance of the sort prohibited the
It is also contended that the trial court erred in not prior policy must have been insurance upon the same
declaring that in as much as the plaintiff failed to subject matter, and upon the same interest therein.
notify the defendant corporation in writing, of other
insurance policies obtained by him, he has violated Furthermore, the appellant cannot invoke the
article 3 of the conditions of the policies in question, violation of article 3 of the conditions of the insurance
thereby rendering these policies null and void. Article policies for the first time on appeal, having failed to
3 of the conditions of the policies in question do so in its answer; besides, as the appellee correctly
prescribes: contends in his brief, Guillermo Cu Unjieng, who was
then president and majority shareholder of the
ART. 3. Any insurance in force upon all or appellant company, the Yek Tong Lin Fire & Marine
part of the things insured must be declared Insurance Co., knew that there were other
in writing by the insured and he should cause insurances, at least from the attempt to raise the
the company to insert or mention it in the insurance premium on the warehouse and the
policy, and without such requisite said policy appellee's tobacco deposited therein to 1 per centum,
will be regarded as null and void, and the and it was later reduced upon petition of the
assured deprived of all rights of indemnity in appellant itself and other assurance companies to
case of loss. 0.75 per centum presented to the association of
assurance companies in the year 1927, and
notwithstanding this, said appellant did not rescind
The following clause has been inserted with a the insurance policies in question, but demanded and
typewriter in the policies: "Subject to clauses G and A collected from the appellee the increased premium.
and other insurances with a special short period
That the defendant had knowledge of the existence case of Glaser vs. Home Ins. Co. (47 Misc. Rep., 89;
of other policies obtained by the plaintiff from other 93 N. Y. Supp., 524; Abbott's Proof of Facts, 3d ed.,
insurance companies, is specifically shown by the p. 847), where it was declared that the cost of the
defendant's answer wherein it alleges, by way of goods destroyed by fire is some evidence of value, in
special defense, the fact that there exist other an action against the insurance company. Exhibits L
policies issued by the companies mentioned therein. to L-20, which are invoices for tobacco purchased by
If, with the knowledge of existence of other the appellee, and the testimony of the public
insurances which the defendant deemed violations of accountant Clemente Uson, who went over them and
the contract, it has preferred to continue the policy, the rest of the appellee's books after the fire, taken
its action amounts to a waiver of the annulment of in connection with reports T and Z, adduced as part
the contract, in accordance with the following of his testimony, show that the cost price of each bale
doctrine in 19 Cyc., 791, 792:. of tobacco belonging to the appellee, damaged by the
fire, was P51.8544, which, multiplied by 6,264, the
FAILURE TO ASSERT FORFEITURE IN number of bales, yields a total of over P320,000.
GENERAL. While the weight of authority is
that a policy conditioned to become void The adjusters of the appellant, White & Page, in
upon a breach of a warranty is void ipso facto ascertaining the market price of the plaintiff's
upon such a breach without formal tobacco deposited in the burnt warehouse, taking the
proceedings on the part of the insurer, yet it information furnished by the Tabacalera and by M.
is true that such conditions are inserted for Pujalte, S. en C., as a basis, thus conclude their
the benefit of the insurer and may be report: "We therefore are obliged to the conclusion
waived, and that the insurer may elect to that the value of the tobacco destroyed was not less
continue the policy despite the breach. If it than P290,000." And, indeed, said adjusters, in behalf
does the policy is revived and restored. Its of the appellant, appraised the appellee's tobacco
failure to assert a forfeiture therefore is at assured and damaged by the fire at P303,052.32,
least evidence tending to show a waiver collecting from the proceeds of the sale of the
thereof. Many authorities go further, tobacco saved from the fire P3,000, the appellants
however, and hold that the failure to assert share in proportion to the to the insurance of
a forfeiture after knowledge of a ground P100,000 belonging to it, and P190,000 belonging to
thereof will amount of itself to waiver. . . . the other assurance companies, and considered the
appellee himself as his own assurer in the amount of
The fifth and sixth assignments of error refer to the P13,052.32 which was the difference between the
quantity of tobacco in the Soler warehouse at the total value of the tobacco damaged and the total
time of the fire, which, according to the appellant, amount of the insurance, P290,000, for which reason
did not exceed 4,930 bales. As may be seen, these the appellee received P129.21, as his proportionate
assignments of error by the appellant involved purely share of the tobacco saved, as shown by Exhibits UU,
questions of fact, and it is for this court to decide WW, and XX.
whether the findings of the trial court are supported
by the evidence. The judgment appealed from sets Hence the last assignment of error is without merit.
forth clearly the evidence presented to the court in
order to determine the quantity of tobacco in the Wherefore, the judgment appealed from is in
warehouse at the time of the fire. We have studied accordance with law, and must be, as it is hereby,
the evidence aforesaid, are fully convinced that the affirmed, with costs against the appellant. So
court's findings are well supported by the same. ordered.
Inasmuch as it has not, in our opinion, been shown
that the trial judge overlooked any fact, which, if duly
considered would have change the result of the case, Republic of the Philippines
we do not feel justified in altering of modifying his SUPREME COURT
findings. Manila

Finally, the appellant contends that the trial court EN BANC


erred in arriving at the damages that plaintiff may
recover under the policies in question by the cost G.R. No. L-27541 November 21, 1927
price of the tobacco damaged by the fire, instead of
computing the same on the market price of the said TAN CHAY HENG, plaintiff-appellee,
tobacco at the time of the fire; and in declaring that vs.
the tobacco damaged was worth more than P300,000. THE WEST COAST LIFE INSURANCE COMPANY,
This error is not well taken, for it is clear that the cost defendant-appellant.
price is competent evidence tending to show the
value of the article in question. And it was so held the
Gibbs and McDonough and Roman Ozaeta for I
appellant.
Hilado and Hilado for appellee. That the insurance policy on the life of Tan
Ceang, upon which plaintiff's action is based,
STATEMENT was obtained by the plaintiff in
confabulation with one Go Chulian, of
Plaintiff alleges that he is of age and a resident of Bacolod, Negros Occidental; Francisco
Bacolod, Occidental Negros; that the defendant is a Sanchez of the same place; and Dr. V. S.
foreign insurance corporation duly organized by the Locsin, of La Carlota, Negros Occidental,
laws of the Philippines to engage in the insurance thru fraud and deceit perpetrated against
business, its main office of which is in the City of this defendant in the following manner, to
Manila; that in the month of April, 1925, on his wit:
application the defendant accepted and approved a
life insurance policy of for the sum of P10,000 in 1. That on or about the 22d day of February,
which the plaintiff was the sole beneficiary; that the 1925, in the municipality of Pulupandan,
policy was issued upon the payment by the said Tan Occidental Negros, the present plaintiff and
Ceang of the first year's premium amounting to P936; the said Go Chulian, Francisco Sanchez and
that in and by its terms, the defendant agreed to pay Dr., V. S. Locsin, conspiring and
the plaintiff as beneficiary the amount of the policy confederating together for the purpose of
upon the receipt of the proofs of the death of the defrauding and cheating the defendant in
insured while the policy was in force; that without any the sum of P10,000, caused one Tan Caeng
premium being due or unpaid, Tan Ceang died on May to sign an application for insurance with the
10, 1925; that in June, 1925, plaintiff submitted the defendant in the sum of P10,000, in which
proofs of the death of Tan Ceang with a claim for the application it was falsely represented to the
payment of the policy which the defendant refused to defendant that the said Tan Ceang was single
pay, for which he prays for a corresponding judgment, and was a merchant, and that the plaintiff
with legal interest from the date of the policy, and Tan Chai Heng, the beneficiary, was his
costs. nephew, whereas in truth and in fact and as
the plaintiff and his said coconspirators well
In February, 1926, the defendant filed an answer to knew, the said Tan Ceang was not single but
the complaint in which it made a general and specific was legally married to Marcelina Patalita
denial, and then announced its intention to file an with whom he had several children; and that
amended answer, alleging special defense, and on he was not a merchant but was a mere
August 31, 1926, it filed the following: employee of another Chinaman by the name
of Tan Quina from whom he received only a
meager salary, and that the present plaintiff
AMENDED ANSWER was not a nephew of the said Tan Ceang.

Comes now the defendant, by its 2. That on said date, February 22, 1925, the
undersigned attorneys, and with leave of said Tan Ceang was seriously ill, suffering
court amends its answer to plaintiff's from pulmonary tuberculosis of about three
complaint herein by making it reads as years' duration, which illness was incurable
follows: and was well known to the plaintiff and his
said coconspirators.
I
3. That on or about the same date, February
That it admits paragraph 1 of said complaint. 22, 1925, the said Dr. V. S. Locsin, in his
capacity as medical examiner for the
II defendant insurance company, pursuant to
the conspiracy above mentioned, prepared
and falsified the necessary medical
That it denies each and every other certificate, in which it was made to appear,
allegation contained in each and every other among other things, that the said Tan Ceang
paragraph of said complaint. had never used morphine, cocaine or any
other drug; that he was then in good health
SPECIAL DEFENSE and had never consulted any physician; that
he had never spit blood; and that there was
By way of special defense, defendant no sign of either present or past disease of
alleges: his lungs; whereas in truth and in fact, as the
plaintiff and his said coconspirators well
knew, the said Tan Ceang was addicted to the regulations of the defendant insurance
morphine, cocaine, and opium and had been company, of which regulations the said
convicted and imprisoned therefor, and was Francisco Sanchez as agent of the defendant
then, and for about three year prior thereto had knowledge, the plaintiff and his said
had been suffering from pulmonary coconspirators in order to secure the
tuberculosis. delivery to them of said temporary policy,
and in accordance with said regulations of
4. That on or about the same date, to wit, the defendant company, caused the said Tan
February 22, 1925, the plaintiff and his said Ceang on April 10, 1925 to sign the following
coconspirators, pursuant to the conspiracy document: lawphil.net
above mentioned, cause a confidential
report to the defendant insurance company WEST COAST LIFE INSURANCE COMPANY
to be signed by one V. Sy Yock Kian, who was SAN FRANCISCO, CALIFORNIA
an employee of Go Chulian, in which
confidential report, among other things, it HEALTH CERTIFICATE FOR RE-INSTATEMENT
was falsely represented to the defendant
insurance company that the said Tan Ceang
was worth about P40,000, had an annual I herewith request the West Coast
income of from eight to ten thousand pesos Life Insurance Company to re-
net, had the appearance of good health, and instate Policy No.
never had tuberculosis; that the plaintiff and ............................. issued by it
his said coconspirators well knew that said upon my life, the first unpaid
representations were false; and that they premium on which became due
were made for the purpose of deceiving the ..............................,
defendant and inducing it to accept the said 19................
application for insurance.
I certify and state that I am now in
5. That after the said application for good and sound health, that since
insurance, medical certificate and the date of my examination under
confidential report had been prepared and the application on which said policy
falsified, as aforesaid, the plaintiff and his was written, I have had no injury,
said coconspirators caused the same to be sickness, impairment of health or
forwarded to the defendant at its office in symptom thereof, and that since
Manila, the medical certificate thru the said said date I have neither consulted a
Dr. V. S. Locsin as medical examiner, and physician nor made any application
said application for insurance and for life insurance that has not been
confidential report thru the said Francisco granted in exact kind and amount
Sanchez in his capacity as one of the agents applied for, except:
of the defendant insurance company in the
Province of Occidental Negros; that the NADA
defendant, believing that the
representations made in said document were (State fully all exceptions to all
true, and relying thereon, provisionally above statements. If no exceptions
accepted the said application for insurance insert "NONE.")
on the life of Tan Ceang in the sum of
P10,000 and issued a temporary policy
pending the final approval or disapproval of I agree that, if said policy re-
said application by defendant's home-office instated, it shall be only on
in San Francisco, California, where in case of condition of the truth of the above
approval a permanent policy was to be statements and such re-
issued; that such permanent policy was instatement shall not operate as a
never delivered to the plaintiff because waiver on the part of said Company
defendant discovered the fraud before its of its right to refuse to accept any
delivery. future overdue premiums or
installments thereof.
6. That the first agreed annual premium on
the insurance in question of P936.50 not Witness: (Sgd.) TAN CHAI HENG
having been paid within sixty (60) days after TAN CAENG
the date of the supposed medical Signature of
examination of the applicant as required by Applicant.
"Dated at Palupandan on this 10 day of April, 1925." case No. 28680), in which the present
that the statements and representations contained in defendant is the plaintiff, for the recovery
the application for reinstatement above set forth with of the amounts of two insurance policies
regard to the health and physical condition of the said aggregating P19,000, fraudulently obtained
Tan Ceang were false and known to the plaintiff and by the said Go Chulian and Sanchez upon the
his said coconspirators to be false; that the said lives of one Tan Deco, who was also suffering
temporary policy was delivered by defendant to the from and died of tuberculosis, and one Tan
insured on April 10, 1925, in the belief that said Anso, who was suffering from and died of
statements and representations were true and in beriberi.
reliance thereon.
III
7. That on May 10, 1925, that is to say, two months
and a half after the supposed medical examination That by reason of all the facts above set
above referred to, and exactly one month after the forth, the temporary policy of insurance on
date of the health certificate for reinstatement above the life of Tan Caeng for the sum of P10,000
set forth, the said Tan Ceang died in Valladolid, upon which the present action is base is null
Occidental Negros, of pulmonary tuberculosis, the and void.
same illnes from which suffering at the time it is
supposed he was examined by Dr. V. S. Locsin, but
that the plaintiff and his said coconspirators, pursuant Wherefore, defendant prays that it be
to their conspiracy, caused the said Dr. V. S. Locsin to absolved from plaintiff's complaint, with
state falsely in the certificate of death that the said costs against the plaintiff.
Tan Ceang had died of cerebral hemorrhage.
To this special defense, the plaintiff, claiming that it
II was a cross-complaint, filed a general demurrer upon
the ground that it does not state facts sufficient to
constitute a cause of defense.
That the plaintiff Tan Chai Heng, on the
dates herein-above mentioned, was, liked V.
Sy Yock Kian who signed the confidential After exhaustive arguments and on September 16,
report above mentioned, an employee of the 1926, the court rendered the following decision:
said Go Chulian; that the latter was the
ringleader of a gang of malefactors, who, After considering the demurrer filed by the
during, and for some years previous to the plaintiff to the special defense contained in
dates above mentioned, were engaged in the the amended answer of the defendant,
illicit enterprise of procuring fraudulent life dated August 31, 1926, without prejudice to
insurances from the present defendant, writing a more extensive decision, said
similar to the one in question, and which demurrer is sustained, and the defendant is
enterprise was capitalized by him by given a period of five days within which to
furnishing the funds with which to pay the amend its aforesaid answer.
premium on said fraudulent insurance; that
the said Go Chulian was the one who So ordered.
furnished the money with which to pay the
first and only annual premium on the
insurance here in question, amounting to To which the defendant duly excepted.
P936.50; that the said Go Chulian, on August
28, 1926, was convicted by the Court of First As a result of the trial the general issues, the lower
Instance of the City of Manila, in criminal court rendered judgment for the plaintiff for P10,000,
case No. 31425 of that court, of the crime of with legal interest from January 4, 1926, and costs,
falsification of private documents in to which the defendant duly excepted and filed a
connection with an fraudulent insurance, motion for a new trial, which was overruled. On
similar to the present, committed against appeal the defendant assigns the following errors:
this defendant in the month of September,
1924; that in the same case the said The trial court erred
Francisco Sanchez was one of the coaccused
of the said Go Chulian but was discharged
from the complaint, because he offered 1. In sustaining plaintiff's demurrer to the
himself and was utilized as a state's witness; special defense contained in defendant's
that there is another civil action now amended answer.
pending against Go Chulian and Sanchez in
the Court of First Instance of Manila (civil
2. In holding, in effect, that an insurer contends that it never did enter into any contract of
cannot avoid a policy which had been insurance on the life of Tan Caeng.
procured by fraud unless he brings an action
to rescind it before he is sued thereon. The word "rescind" has a well defined legal meaning,
and as applied to contracts, it presupposes the
3. In rejecting all proofs offered by the existence of a contract to rescind.
defendant during the trial for the purpose of
defeating plaintiff's fraudulent claim. Word & Phrases, volume 7, page 6139, says:

4. In not absolving the defendant from To rescind is to abrogate, annual, avoid, or


plaintiff's complaint. cancel a contract.

The word "rescind," as used in a statement


by a party to a contrary as follows. "I hereby
JOHNS, J.: terminate and rescind my said written
contract," is synonymous with the word
It will thus be noted that the premium was paid on `terminate,' and the rescission therefore
April 10, 1925, at which time the temporary policy relates only to the unfulfilled part, and not
was issued; that the plaintiff's action was commenced to the entire agreement, making the party
on January 4, 1926; that the original answer of the rescinding liable on notes executed pursuant
defendant, consisting of a general and specific denial, to the contract which matured before the
was filed on February 27, 1926; and that its amended rescission.
answer was filed on August 31, 1926.
The rescission is the unmaking of a contract,
Based upon those facts the plaintiff vigorously requiring the same concurrence of wills as
contended in the lower court and now contends in the that which made it, and nothing short of this
court, that section 47 of the Insurance Act should be will suffice. There is a wide difference
applied, and that when so applied, defendant is between the rescission of a contract and its
barred and estopped to plead and set forth the mere termination or cancellation.
matters alleged in its special defense. That section is
as follows: After a contract has been broken, whether
by an inability to perform it, or by rescinding
Whenever a right to rescind a contract of against right or otherwise, the party not in
insurance is given to the insurer by any fault may sue the other for the damages
provision of this chapter, such right must be suffered, or, if the parties can be placed in
exercised previous to the commencement of status quo, he may, should he prefer, return
an action on the contract. what he has received and recover in a suit
value of what he has paid or done. The latter
remedy is termed "rescission."
The defendant contended in the lower court and now
contends in this court, that section 47 does not apply
to the new matters alleged in the special defense. If In the instant case, it will be noted that even in its
in legal effect defendant's special defense is in the prayer, the defendant does not seek to have the
nature of an act to rescind "a contract of insurance," alleged insurance contract rescinded. It denies that it
then such right must be exercised prior to an action ever made any contract of insurance on the life of Tan
enforce the contract. That is the real question Ceang or that any such a contract ever existed, and
involved in this appeal. that is the question which it seeks to have litigated by
its special defense. In the very nature of things, if the
defendant never made or entered into the contract in
Defendant's original answer was a general and specific question, there is no contract to rescind, and, hence,
denial. In other words, it specifically denied that if section 47 upon which the lower based its decision in
ever issued the policy in question, or that it ever sustaining the demurrer does not apply. As stated, an
agreed with Tan Ceang in the even of his death to pay action to rescind a contract is founded upon and
P10,000 to the plaintiff or any one else. In its presupposes the existence of the contract which is
amended answer the defendant again makes a general sought to be rescinded. If all of the material matters
and specific denial, and alleges the reasons, the set forth and alleged in the defendant's special plea
specific facts, and the reasons why it never made or are true, there was no valid contract of insurance, for
entered into the contract alleged in the complaint, the simple reason that the minds of the parties never
and based upon those alleged facts, defendant met and never agreed upon the terms and conditions
of the contract. We are clearly of the opinion that, if
such matters are known to exist by a preponderance (b) Under the second cause of action, the
of the evidence, they would constitute a valid defense sum of P150,000;
to plaintiff's cause of action. Upon the question as to
whether or not they or are not true, we do not at this (c) Under the third cause of action, the sum
time have or express any opinion, but we are clear of P5,000;
that section 47 does not apply to the allegations made
in the answer, and that the trial court erred in
sustaining the demurrer. (d) Under the fourth cause of action, the sum
of P15,000; and
The judgment of the lower court is reversed and the
case is remanded for such other and further (e) Under the fifth cause of action, the sum
proceedings as are not inconsistent with this opinion, of P40,000;
with costs against the plaintiff. So ordered.
all of which shall bear interest at the rate of 8% per
Republic of the Philippines annum in accordance with Section 91 (b) of the
SUPREME COURT Insurance Act from September 26, 1940, until each is
Manila paid, with costs against the defendant.

EN BANC The complaint in intervention of the Philippine


National Bank is dismissed without costs. (Record on
Appeal, 166-167.)
G.R. No. L-4611 December 17, 1955
From the decision, the defendant Insurance Company
QUA CHEE GAN, plaintiff-appellee, appealed directly to this Court.
vs.
LAW UNION AND ROCK INSURANCE CO., LTD.,
represented by its agent, WARNER, BARNES AND The record shows that before the last war, plaintiff-
CO., LTD., defendant-appellant. appellee owned four warehouses or bodegas
(designated as Bodegas Nos. 1 to 4) in the municipality
of Tabaco, Albay, used for the storage of stocks of
Delgado, Flores & Macapagal for appellant. copra and of hemp, baled and loose, in which the
Andres Aguilar, Zacarias Gutierrez Lora, Gregorio appellee dealth extensively. They had been, with
Sabater and Perkins, Ponce Enrile & Contreras for their contents, insured with the defendant Company
appellee. since 1937, and the lose made payable to the
Philippine National Bank as mortgage of the hemp and
crops, to the extent of its interest. On June, 1940, the
insurance stood as follows:
REYES, J. B. L., J.:
Policy No. Property Insured
Qua Chee Gan, a merchant of Albay, instituted this
action in 1940, in the Court of First Instance of said 2637164 (Exhibit "LL") Bodega No. 1 (Building)
province, seeking to recover the proceeds of certain
fire insurance policies totalling P370,000, issued by Bodega No. 2 (Building)
the Law Union & Rock Insurance Co., Ltd., upon
certain bodegas and merchandise of the insured that Bodega No. 3 (Building)
were burned on June 21, 1940. The records of the 2637165 (Exhibit "JJ")
original case were destroyed during the liberation of Bodega No. 4 (Building)
the region, and were reconstituted in 1946. After a
Hemp Press moved by steam eng
trial that lasted several years, the Court of First
Instance rendered a decision in favor of the plaintiff, 2637345 (Exhibit "X") Merchandise contents (copra and e
the dispositive part whereof reads as follows:
2637346 (Exhibit "Y") Merchandise contents (hemp) of Bo
Wherefore, judgment is rendered for the
plaintiff and against the defendant 2637067 (Exhibit "GG") Merchandise contents (loose hemp)
condemning the latter to pay the former
Total
(a) Under the first cause of action, the sum
of P146,394.48;
Fire of undetermined origin that broke out in the early
morning of July 21, 1940, and lasted almost one week,
gutted and completely destroyed Bodegas Nos. 1, 2 places, the hydrants being supplied with
and 4, with the merchandise stored theren. Plaintiff- water pressure by a pumping engine, or from
appellee informed the insurer by telegram on the some other source, capable of discharging at
same date; and on the next day, the fire adjusters the rate of not less than 200 gallons of water
engaged by appellant insurance company arrived and per minute into the upper story of the
proceeded to examine and photograph the premises, highest building protected, and a trained
pored over the books of the insured and conducted an brigade of not less than 20 men to work the
extensive investigation. The plaintiff having same.'
submitted the corresponding fire claims, totalling
P398,562.81 (but reduced to the full amount of the It is argued that since the bodegas insured had an
insurance, P370,000), the Insurance Company resisted external wall perimeter of 500 meters or 1,640 feet,
payment, claiming violation of warranties and the appellee should have eleven (11) fire hydrants in
conditions, filing of fraudulent claims, and that the the compound, and that he actually had only two (2),
fire had been deliberately caused by the insured or by with a further pair nearby, belonging to the
other persons in connivance with him. municipality of Tabaco.

With counsel for the insurance company acting as We are in agreement with the trial Court that the
private prosecutor, Que Chee Gan, with his brother, appellant is barred by waiver (or rather estoppel) to
Qua Chee Pao, and some employees of his, were claim violation of the so-called fire hydrants
indicted and tried in 1940 for the crime of arson, it warranty, for the reason that knowing fully all that
being claimed that they had set fire to the destroyed the number of hydrants demanded therein never
warehouses to collect the insurance. They were, existed from the very beginning, the appellant
however, acquitted by the trial court in a final neverthless issued the policies in question subject to
decision dated July 9, 1941 (Exhibit WW). Thereafter, such warranty, and received the corresponding
the civil suit to collect the insurance money premiums. It would be perilously close to conniving at
proceeded to its trial and termination in the Court fraud upon the insured to allow appellant to claims
below, with the result noted at the start of this now as void ab initio the policies that it had issued to
opinion. The Philippine National Bank's complaint in the plaintiff without warning of their fatal defect, of
intervention was dismissed because the appellee had which it was informed, and after it had misled the
managed to pay his indebtedness to the Bank during defendant into believing that the policies were
the pendecy of the suit, and despite the fire losses. effective.

In its first assignment of error, the insurance company The insurance company was aware, even before the
alleges that the trial Court should have held that the policies were issued, that in the premises insured
policies were avoided for breach of warranty, there were only two fire hydrants installed by Qua
specifically the one appearing on a rider pasted (with Chee Gan and two others nearby, owned by the
other similar riders) on the face of the policies municipality of TAbaco, contrary to the requirements
(Exhibits X, Y, JJ and LL). These riders were attached of the warranty in question. Such fact appears from
for the first time in 1939, and the pertinent portions positive testimony for the insured that appellant's
read as follows: agents inspected the premises; and the simple denials
of appellant's representative (Jamiczon) can not
Memo. of Warranty. The undernoted overcome that proof. That such inspection was made
Appliances for the extinction of fire being is moreover rendered probable by its being a
kept on the premises insured hereby, and it prerequisite for the fixing of the discount on the
being declared and understood that there is premium to which the insured was entitled, since the
an ample and constant water supply with discount depended on the number of hydrants, and
sufficient pressure available at all seasons the fire fighting equipment available (See "Scale of
for the same, it is hereby warranted that the Allowances" to which the policies were expressly
said appliances shall be maintained in made subject). The law, supported by a long line of
efficient working order during the currency cases, is expressed by American Jurisprudence (Vol.
of this policy, by reason whereof a discount 29, pp. 611-612) to be as follows:
of 2 1/2 per cent is allowed on the premium
chargeable under this policy. It is usually held that where the insurer, at
the time of the issuance of a policy of
Hydrants in the compound, not less in insurance, has knowledge of existing facts
number than one for each 150 feet of which, if insisted on, would invalidate the
external wall measurement of building, contract from its very inception, such
protected, with not less than 100 feet of knowledge constitutes a waiver of conditions
hose piping and nozzles for every two in the contract inconsistent with the facts,
hydrants kept under cover in convenient and the insurer is stopped thereafter from
asserting the breach of such conditions. The premium discount at the minimum of 2 1/2 per cent,
law is charitable enough to assume, in the thereby giving the insurance company a double
absence of any showing to the contrary, that benefit. No reason is shown why appellant's premises,
an insurance company intends to executed a that had been insured with appellant for several years
valid contract in return for the premium past, suddenly should be regarded in 1939 as so
received; and when the policy contains a hazardous as to be accorded a treatment beyond the
condition which renders it voidable at its limits of appellant's own scale of allowances. Such
inception, and this result is known to the abnormal treatment of the insured strongly points at
insurer, it will be presumed to have intended an abuse of the insurance company's selection of the
to waive the conditions and to execute a words and terms of the contract, over which it had
binding contract, rather than to have absolute control.
deceived the insured into thinking he is
insured when in fact he is not, and to have These considerations lead us to regard the parol
taken his money without consideration. (29 evidence rule, invoked by the appellant as not
Am. Jur., Insurance, section 807, at pp. 611- applicable to the present case. It is not a question
612.) here whether or not the parties may vary a written
contract by oral evidence; but whether testimony is
The reason for the rule is not difficult to find. receivable so that a party may be, by reason of
inequitable conduct shown, estopped from enforcing
The plain, human justice of this doctrine is forfeitures in its favor, in order to forestall fraud or
perfectly apparent. To allow a company to imposition on the insured.
accept one's money for a policy of insurance
which it then knows to be void and of no Receipt of Premiums or Assessments afte
effect, though it knows as it must, that the Cause for Forfeiture Other than
assured believes it to be valid and binding, is Nonpayment. It is a well settled rule of law
so contrary to the dictates of honesty and that an insurer which with knowledge of
fair dealing, and so closely related to facts entitling it to treat a policy as no longer
positive fraud, as to the abhorent to in force, receives and accepts a preium on
fairminded men. It would be to allow the the policy, estopped to take advantage of
company to treat the policy as valid long the forfeiture. It cannot treat the policy as
enough to get the preium on it, and leave it void for the purpose of defense to an action
at liberty to repudiate it the next moment. to recover for a loss thereafter occurring and
This cannot be deemed to be the real at the same time treat it as valid for the
intention of the parties. To hold that a literal purpose of earning and collecting further
construction of the policy expressed the true premiums." (29 Am. Jur., 653, p. 657.)
intention of the company would be to indict
it, for fraudulent purposes and designs which It would be unconscionable to permit a
we cannot believe it to be guilty of (Wilson company to issue a policy under
vs. Commercial Union Assurance Co., 96 Atl. circumstances which it knew rendered the
540, 543-544). policy void and then to accept and retain
premiums under such a void policy. Neither
The inequitableness of the conduct observed by the law nor good morals would justify such
insurance company in this case is heightened by the conduct and the doctrine of equitable
fact that after the insured had incurred the expense estoppel is peculiarly applicable to the
of installing the two hydrants, the company collected situation. (McGuire vs. Home Life Ins. Co. 94
the premiums and issued him a policy so worded that Pa. Super Ct. 457.)
it gave the insured a discount much smaller than that
he was normaly entitledto. According to the "Scale of Moreover, taking into account the well known rule
Allowances," a policy subject to a warranty of the that ambiguities or obscurities must be strictly
existence of one fire hydrant for every 150 feet of interpreted aganst the prty that caused them, 1the
external wall entitled the insured to a discount of 7 "memo of warranty" invoked by appellant bars the
1/2 per cent of the premium; while the existence of latter from questioning the existence of the
"hydrants, in compund" (regardless of number) appliances called for in the insured premises, since its
reduced the allowance on the premium to a mere 2 initial expression, "the undernoted appliances for the
1/2 per cent. This schedule was logical, since a extinction of fire being kept on the premises insured
greater number of hydrants and fire fighting hereby, . . . it is hereby warranted . . .", admists of
appliances reduced the risk of loss. But the appellant interpretation as an admission of the existence of
company, in the particular case now before us, so such appliances which appellant cannot now
worded the policies that while exacting the greater contradict, should the parol evidence rule apply.
number of fire hydrants and appliances, it kept the
The alleged violation of the warranty of 100 feet of in favor of the insured, specially to avoid a forfeiture
fire hose for every two hydrants, must be equally (44 C. J. S., pp. 1166-1175; 29 Am. Jur. 180).
rejected, since the appellant's argument thereon is
based on the assumption that the insured was bound Insurance is, in its nature, complex and
to maintain no less than eleven hydrants (one per 150 difficult for the layman to understand.
feet of wall), which requirement appellant is Policies are prepared by experts who know
estopped from enforcing. The supposed breach of the and can anticipate the hearing and possible
wter pressure condition is made to rest on the complications of every contingency. So long
testimony of witness Serra, that the water supply as insurance companies insist upon the use
could fill a 5-gallon can in 3 seconds; appellant of ambiguous, intricate and technical
thereupon inferring that the maximum quantity provisions, which conceal rather than frankly
obtainable from the hydrants was 100 gallons a disclose, their own intentions, the courts
minute, when the warranty called for 200 gallons a must, in fairness to those who purchase
minute. The transcript shows, however, that Serra insurance, construe every ambiguity in favor
repeatedly refused and professed inability to of the insured. (Algoe vs. Pacific Mut. L. Ins.
estimate the rate of discharge of the water, and only Co., 91 Wash. 324, LRA 1917A, 1237.)
gave the "5-gallon per 3-second" rate because the
insistence of appellant's counsel forced the witness to
hazard a guess. Obviously, the testimony is worthless An insurer should not be allowed, by the use
and insufficient to establish the violation claimed, of obscure phrases and exceptions, to defeat
specially since the burden of its proof lay on the very purpose for which the policy was
appellant. procured (Moore vs. Aetna Life Insurance
Co., LRA 1915D, 264).
As to maintenance of a trained fire brigade of 20 men,
the record is preponderant that the same was We see no reason why the prohibition of keeping
organized, and drilled, from time to give, altho not gasoline in the premises could not be expressed
maintained as a permanently separate unit, which the clearly and unmistakably, in the language and terms
warranty did not require. Anyway, it would be that the general public can readily understand,
unreasonable to expect the insured to maintain for his without resort to obscure esoteric expression (now
compound alone a fire fighting force that many derisively termed "gobbledygook"). We reiterate the
municipalities in the Islands do not even possess. rule stated in Bachrach vs. British American Assurance
There is no merit in appellant's claim that subordinate Co. (17 Phil. 555, 561):
membership of the business manager (Co Cuan) in the
fire brigade, while its direction was entrusted to a If the company intended to rely upon a
minor employee unders the testimony improbable. A condition of that character, it ought to have
business manager is not necessarily adept at fire been plainly expressed in the policy.
fighting, the qualities required being different for
both activities. This rigid application of the rule on ambiguities has
become necessary in view of current business
Under the second assignment of error, appellant practices. The courts cannot ignore that nowadays
insurance company avers, that the insured violated monopolies, cartels and concentrations of capital,
the "Hemp Warranty" provisions of Policy No. 2637165 endowed with overwhelming economic power,
(Exhibit JJ), against the storage of gasoline, since manage to impose upon parties dealing with them
appellee admitted that there were 36 cans (latas) of cunningly prepared "agreements" that the weaker
gasoline in the building designed as "Bodega No. 2" party may not change one whit, his participation in
that was a separate structure not affected by the fire. the "agreement" being reduced to the alternative to
It is well to note that gasoline is not specifically take it or leave it" labelled since Raymond Baloilles"
mentioned among the prohibited articles listed in the contracts by adherence" (con tracts d'adhesion), in
so-called "hemp warranty." The cause relied upon by contrast to these entered into by parties bargaining
the insurer speaks of "oils (animal and/or vegetable on an equal footing, such contracts (of which policies
and/or mineral and/or their liquid products having a of insurance and international bills of lading are prime
flash point below 300o Fahrenheit", and is decidedly examples) obviously call for greater strictness and
ambiguous and uncertain; for in ordinary parlance, vigilance on the part of courts of justice with a view
"Oils" mean "lubricants" and not gasoline or kerosene. to protecting the weaker party from abuses and
And how many insured, it may well be wondered, are imposition, and prevent their becoming traps for the
in a position to understand or determine "flash point unwarry (New Civil Coee, Article 24; Sent. of Supreme
below 003o Fahrenheit. Here, again, by reason of the Court of Spain, 13 Dec. 1934, 27 February 1942).
exclusive control of the insurance company over the
terms and phraseology of the contract, the ambiguity Si pudiera estimarse que la condicion 18 de
must be held strictly against the insurer and liberraly la poliza de seguro envolvia alguna
oscuridad, habra de ser tenido en cuenta que in use by the insured) and receipts was positively
al seguro es, practicamente un contrato de unreasonable, considering that such listing was
los llamados de adhesion y por consiguiente superfluous because the insurer was not denied access
en caso de duda sobre la significacion de las to the records, that the volume of Qua Chee Gan's
clausulas generales de una poliza business ran into millions, and that the demand was
redactada por las compafijas sin la made just after the fire when everything was in
intervencion alguna de sus clientes se ha turmoil. That the representatives of the insurance
de adoptar de acuerdo con el articulo 1268 company were able to secure all the date they needed
del Codigo Civil, la interpretacion mas is proved by the fact that the adjuster Alexander
favorable al asegurado, ya que la obscuridad Stewart was able to prepare his own balance sheet
es imputable a la empresa aseguradora, que (Exhibit L of the criminal case) that did not differ
debia haberse explicado mas claramante. from that submitted by the insured (Exhibit J) except
(Dec. Trib. Sup. of Spain 13 Dec. 1934) for the valuation of the merchandise, as expressly
found by the Court in the criminal case for arson.
The contract of insurance is one of perfect good faith (Decision, Exhibit WW).
(uferrimal fidei) not for the insured alone, but
equally so for the insurer; in fact, it is mere so for the How valuations may differ honestly, without fraud
latter, since its dominant bargaining position carries being involved, was strikingly illustrated in the
with it stricter responsibility. decision of the arson case (Exhibit WW) acquiting Qua
Choc Gan, appellee in the present proceedings. The
Another point that is in favor of the insured is that the decision states (Exhibit WW, p. 11):
gasoline kept in Bodega No. 2 was only incidental to
his business, being no more than a customary 2 day's Alexander D. Stewart declaro que ha
supply for the five or six motor vehicles used for examinado los libros de Qua Choc Gan en
transporting of the stored merchandise (t. s. n., pp. Tabaco asi como su existencia de copra y
1447-1448). "It is well settled that the keeping of abaca en las bodega al tiempo del incendio
inflammable oils on the premises though prohibited durante el periodo comprendido desde el 1.o
by the policy does not void it if such keeping is de enero al 21 de junio de 1940 y ha
incidental to the business." Bachrach vs. British encontrado que Qua Choc Gan ha sufrico una
American Ass. Co., 17 Phil. 555, 560); and "according perdida de P1,750.76 en su negocio en
to the weight of authority, even though there are Tabaco. Segun Steward al llegar a este
printed prohibitions against keeping certain articles conclusion el ha tenidoen cuenta el balance
on the insured premises the policy will not be avoided de comprobacion Exhibit 'J' que le ha
by a violation of these prohibitions, if the prohibited entregado el mismo acusado Que Choc Gan
articles are necessary or in customary use in carrying en relacion con sus libros y lo ha encontrado
on the trade or business conducted on the premises." correcto a excepcion de los precios de abaca
(45 C. J. S., p. 311; also 4 Couch on Insurance, section y copra que alli aparecen que no estan de
966b). It should also be noted that the "Hemp acuerdo con los precios en el mercado. Esta
Warranty" forbade storage only "in the building to comprobacion aparece en el balance
which this insurance applies and/or in any building mercado exhibit J que fue preparado por el
communicating therewith", and it is undisputed that mismo testigo.
no gasoline was stored in the burned bodegas, and
that "Bodega No. 2" which was not burned and where In view of the discrepancy in the valuations between
the gasoline was found, stood isolated from the other the insured and the adjuster Stewart for the insurer,
insured bodegas. the Court referred the controversy to a government
auditor, Apolonio Ramos; but the latter reached a
The charge that the insured failed or refused to different result from the other two. Not only that, but
submit to the examiners of the insurer the books, Ramos reported two different valuations that could
vouchers, etc. demanded by them was found be reached according to the methods employed
unsubstantiated by the trial Court, and no reason has (Exhibit WW, p. 35):
been shown to alter this finding. The insured gave the
insurance examiner all the date he asked for (Exhibits La ciencia de la contabilidad es buena, pues
AA, BB, CCC and Z), and the examiner even kept and ha tenido sus muchos usos buenos para
photographed some of the examined books in his promovar el comercio y la finanza, pero en
possession. What does appear to have been rejected el caso presente ha resultado un tanto
by the insured was the demand that he should submit cumplicada y acomodaticia, como lo prueba
"a list of all books, vouchers, receipts and other el resultado del examen hecho por los
records" (Age 4, Exhibit 9-c); but the refusal of the contadores Stewart y Ramos, pues el juzgado
insured in this instance was well justified, since the no alcanza a ver como habiendo examinado
demand for a list of all the vouchers (which were not las mismas partidas y los mismos libros
dichos contadores hayan de llegara dos copra and hemp stated in the fire claims, the insurer's
conclusiones que difieron sustancialmente case rests almost exclusively on the estimates,
entre si. En otras palabras, no solamente la inferences and conclusions of its adjuster
comprobacion hecha por Stewart difiere de investigator, Alexander D. Stewart, who examined the
la comprobacion hecha por Ramos sino que, premises during and after the fire. His testimony,
segun este ultimo, su comprobacion ha dado however, was based on inferences from the
lugar a dos resultados diferentes photographs and traces found after the fire, and must
dependiendo del metodo que se emplea. yield to the contradictory testimony of engineer
Andres Bolinas, and specially of the then Chief of the
Clearly then, the charge of fraudulent overvaluation Loan Department of the National Bank's Legaspi
cannot be seriously entertained. The insurer branch, Porfirio Barrios, and of Bank Appraiser Loreto
attempted to bolster its case with alleged Samson, who actually saw the contents of the bodegas
photographs of certain pages of the insurance book shortly before the fire, while inspecting them for the
(destroyed by the war) of insured Qua Chee Gan mortgagee Bank. The lower Court was satisfied of the
(Exhibits 26-A and 26-B) and allegedly showing veracity and accuracy of these witnesses, and the
abnormal purchases of hemp and copra from June 11 appellant insurer has failed to substantiate its charges
to June 20, 1940. The Court below remained aganst their character. In fact, the insurer's repeated
unconvinced of the authenticity of those photographs, accusations that these witnesses were later
and rejected them, because they were not mentioned "suspended for fraudulent transactions" without giving
not introduced in the criminal case; and considering any details, is a plain attempt to create prejudice
the evident importance of said exhibits in establishing against them, without the least support in fact.
the motive of the insured in committing the arson
charged, and the absence of adequate explanation for Stewart himself, in testifying that it is impossible to
their omission in the criminal case, we cannot say that determine from the remains the quantity of hemp
their rejection in the civil case constituted reversible burned (t. s. n., pp. 1468, 1470), rebutted appellant's
error. attacks on the refusal of the Court below to accept its
inferences from the remains shown in the photographs
The next two defenses pleaded by the insurer, that of the burned premises. It appears, likewise, that the
the insured connived at the loss and that the adjuster's calculations of the maximum contents of
fraudulently inflated the quantity of the insured stock the destroyed warehouses rested on the assumption
in the burnt bodegas, are closely related to each that all the copra and hemp were in sacks, and on the
other. Both defenses are predicted on the assumption result of his experiments to determine the space
that the insured was in financial difficulties and set occupied by definite amounts of sacked copra. The
the fire to defraud the insurance company, error in the estimates thus arrived at proceeds from
presumably in order to pay off the Philippine National the fact that a large amount of the insured's stock
Bank, to which most of the insured hemp and copra were in loose form, occupying less space than when
was pledged. Both defenses are fatally undermined by kept in sacks; and from Stewart's obvious failure to
the established fact that, notwithstanding the give due allowance for the compression of the
insurer's refusal to pay the value of the policies the material at the bottom of the piles (t. s. n., pp. 1964,
extensive resources of the insured (Exhibit WW) 1967) due to the weight of the overlying stock, as
enabled him to pay off the National Bank in a short shown by engineer Bolinas. It is probable that the
time; and if he was able to do so, no motive appears errors were due to inexperience (Stewart himself
for attempt to defraud the insurer. While the admitted that this was the first copra fire he had
acquittal of the insured in the arson case is not res investigated); but it is clear that such errors render
judicata on the present civil action, the insurer's valueles Stewart's computations. These were in fact
evidence, to judge from the decision in the criminal twice passed upon and twice rejected by different
case, is practically identical in both cases and must judges (in the criminal and civil cases) and their
lead to the same result, since the proof to establish concordant opinion is practically conclusive.
the defense of connivance at the fire in order to
defraud the insurer "cannot be materially less The adjusters' reports, Exhibits 9-A and 9-B, were
convincing than that required in order to convict the correctly disregarded by the Court below, since the
insured of the crime of arson"(Bachrach vs. British opinions stated therein were based on ex parte
American Assurance Co., 17 Phil. 536). investigations made at the back of the insured; and
the appellant did not present at the trial the original
As to the defense that the burned bodegas could not testimony and documents from which the conclusions
possibly have contained the quantities of copra and in the report were drawn.lawphi1.net
hemp stated in the fire claims, the insurer's case rests
almost exclusively on the estimates, inferences and Appellant insurance company also contends that the
conclusionsAs to the defense that the burned bodegas claims filed by the insured contained false and
could not possibly have contained the quantities of fraudulent statements that avoided the insurance
policy. But the trial Court found that the We find no reversible error in the judgment appealed
discrepancies were a result of the insured's erroneous from, wherefore the smae is hereby affirmed. Costs
interpretation of the provisions of the insurance against the appellant. So ordered.
policies and claim forms, caused by his imperfect
knowledge of English, and that the misstatements Colado v. Insular Life - Tender of Overdue Payments
were innocently made and without intent to defraud.
Our review of the lengthy record fails to disclose
reasons for rejecting these conclusions of the Court 51 OG (No 12) 6269
below. For example, the occurrence of previous fires
in the premises insured in 1939, altho omitted in the Facts:
claims, Exhibits EE and FF, were nevertheless
revealed by the insured in his claims Exhibits Q (filed > Vivencio Collado applied for an insurance contract
simultaneously with them), KK and WW. Considering with Insular life in 1948. His application was
that all these claims were submitted to the smae approved and he began started making premium
agent, and that this same agent had paid the loss payments. However, he defaulted and the insurance
caused by the 1939 fire, we find no error in the trial was cancelled.
Court's acceptance of the insured's explanation that
the omission in Exhibits EE and FF was due to
inadvertance, for the insured could hardly expect > He then applied for the reinstatement of his
under such circumstances, that the 1939 would pass insurance policy in Nov. of 1951 and tendered the
unnoticed by the insurance agents. Similarly, the 20 amount of premium for the years 1950-1951.
per cent overclaim on 70 per cent of the hemo stock,
was explained by the insured as caused by his belief > He stated that he was as of Nov. 1951 of good
that he was entitled to include in the claim his health, and that he had no injuries, ailments or
expected profit on the 70 per cent of the hemp, illnesses and had not been sick for any case since 1948
because the same was already contracted for and sold (his medical check up when he applied for insurance)
to other parties before the fire occurred. Compared and that he had not consulted any physician or
with other cases of over-valuation recorded in our practitioner for any case since the date of such latest
judicial annals, the 20 per cent excess in the case of medical exam.
the insured is not by itself sufficient to establish
fraudulent intent. Thus, in Yu Cua vs. South British > However, when Vivencio applied for the
Ins. Co., 41 Phil. 134, the claim was fourteen (14) reinstatement, he was already sick of a fatal disease
times (1,400 per cent) bigger than the actual loss; in known as carcinoma of the liver and that 4 days prior
Go Lu vs. Yorkshire Insurance Co., 43 Phil., 633, eight to his application for insurance, he consulted a doctor
(8) times (800 per cent); in Tuason vs. North China regarding his condition.
Ins. Co., 47 Phil. 14, six (6) times (600 per cent); in
Tan It vs. Sun Insurance, 51 Phil. 212, the claim
totalled P31,860.85 while the goods insured were > The reinstatement was approved. Vivencio again
inventoried at O13,113. Certainly, the insured's failed to pay the premiums for the last quarter of Nov.
overclaim of 20 per cent in the case at bar, duly 1951 and as such, Insular life sent him a notice
explained by him to the Court a quo, appears puny by canceling the policy.
comparison, and can not be regarded as "more than
misstatement, more than inadvertence of mistake, > Vivencio then died. The beneificiaries instituted
more than a mere error in opinion, more than a slight the present action to recover from Insular life the
exaggeration" (Tan It vs. Sun Insurance Office, ante) death benefits of a life insurance policy valued at
that would entitle the insurer to avoid the policy. It 2T. Insular refused to pay claiming concealment on
is well to note that the overchange of 20 per cent was the part of Vivencio.
claimed only on a part (70 per cent) of the hemp
stock; had the insured acted with fraudulent intent, > Collado contends that Insular life had waived the
nothing prevented him from increasing the value of right to rescine the policy in view of its repeated
all of his copra, hemp and buildings in the same acceptance of the overdue premiums for the second
proportion. This also applies to the alleged fraudulent and third years.
claim for burned empty sacks, that was likewise
explained to our satisfaction and that of the trial
Court. The rule is that to avoid a policy, the false > Municipal court of Manila found for Collado and
swearing must be wilful and with intent to defraud Insular filed an appeal with CFI of Manila. CFI
(29 Am. Jur., pp. 849-851) which was not the cause. rendered judgment in favor of Insular and dismissed
Of course, the lack of fraudulent intent would not Collados complaint.
authorize the collection of the expected profit under
the terms of the polices, and the trial Court correctly Issue:
deducte the same from its award.
Whether or nor Insular life was estopped and could no
longer cancel the contract due to the fact that it
accepted the tender of overdue payments from
Vivencio.

Held:

NO.

It is enormously clear that when the deceased


applied for a reinstatement of his policy in Nov. 1951,
he had already been afflicted with the fatal ailment
for a period of about four months. Furthermore, in
submitting together with his application for
reinstatement, a health statement to the effect that
he was in good health, Vivencio concealed the
material fact that he had consulted a doctor and was
then found to be afflicted with the malady.

The acceptance of Insular life of the overdue


premiums did not necessarily deprive it of the right to
cancel the policy in case of default incurred by the
Insured in the payment of future premiums. The case
would be different had the insured died at any time
after the payment of overdue premiums but previous
to the reinstatement of the policy, for the, Insular, by
its acceptance of its overdue premiums is deemed to
have waived its right to rescind the policy.

The evidence at hand shows that insofar as the


payment of the last quarterly premium for 1951 was
concerned, Insular had availed of the right to rescind
the policy by notifying the Insured that the policy had
lapsed.

Вам также может понравиться